Você está na página 1de 114

MedCosmos Surgery

Surgery Lecture Notes, Books, MCQ and Good Articles

SATURDAY, SEPTEMBER 6, 2008


Liver and Pancreas MCQ
1. Which of the following statements about the segmental anatomy of the liver are not true?
A. Segments are subdivisions in both the French and American systems.
B. Segments are determined primarily by the hepatic venous drainage.
C. The French anatomic system is more applicable than the American system to clinical hepatic
resection.
D. Segments are important to the understanding of the topographic anatomy of the liver.
Answer: D

DISCUSSION: Segments are the major subdivision of the right and left lobes of the liver. In either the
classic lobar (American) or the segmental (French) system, the most variable aspect is the biliary
system. Therefore the hepatic venous or portal system defines most segments. The French system
depicts eight segments, with the caudate lobe as segment I and the other seven segments defined
primarily by the hepatic venous system. Segments are not well-depicted by topography.

2. Which of the following anatomic features of the biliary system are important considerations in
operative cholangiography?
A. The left hepatic duct comes off farther anterior than the right one.
B. At the confluence there may be more than just a right and a left hepatic duct.
C. Dissection of the triangle of Calot is more important than cholangiography in preventing bile duct
injury.
D. Segments V, VII, or VIII sometimes join the biliary system below the confluence.
Answer: ABCD

DISCUSSION: All of these features are important. The angle of takeoff of the left hepatic duct may
make operative visualization difficult with the patient in the supine position. Because there may be
more than two major hepatic ducts, visualization of two large ducts does not ensure that the system is
normal. Ducts from any of the right-side segments can join below the confluence. Dissecting one
superior edge of the gallbladder before it joins the cystic duct is particularly important in preventing
injury. 

3. The hepatic artery:


A. Supplies the same amount of blood to the liver as the portal vein.
B. Provides more blood to the bile ducts than the portal vein.
C. Is autoregulated just as the portal vein is.
D. Supplies most of the blood to hepatic metastases.
Answer: BD

DISCUSSION: The portal vein provides two thirds to three quarters of the total hepatic blood flow. The
portal vein is incapable of direct autoregulation. The hepatic artery after transplantation classically
infarcts portions of the biliary system, whereas hepatic metastases often arrive there via the portal
vein. Most of their blood supply comes from the hepatic artery.

4. Bile formation is:


A. An active secretory process.
B. Determined at two sites principally.
C. Regulated physiologically by hormones.
D. Largely determined by the intactness of the enterohepatic circulation (EHC).
Answer: ABCD

DISCUSSION: Bile formation is an active process at both the canalicular and ductular sites. The
paracellular pathway probably plays a minor physiologic role. Secretin and glucagon are likely
physiologic regulators of biliary secretion. Bile salts are extremely important and are probably the most
important agent in the changes that occur when enterohepatic circulation is interrupted.

5. Generally, the two most important hepatic functions to consider after hepatic resection are:
A. Hepatic synthetic function.
B. Glucose metabolism.
C. The liver's role in lipid metabolism.
D. The liver's role in vitamin metabolism.
Answer: AB
DISCUSSION: While other functions undoubtedly may be important postoperatively, the most common
abnormalities occurring after a major hepatic resection are related to loss of protein synthesis and
consequences of glucose metabolism. Therefore, it is usually advisable to administer supplemental
amounts of protein and sugar postoperatively.

6. Which of the following statements about pyogenic abscess of the liver are true?
A. The right lobe is more commonly involved than the left lobe.
B. Appendicitis with perforation and abscess is the most common underlying cause of hepatic abscess.
C. Mortality is largely determined by the underlying disease.
D. Mortality from hepatic abscess is currently greater than 40%.
Answer: AC

DISCUSSION: Involvement of the right lobe with abscess formation approximates 70% of pyogenic
abscesses. This is thought to be due to the streaming effect of superior mesenteric venous inflow to
the right lobe. In addition, the greater volume of the right lobe predisposes more tissue to seeding by
bacterial organisms. While appendicitis comprised 25% to 40% of cases in early series, early recognition
and operative therapy for appendicitis have reduced its importance significantly. In current series,
malignant or benign biliary obstruction is the underlying cause of 35% to 50% of cases. Recent studies
have shown that the underlying disease or an immunocompromised host is more important
prognostically than solitary versus multiple abscesses.

7. Which of the following statements most accurately describes the current therapy for pyogenic
hepatic abscess?
A. Antibiotics alone are adequate for the treatment of most cases.
B. All patients require open surgical drainage for optimal management.
C. Optimal treatment involves treatment of not only the abscess but the underlying source as well.
D. Percutaneous drainage is more successful for multiple lesions than for solitary ones.
Answer: C

DISCUSSION: The development of ultrasonography and computed tomography (CT) in the past two
decades has enabled earlier diagnosis and advances in treatment of hepatic abscess. Formerly, open
surgical drainage was considered necessary in essentially all cases of pyogenic abscess. Numerous
recent series, however, have reported high success rates and low mortality from the percutaneous
catheter drainage of abscesses under CT or ultrasonographic guidance. Optimal management of
pyogenic abscess, however, involves not only treatment of the abscess, whether by percutaneous or
surgical methods, but correction of the underlying source as well. All modes of therapy are more
successful in treating solitary lesions than multiple ones.

8. Which of the following statements characterize amebic abscess?


A. Mortality is higher than that for similarly located pyogenic abscesses.
B. The diagnosis of amebic abscess may be based on serologic tests and resolution of symptoms.
C. In contrast to pyogenic abscess, the treatment of amebic abscess is primarily medical.
D. Patients with amebic abscess tend to be older than those with pyogenic abscess.
BC

DISCUSSION: Mortality for uncomplicated amebic abscess should be less than 5%, in contrast to the 15%
to 20% rate for pyogenic abscess. After the demonstration by radiologic examination of an abscess,
appropriate serologic tests and resolution of symptoms after a course of treatment with an antiamebic
agent such as metronidazole constitute presumptive diagnosis of amebic abscess. Aspiration of abscess
contents rarely yields amebic organisms. In contrast to pyogenic abscess, amebic abscess rarely
requires surgical or percutaneous drainage, except in the case of an extremely large abscess or
bacterial superinfection. Amebic abscess affects males in a 9:1 to 10:1 ratio and generally affects a
younger population than pyogenic abscess. Additionally, in the United States the populations most
affected are immigrants from endemic areas such as Mexico or Latin America and American tourists to
those regions.

9. Which of the following statement(s) is/are true about benign lesions of the liver?
A. Adenomas are true neoplasms with a predisposition for complications and should usually be
resected.
B. Focal nodular hyperplasia (FNH) is a neoplasm related to birth control pills (BCPs) and usually
requires resection.
C. Hemangiomas are the most common benign lesions of the liver that come to the surgeon's attention.
D. Nodular regenerative hyperplasia does not usually accompany cirrhosis.
Answer: A

DISCUSSION: Adenomas typically enlarge and cause symptoms, may rupture, and have a definite
malignant potential. Therefore they should generally be resected when found. FNH is not a true
neoplasm and generally has an uneventful course. Both are related to BCPs, although the relationship
of adenoma is more firmly established. While small bile duct hamartomas are much more common,
hemangiomas are the most common lesion to come to the attention of surgeons. They should not
generally be biopsied because of possible hemorrhage. By definition, nodular regenerative hyperplasia
occurs in the absence of cirrhosis.

10. Which of the following statement(s) about malignant neoplasms of the liver is/are true?
A. Hepatocellular carcinoma is probably the number 1 cause of death from cancers worldwide.
B. The most common resectable hepatic malignant neoplasm in the United States is colorectal
metastasis.
C. Hepatoma has at least one variant that has a much more benign course than hepatomas in general.
D. Hepatomas are generally slower growing than was formerly believed.
Answer: ABCD

DISCUSSION: Although exact comparisons are impossible, hepatoma seems to be the most common
cause of cancer death worldwide, despite its relative infrequency in the United States. Colorectal
metastasis is a more common indication for surgical treatment in the United States. The fibrolamellar
variant and possibly the very well-differentiated tumor probably have a better prognosis than
hepatomas in general. Previous studies from Africa in which there was a high incidence of rupture
account for the poor prognosis that was generally attributed to hepatoma. Recent studies from Europe
and the United States have shown that survival after presentation is usually measured in years.

11. Which of the following statement(s) is/are true about bile duct cancers?
A. If resected, proximal lesions are usually curable.
B. The more proximal the lesion, the more likely is resection to be curative.
C. Radiation clearly prolongs survival.
D. Transplantation is usually successful if the lesion seems confined to the liver.
E. None of the above is true.
Answer: E

DISCUSSION: Most bile duct cancers are discovered after they are incurable, and only a tiny subset of
resected proximal lesions are cured. The more distal the lesion, the more likely is resection to achieve
cure (e.g., approximately 30% 5-year survival for periampullary lesions as compared with 0% to 10% for
hilar lesions). The use of adjuvant or primary radiation remains controversial because of the
heterogeneity of the patient populations on which this modality has been used. Because of the
localized nature of this disease it would seem that transplantation would produce favorable results;
however, this has not been the case.

12. Echinococcosis liver disease caused by Echinococcus granulosus:


A. Is not a neoplasm.
B. Is endemic to parts of Europe, but not the United States.
C. Is usually curable by resection.
D. Is more deadly than in its Echinococcus multilocularis form.
Answer: ABC

DISCUSSION: The parasitic infection is fairly common in certain parts of Europe but very rare in the
United States. Resection without peritoneal soilage is the treatment of choice. The E. multilocularis
form, which is endemic to parts of the United States, is more likely to be fatal because it is rarely
resectable. This form is more likely to resemble a malignancy than E. granulosus, although the natural
course of the disease usually spans many years.

13. Which of the following statements about hemobilia are true?


A. Tumors are the most common cause.
B. The primary treatment of severe hemobilia is an operation.
C. Percutaneous cholangiographic hemobilia is usually minor.
D. Ultrasonography usually reveals a specific diagnosis.
Answer: C

DISCUSSION: By far the most common cause of hemobilia is trauma. Tumors also may cause the
syndrome but are relatively uncommon causes. For severe hemobilia the best therapy is arteriographic
embolization. Usually the site of bleeding or a false aneurysm can be identified. Operation should be
reserved as a last resort or when the condition is recognized intraoperatively. Percutaneous
cholangiography–associated intrabiliary hemorrhage is usually, but not always, minor and self-limiting.
Ultrasonography is a very nonspecific diagnostic technique for hemobilia. Arteriography remains the
best diagnostic method.

14. Ligation of all of the following arteries usually causes significant hepatic enzyme abnormalities
except:
A. Ligation of the right hepatic artery.
B. Ligation of the left hepatic artery.
C. Ligation of the hepatic artery distal to the gastroduodenal branch.
D. Ligation of the hepatic artery proximal to the gastroduodenal artery.
Answer: D

DISCUSSION: Ligation of the right or left hepatic artery frequently causes enzyme elevation but is
usually tolerated by the patient, particularly when this is a life-saving maneuver. Ligation of the
hepatic artery distal to the gastroduodenal branch is more risky but is also usually tolerated. Ligation
of the hepatic artery proximal to the gastroduodenal one does not normally cause enzyme
abnormalities because of abundant collateral flow through that branch.

15. Which of the following is the most common acid-base disturbance in patients with cirrhosis and
portal hypertension?
A. Metabolic acidosis.
B. Respiratory alkalosis.
C. Metabolic alkalosis.
D. Respiratory acidosis.
Answer: C

DISCUSSION: Metabolic alkalosis and hypokalemia are common in patients with cirrhosis because they
often have associated secondary hyperaldosteronism (especially those with ascites), diarrhea, and
frequent emesis. Hyperaldosteronism enhances H+ and K+ exchange for Na+ in the distal tubule of the
kidney. The cause of diarrhea in patients with cirrhosis is unknown, but malabsorption secondary to
splanchnic venous hypertension may be a contributing factor. Emesis is common in alcoholic cirrhotics
and patients with tense ascites. Deleterious effects of metabolic alkalosis include impaired tissue
oxygen delivery secondary to shift of the oxyhemoglobin dissociation curve to the left and conversion
of ammonium chloride to ammonia, which may contribute to encephalopathy.

16. A portal venous pressure of 30 mm. Hg (elevated) and a hepatic venous wedge pressure of 5 mm.
Hg (normal) may be associated with which of the following causes of portal hypertension?
A. Portal vein thrombosis.
B. Alcoholic cirrhosis.
C. Schistosomiasis.
D. Alcoholic hepatitis.
Answer: AC

DISCUSSION: Pressure measured by wedging a catheter into a hepatic vein (hepatic venous wedge
pressure) closely correlates with directly measured portal venous pressure in patients with portal
hypertension when the site of elevated resistance is at the sinusoidal or postsinusoidal level. Such is
the case in alcoholic cirrhosis and alcoholic hepatitis. When the site of increased resistance is at the
presinusoidal level, either within (schistosomiasis) or outside (portal vein thrombosis) the liver, the
hepatic venous wedge pressure is normal despite markedly elevated portal vein pressure. Although
schistosomiasis is one of the more frequent causes of portal hypertension worldwide, in North America
presinusoidal portal hypertension is considerably less common than alcoholic liver disease. A normal
hepatic venous wedge pressure in a patient who has bled from varices should lead one to suspect a
presinusoidal cause. A specific diagnosis can often be made by visceral angiography or liver biopsy.

17. Which of the following is the most effective definitive therapy for both prevention of recurrent
variceal hemorrhage and control of ascites?
A. Endoscopic sclerotherapy.
B. Distal splenorenal shunt.
C. Esophagogastric devascularization (Sugiura procedure).
D. Side-to-side portacaval shunt.
E. End-to-side portacaval shunt.
Answer: D

DISCUSSION: Shunt operations are the most effective means of preventing recurrent variceal
hemorrhage. Rebleeding rates after endoscopic sclerotherapy range from 40% to 60%. Although
extensive esophagogastric devascularization has effectively prevented recurrent bleeding in Japanese
series, these operations have been followed by rebleeding rates in excess of 25% in most Western
series. Although one controlled trial has shown more frequent recurrent hemorrhage following the
distal splenorenal shunt than after the portacaval shunt, most series have reported rebleeding rates of
less than 10% for both of these operations. Both the liver and the splanchnic viscera are important sites
of ascites formation. Since the distal splenorenal shunt maintains sinusoidal and mesenteric venous
hypertension and requires interruption of important retroperitoneal lymphatics, it tends to aggravate
rather than relieve ascites. Hepatic sinusoidal pressure may be unchanged or even increased after an
end-to-side portacaval shunt. Only side-to-side portal-systemic shunts, such as the side-to-side
portacaval shunt, reliably decompress both the liver and splanchnic viscera, thus preventing ascites
formation.

18. Which of the following treatments most effectively preserves hepatic portal perfusion?
A. Distal splenorenal shunt.
B. Conventional splenorenal shunt.
C. Endoscopic sclerotherapy.
D. Side-to-side portacaval shunt.
Answer: C

DISCUSSION: The conventional splenorenal shunt and side-to-side portacaval shunts completely divert
portal flow away from the liver (nonselective shunts). The distal splenorenal shunt is a selective shunt
that preserves hepatic portal perfusion in the majority of patients; however, the magnitude of portal
flow is decreased because the gastrosplenic component is diverted into the renal vein. Additionally,
many patients (especially alcoholic cirrhotics) develop collaterals between the mesenteric venous
circulation and the shunt, resulting in gradual attrition of the remaining portal flow. Although there
have been anecdotal reports of portal vein thrombosis after endoscopic sclerotherapy, two controlled
trials have demonstrated better preservation of hepatic portal perfusion in sclerotherapy patients than
in persons who receive the distal splenorenal shunt.

19. Which of the following veins is preserved in performing the extensive esophagogastric
devascularization procedure described by Sugiura?
A. Left gastric (coronary) vein.
B. Short gastric vein.
C. Splenic vein.
D. Left gastroepiploic vein.
Answer: A

DISCUSSION: The Sugiura procedure consists of devascularization of the esophagus to the inferior
pulmonary vein and the proximal two thirds of the stomach, splenectomy, and distal esophageal
transection. The devascularization component should be done as close to the esophagus and stomach
as possible. The coronary vein and paraesophageal collaterals are preserved to maintain an effective
portal-systemic collateral pathway and thereby discourage reformation of varices.
20. Which of the following complications of portal hypertension often require surgical intervention (for
more than 25% of patients)?
A. Hypersplenism.
B. Variceal hemorrhage.
C. Ascites.
D. Encephalopathy.
Answer: B

DISCUSSION: While many patients with portal hypertension develop hypersplenism, it is rarely clinically
significant. A splenectomy should not be performed unless platelet counts are persistently less than
20,000 per cu. mm. or white blood cell counts are less than 1200 per cu. mm. Unfortunately,
splenectomy is sometimes done for clinically insignificant hypersplenism, thus obviating a distal
splenorenal shunt if the patient should subsequently bleed from varices. The initial treatment for most
patients with bleeding esophageal varices should be endoscopic sclerotherapy; however, operation is
required for the approximately one third of patients who fail sclerotherapy and for noncompliant
persons, those living in remote geographic locations, and patients bleeding from gastric varices. Ascites
can be controlled by a medical regimen of dietary salt restriction and diuretic therapy in more than
95% of patients. When ascites is intractable to medical management, either intermittent large-volume
paracenteses or a surgical peritoneovenous shunt should be done. With rare exceptions,
encephalopathy should be treated medically. Most important is elimination of any precipitating factors
that led to the neuropsychological disturbance. Lactulose, neomycin, and dietary protein restriction
may also be components of the medical treatment regimen.

21. Which of the following effects are advantages of combined vasopressin and nitroglycerin
intravenous infusion, as compared with vasopressin infusion alone, in controlling acute variceal
bleeding?
A. Lower frequency of encephalopathy.
B. Lower incidence of vasopressin side effects.
C. More effective control of bleeding.
D. Less “rebound effect” when discontinuing the infusion.
Answer: BC

DISCUSSION: Vasopressin acts through vasoconstriction of splanchnic arterioles. Both portal venous
inflow and portal venous pressure are reduced, resulting in control of acute variceal bleeding in
approximately 50% of patients. However, the adverse side effects of systemic hypertension,
bradycardia, decreased cardiac output, and coronary vasoconstriction are quite common during
vasopressin infusion. Simultaneous administration of nitroglycerin or nitroprusside eliminates these side
effects—and in one controlled trial enhanced therapeutic effectiveness. Although the mechanism of
action of this combined infusion is not clear, vasodilation of portal-systemic collaterals, resulting in a
further reduction in portal pressure, may be responsible.

22. Which of the following statements about the peritoneovenous shunt (PVS) is/are correct?
A. For cirrhotic patients with intractable ascites, the LeVeen shunt is an effective “bridge” to liver
transplantation.
B. Replacement of ascites with saline or lactated Ringer's solution reduces the coagulopathy following
PVS.
C. For patients with cirrhotic ascites, the survival using repeated paracentesis with 5% albumin infusion
is equivalent to that with the PVS.
D. Oliguria (less than 25 ml. per hour) in the immediate postoperative period following PVS should be
treated with a 5% albumin infusion.
E. The transjugular intrahepatic portacaval shunt with stent (TIPSS) works on the same principle as the
PVS.
Answer: BC

DISCUSSION: The PVS is a palliative procedure that does not prolong life. In comparing the early risks of
the procedure with those of repeated paracentesis, the shunt cannot be justified as a temporizing
procedure to facilitate ascites control in the patient awaiting liver transplantation. Oliguria is common
in the first 24 hours after shunt insertion. A correctly placed PVS (patency confirmed using an
intraoperative “shuntogram”) expands the intravascular volume with a continuous reinfusion of ascites.
Inspection should identify elevation of the jugular venous pressure, and a diuretic (usually furosemide)
is needed. The mechanisms of action of the two shunts are very different. TIPSS reduces portal
pressure and controls ascites by reducing the rate of ascites formation. PVS reinfuses the ascites fluid,
thereby reducing the prerenal stimulus to sodium retention and making the patient more responsive to
diuretic therapy.

23. Which of the following clinical situations are considered good indications for PVS?
A. A 50-year-old cirrhotic man had an emergency portacaval shunt for bleeding varices and
postoperatively had an ascites leak and mild superficial wound infection.
B. A 57-year-old woman with primary biliary cirrhosis (PBC) has difficult to control ascites and diuretic-
induced encephalopathy.
C. A 46-year-old resistant alcoholic has chronic ascites uncontrolled by diuretics combined with repeat
paracentesis.
D. A 34-year-old woman taking BCPs had rapid onset of ascites and is found to have hepatic vein
thrombosis causing the Budd-Chiari syndrome.
Answer: C

DISCUSSION: Because of the high complication rate and the long-term failure rate, the PVS is used only
when other, more lasting options for therapy either are not available or are contraindicated. The
chronic alcoholic patient may benefit from a peritoneovenous shunt because his ascites is the dominant
problem related to his chronic liver disease, and persistent alcoholism is a contraindication to liver
replacement in most centers. PVS may be quite effective for the temporary management of acute
intractable postoperative ascites, such as in patient A; however, it is absolutely contraindicated in the
presence of infection. Patient B has ascites as her dominant problem as well; however, with PBC as the
underlying liver disease, she is an excellent candidate for transplantation. Patient D also has ascites as
the major problem; however, the side-to-side portosystemic shunt is a far better long-term treatment
option than PVS.

24. Which of the following explanations account(s) for the fact that hepatitis C is the most common
cause of posttransfusion hepatitis?
A. There are more carriers of hepatitis C virus (HCV) in the normal population who serve as blood
donors.
B. Blood infected with hepatitis B virus (HBV) is eliminated through routine testing, leaving only HCV as
the other blood-borne pathogen.
C. Current serologic tests for HCV antigen do not exclude carriers.
D. Questions designed to eliminate risk groups for HCV from the normal donor population may not be as
specific as would be desirable.
E. Hepatitis C is a more virulent form of viral hepatitis, so it is expected that more cases of
posttransfusion hepatitis would occur.
Answer: BD

DISCUSSION: The ability to specifically identify persons infected with HCV has only recently become
available. Therefore, data about epidemiology are less than complete. It is very likely not true that
more blood donors carry HCV because of the large preponderance of HBV in the United States. It is
true, however, that successful elimination of most of the HBV carriers occurs through routine testing.
Although serologic tests are available for HCV, they are tests, not of antigen, but of antibody.
Therefore, this test alone may not screen out persons who are infected but have not yet developed or
may never develop antibody. Risk groups for the relatively newly defined HCV may well not be
comprehensively established, and therefore this explanation may be a contributor. There are no
differences in virulence between these classes of hepatitis virus.

25. True or false: HBV infections:


A. Are usually asymptomatic.
B. May not be clinically recognized but may lead to chronic hepatitis.
C. Reliably protect against subsequent HBV infection regardless of the measured antibody titer to
hepatitis B surface antigen (HBsAg).
D. Are completely prevented by postexposure administration of HBIg hepatitis B immunoglobulin
(HBIg).
E. Preclude subsequent infection with HDV.
Answer: TRUE: BC, FALSE ADE

DISCUSSION: Although some types of hepatitis are more often asymptomatic than symptomatic, that is
not the case for hepatitis B. Further, even if the HBV infection is asymptomatic, serious long-term side
effects may occur. A prior infection with hepatitis B confers lifelong immunity even if the antibody
titer wanes below the protective level of 10 mIU. HBIg is useful in reducing the incidence of
postexposure HBV infection from around 30% with no intervention, to 15% with standard immune
globulin, to about 5% to 7% with HBIg. HBV infection is required for infection with HDV and is therefore
an essential step toward, rather than preventive of, HBV infection.

26. Which of the following statements about choledocholithiasis are correct?


A. Common duct stones can originate in the gallbladder and migrate to the common duct, and stones
can form de novo in the duct system.
B. Calcium bilirubinate stones are associated with the presence of bacteria in the duct system.
C. Common duct stones discovered at laparoscopic cholecystectomy should be treated by postoperative
endoscopic extraction.
D. The serum bilirubin value is usually greater than 15 mg. per dl. in the patient with a symptomatic
common duct stone.
Answer: ABC
DISCUSSION: Most common duct stones originate in the gallbladder and migrate to the common duct,
where they may become larger. These stones tend to consist predominantly of cholesterol (about 80%
of gallbladder stones are predominantly cholesterol). Stones found in the bile ducts after
cholecystectomy may have been overlooked, but de novo stone formation does occur. Arbitrarily,
stones found 2 years after cholecystectomy are assumed to have formed within the duct system.
Calcium bilirubinate stones are thought to result from precipitation of insoluble bilirubin
monoglucuronide formed by deconjugation of bilirubin diglucuronide, a reaction promoted by the
enzyme beta-glucuronidase, which is produced by bacteria in the biliary tract. Calcium bilirubinate
stones are found almost exclusively in patients who have some form of biliary tract lesion that causes
partial obstruction, and these patients tend to have bactibilia. Stones smaller than approximately 5
mm. often can be extracted through a dilated cystic duct or pushed into the duodenum. Larger stones
are best left for postoperative endoscopic sphincterotomy and extraction. Patients with more than five
stones or stones larger than 1.5 cm. should be treated by open choledocholithotomy or, when
indicated, a biliary-enteric anastomosis. Not all patients with symptomatic common duct stones have
elevated serum bilirubin, but when jaundice is present the bilirubin is only rarely greater than 15 mg.
per dl.

27. A benign biliary duct stricture:


A. Need not be treated unless it causes clinical jaundice.
B. Should always be treated by percutaneous balloon drainage.
C. Is prone to recur after treatment with biliary-enteric anastomosis.
D. When due to chronic pancreatitis should be treated by side-to-side choledochoduodenostomy.
Answer: CD

DISCUSSION: Even a minor obstructing lesion in the extrahepatic duct system can produce cirrhosis over
time, and the development of portal hypertension, ascites, and esophageal varices. Therefore, all
biliary strictures should be treated unless this is not possible or there is no chance for success. The
presence or absence of jaundice is of no significance. Often, the only biochemical abnormality is mild
elevation of alkaline phosphatase. The long-term results of percutaneous balloon dilatation are not yet
known, but short-term results are good. Although some argue that balloon dilatation should be the
initial treatment, its role is ill-defined, and it should not be viewed as standard therapy at this time.
Biliary-enteric anastomoses are predisposed to stricture, for reasons that are ill-understood. A mucosa-
to-mucosa anastomosis, large size of the anastomosis, a normal duct at the point of anastomosis, and
stenting appear to be elements that work against stricture. About 70% of anastomoses are not
complicated by strictures. Common duct strictures caused by chronic pancreatitis are located in the
distal portion of the duct and are easily treated by side-to-side choledochoduodenostomy. A wide
anastomosis is usually possible, and because of this stenting often is not necessary. Although a Roux-
en-Y biliary-enteric reconstruction is acceptable treatment, no advantage over
choledochoduodenostomy has been demonstrated.

28. Which statements about extrahepatic bile duct cancer are correct?
A. Cholangiography is essential in evaluating patients for resectability.
B. The prognosis is excellent when appropriate surgical and adjuvant therapy are given.
C. The location of the tumor determines the type of surgical procedure.
D. The disease usually becomes manifest by moderate to severe right-side upper quadrant pain.
Answer: AC

DISCUSSION: Cholangiography is essential for both diagnosis and evaluation of resectability. Brushings
of the lesion for diagnosis and temporary stenting, done percutaneously or endoscopically, are often
done at the time of cholangiography. Angiography and CT are helpful, but in the absence of hepatic
artery or portal vein occlusion these tests are not accurate predictors of resectability. The primary
obstacles to complete resection are invasion of the portal vein or the hepatic artery and proximal
extension of the tumor into the liver. The long-range prognosis for patients who undergo treatment for
extrahepatic bile duct cancer is poor, even when the lesion is surgically resectable and adjuvant
therapy is given. Only about 10% of patients are alive without disease at 10 years. Nevertheless, bile
duct cancer tends not to metastasize to distant sites, so resection and radiation therapy are useful in
prolonging symptom-free life. Tumors in the proximal third of the extrahepatic bile duct system are
treated by a Roux-en-Y biliary-enteric anastomosis. To ensure excision of the entire tumor this
anastomosis usually must be made to the individual hepatic ducts, which must be stented individually.
Tumors of the middle third usually require anastomosis to the proximal hepatic duct. In contrast,
lesions of the distal third require Whipple's procedure with appropriate reconstruction. Thus, the
treatment of extrahepatic bile duct cancer depends on the location of the tumor. Pain is not a
prominent feature of bile duct cancer. Most cases become manifest by the insidious development of
jaundice.

29. Which of the following statements about biliary tract problems are correct?
A. Choledochal cyst should be treated by Roux-en-Y cystojejunostomy.
B. Sclerosing cholangitis is characterized by long, narrow strictures in the extrahepatic biliary duct
system.
C. Operative (needle) cholangiography is indicated in patients who at operation appear to have no
gallbladder.
D. The long cystic duct, which appears to be fused with the common duct and enters it distally, should
be dissected free and ligated at its entrance into the common duct.
Answer: C

DISCUSSION: In the past, choledochal cyst was treated by Roux-en-Y cystojejunostomy, but long-term
results were poor. Excision of the cyst is essential to prevent recurrent pancreatitis. In addition, the
development of carcinoma in about 25% of patients mandates cyst excision. Accordingly, excision of
the cyst with biliary reconstruction by Roux-en-Y hepaticojejunostomy and diversion of the flow of
pancreatic juice through the ampulla of Vater is currently the standard treatment. Sclerosing
cholangitis causes fibrosis of bile ducts both within and outside the liver. This process, which is poorly
understood, causes strictures in the duct system, characteristically with normal or dilated segments
between strictures. Unfortunately, this anatomic arrangement does not lend itself to biliary
reconstructive procedures. Each case must be analyzed, however, because in some patients the
anatomic situation may lend itself to balloon dilatation or reconstruction. When the gallbladder
appears to be absent, a search should be made for an ectopically located organ in the retroduodenal
area, within the falciform ligament, and within the substance of the right lobe of the liver. With true
gallbladder agenesis the common duct may be dilated, and choledocholithiasis is present in about one
fourth of those who undergo operation. Therefore, operative needle cholangiography should always be
done. Dissection of a long, fused cystic duct is fraught with hazard because the cystic and common
ducts may share a common wall and serious duct damage may occur. The cystic duct should be ligated
and divided immediately proximal to the area of fusion.

30. Which of the following statements about the diagnosis of acute calculous cholecystitis are true?
A. Pain is so frequent that its absence almost precludes the diagnosis.
B. Jaundice is present in a majority of patients.
C. Ultrasonography is the definitive diagnostic test.
D. Cholescintigraphy is the definitive diagnostic test.
Answer: AD

DISCUSSION: The presence of pain is the sine qua non of acute calculous cholecystitis. Chronic
cholecystitis associated with cholelithiasis may develop in the absence of pain, and in critically ill
patients pain may not be a prominent feature of acute acalculous cholecystitis. Only about 10% of
patients with acute cholecystitis are jaundiced. Although an occasional patient may have concomitant
bile duct obstruction, the jaundice associated with acute cholecystitis is probably due to absorption of
bile pigments from the diseased gallbladder. The presence of jaundice in a patient with right-side
upper quadrant pain should also suggest the possibility of acute cholangitis secondary to bile duct
obstruction. Ultrasonography is very accurate in the detection of gallstones, but stones may be present
in the absence of acute cholecystitis. Thickening of the gallbladder wall and a collection of fluid
around the gallbladder are ultrasonographic findings in some patients with acute cholecystitis, but they
are not always present and are not specific. Ultrasonography may be useful when the diagnosis is
obscure because other conditions in the liver, pancreas, and kidney can be detected; however, it is not
the definitive test for acute cholecystitis. Cholescintigraphy is specific for the diagnosis of acute
calculous cholecystitis (accuracy over 95% in experienced hands). The rapidity, simplicity, and accuracy
make cholescintigraphy the definitive diagnostic test in acute calculous cholecystitis; however, it must
be interpreted cautiously in the context of another critical illness or recent surgery or trauma, because
false-positives are not unusual in these situations.

31. Which statements about acute acalculous cholecystitis are correct?


A. The disease is often accompanied by or associated with other conditions.
B. The diagnosis is often difficult.
C. The mortality rate is higher than that for acute calculous cholecystitis.
D. The disease has been treated successfully by percutaneous cholecystostomy.
Answer: ABCD

DISCUSSION: About half of the cases of acute acalculous cholecystitis are associated with other
conditions, including sepsis, sarcoidosis, polyarteritis nodosa, and systemic lupus erythematosus. A
majority of cases occur after trauma, burns, or major surgical procedures performed for other
conditions. The precise pathogenesis has not been determined. The diagnosis of acute acalculous
cholecystitis is often difficult because symptoms may be masked by another illness, injury, or the
postoperative state. Unlike acute calculous cholecystitis, in which pain is always present, pain occurs
in only about 70% of cases. In addition, cholescintigraphy is sometimes inaccurate. These factors make
the diagnosis difficult, and a high index of suspicion is necessary, especially in patients who have had
operations or trauma. Unexplained abdominal pain, sepsis, and ileus should prompt a thorough
investigation. The mortality rate for acute acalculous cholecystitis is higher than that of the calculous
type. The incidence of gangrene and perforation of the gallbladder is higher. The accompanying
illnesses and conditions and the frequent delays in diagnosis undoubtedly contribute to the higher
death rate. Percutaneous cholecystostomy has been used as a diagnostic and therapeutic maneuver in
patients who are thought to have acute acalculous cholecystitis. Aspiration and culture of bile assist in
confirming the diagnosis, and continuous drainage successfully treats the acute condition. Surprisingly,
persistent gangrene and subsequent complications have been infrequent. Immediate cholecystectomy
should be done if significant improvement does not take place within 12 hours of percutaneous
cholecystostomy. Long-term management of the tube and the need for elective cholecystectomy must
be individualized. The experience with percutaneous cholecystostomy is too small to determine
whether this technique reduces the mortality rate.

32. True statements about the surgical management of patients with acute calculous cholecystitis
include:
A. Operation should be performed in all patients as soon as the diagnosis is made.
B. Antibiotic therapy should be initiated as soon as the diagnosis is made.
C. Dissection of the gallbladder is facilitated by decompression of the organ with the use of a trocar.
D. An operative cholangiogram should be done in every patient.
Answer: BC

DISCUSSION: Cholecystectomy should be done in an otherwise healthy patient as soon as the diagnosis
is made and the patient is properly prepared for surgery. However, patients who have one or more
significant risk factors such as a recent myocardial infarction, unstable angina, clinically significant
coronary artery disease, or cirrhosis should not have immediate cholecystectomy unless they do not
improve within 24 to 36 hours in response to antibiotic administration and supportive care. Antibiotic
administration should commence as soon as the diagnosis is made and should be continued for 24 hours
postoperatively—or for 7 days if significant peritonitis is present. This use of antibiotics has
significantly reduced septic complications after cholecystectomy for acute cholecystitis. In most cases
the gallbladder is tensely distended, making visualization and dissection of the cystic duct area
difficult and perhaps dangerous. Decompression of the gallbladder by insertion of a needle facilitates
retraction and dissection of the gallbladder. Although some advise that operative cholangiography be
done only on a selective basis, its routine use helps to delineate anatomy and facilitates detection of
an occasionally unsuspected bile duct stone. Accordingly, it is used routinely in elective cases. In acute
cholecystitis, however, the biliary duct system may be very friable, and operative cholangiography
should be done only when it is safe to do so.

33. Which of the following are indications for cholecystectomy?


A. The presence of gallstones in a patient with intermittent episodes of right-side upper quadrant pain.
B. The presence of gallstones in an asymptomatic patient.
C. The presence of symptomatic gallstones in a patient with angina pectoris.
D. The presence of asymptomatic gallstones in a patient who has insulin-dependent diabetes.
Answer: A

DISCUSSION: Cholecystectomy (and concomitant operative cholangiography) are indicated for


symptomatic patients to relieve pain and to prevent the development of acute cholecystitis and its
complications. Morbidity and expense are not as great for elective cholecystectomy as they are for
cholecystectomy for acute cholelithiasis. The risk of the development of symptoms in patients who
have asymptomatic stones is approximately 2% per year, a rate associated with mortality and morbidity
that do not exceed those of elective cholecystectomy. Therefore, cholecystectomy is not indicated for
asymptomatic patients. Patients who have angina pectoris should not have cholecystectomy until their
coronary artery disease has been treated adequately, even if this requires a coronary artery bypass
procedure. Heart disease is the most frequent cause of death after cholecystectomy. Prophylactic
cholecystectomy, formerly recommended for insulin-dependent diabetics, is not indicated because
several studies have shown that the mortality rate from acute cholecystitis is no higher for diabetics
than for nondiabetics.

34. Which of the following statements about laparoscopic cholecystectomy are correct?
A. The procedure is associated with less postoperative pain and earlier return to normal activity.
B. The incidence of bile duct injury is higher than for open cholecystectomy.
C. Laparoscopic cholecystectomy should be used in asymptomatic patients because it is safer than open
cholecystectomy.
D. Pregnancy is a contraindication.
Answer: AB

DISCUSSION: Studies have clearly documented that postoperative pain following laparoscopic
cholecystectomy is less than that experienced after open cholecystectomy and that patients can
resume normal activity sooner. This appears to be related to the reduced trauma to the abdominal wall
by virtue of the very small incisions used in laparoscopic procedures. The best evidence is that the bile
duct injury rate (0.4%) is approximately double that for open cholecystectomy. The incidence of this
serious complication will probably decrease with improved techniques, better training, and more
advanced instrumentation. Only symptomatic patients should have cholecystectomy. Prophylactic
removal of the gallbladder is not cost effective. All elective operations are contraindicated in the first
trimester, so as to prevent fetal anomalies and spontaneous abortion. The laparoscopic technique is
not contraindicated thereafter except in patients in whom peritoneal access cannot safely be
established. This is rarely a problem. Premature labor is a risk in the third trimester. Thus, unless
cholecystectomy can be avoided altogether during pregnancy, the second trimester is the most
propitious time.

35. Which of the following statements about cholangitis are correct?


A. Charcot's triad is always present.
B. Associated biliary tract disease is always present.
C. Chills and fever are due to the presence of bacteria in the bile duct system.
D. The most common cause of cholangitis is choledocholithiasis.
Answer: BCD

DISCUSSION: Although Charcot's triad (pain, chills and fever, jaundice) is diagnostic of cholangitis, the
complete triad occurs only in 50% to 70% of patients. Fever is the most common symptom; therefore,
cholangitis should be considered in all patients who have unexplained fever. Episodes of pain, chills,
and fever are often so brief as not to concern the patient. Cholangitis does not occur in the absence of
partial or complete bile duct obstruction. All patients diagnosed as having cholangitis should have
appropriate diagnostic studies to determine the cause. This usually involves cholangiography. The
presence of bacteria in bile does not produce symptoms in the absence of partial or complete
obstruction of the bile duct system. When obstruction is present, pressure within the system increases,
giving rise to reflux of bacteria or their toxic products into the hepatic venous circulation. This
cholangiovenous reflux produces chills, fever, and the hemodynamic changes of sepsis. Death may
ensue if treatment is not instituted promptly. Choledocholithiasis, the most commonly associated
problem, may produce partial or complete obstruction. When bacteria are not present in the bile duct
system, choledocholithiasis may go undetected unless the degree of obstruction is sufficient to cause
jaundice. Other causes of cholangitis are benign and malignant strictures, biliary-enteric anastomoses,
invasive procedures, foreign bodies, and parasitic infestation of the bile ducts.

36. Recurrent episodes of cholangitis:


A. Suggest the presence of undetected or overlooked bile duct pathology.
B. Occur frequently in patients who have indwelling biliary tubes or stents.
C. May be ameliorated by long-term administration of antibiotics.
D. May be associated with the development of secondary biliary cirrhosis.
Answer: ABCD
DISCUSSION: Cholangitis does not occur in the presence of a normal bile duct system, and all patients
with cholangitis have an abnormality. Thus, recurrent episodes of cholangitis signal the need for
diagnostic studies. Cholangiography usually will be necessary. The presence of any foreign body in the
biliary tract is frequently associated with bactibilia and recurrent episodes of cholangitis. Even a silk
suture exposed to the lumen of a bile duct has been known to cause cholangitis. Pigment stone and
sludge formation may result from the bacterial deconjugation of bilirubin diglucuronide to bilirubin
monoglucuronide, which precipitates as calcium bilirubinate. This material can occlude indwelling
tubes and predispose to more frequent episodes of cholangitis. Long-term administration of an oral
antibiotic may reduce the frequency and severity of attacks of cholangitis; however, this method of
management should not be routine. Correction of the underlying problem is essential. Chronic
obstruction and recurrent infection eventually lead to secondary biliary cirrhosis and its complications
of portal hypertension, ascites, and bleeding esophageal varices. Once this stage of the disease is
reached, correction of the underlying biliary tract problem does not reverse the changes in the liver.
Once again, every effort should be made to eliminate the cause of the cholangitis early in the course of
disease. The only effective treatment for end-stage liver disease is hepatic transplantation.

37. The initial goal of therapy for acute toxic cholangitis is to:
A. Prevent cholangiovenous reflux by decompressing the duct system.
B. Remove the obstructing stone, if one is present.
C. Alleviate jaundice and prevent permanent liver damage.
D. Prevent the development of gallstone pancreatitis.
Answer: A

DISCUSSION: Uncontrolled sepsis and the consequent multisystem organ failure are the life-threatening
sequelae of acute toxic cholangitis. Thus, the initial goal of treatment is to decompress the biliary duct
system to prevent reflux of bacteria and their toxic products into the circulation. This can be done by
intubating the duct system through the percutaneous, transhepatic, or the endoscopic route or by
insertion of a T tube in the common duct at operation. Removal of the stone causing the obstruction is
not necessary to stabilize the patient. Only after the duct is decompressed should the cause of the
obstruction be addressed. When transhepatic biliary drainage has been used, endoscopic or surgical
removal of the stone can be carried out after the patient has recovered completely. When initial
therapy is sphincterotomy, the stone should be removed as part of the procedure. Often the stone falls
out without manipulation. If surgical placement of a T tube is the initial treatment, the stone should
be removed only if it is convenient to do so. The long-range goal of treatment of patients with bile
duct obstruction is to prevent cirrhosis, ascites, portal hypertension, and hemorrhage from esophageal
varices; however, death from sepsis is the immediate threat in acute toxic cholangitis. Gallstone
pancreatitis may occur in patients who have an impacted stone in the distal duct, independent of the
presence or absence of acute toxic cholangitis; however, gallstone pancreatitis is more often
associated with the passage of a stone into the duodenum.

38. The clinical picture of gallstone ileus includes which of the following?
A. Air in the biliary tree.
B. Small bowel obstruction.
C. A stone at the site of obstruction.
D. Acholic stools.
E. Associated bouts of cholangitis.
Answer: ABCE

DISCUSSION: An antecedent biliary-enteric fistula is necessary to allow stone migration into the
intestinal tract, and this results in air entering the biliary tree (pneumobilia). It also allows
contamination of the bile ducts with intestinal bacteria, which in fact occurs in only a minority of such
cases. The stone obstructs the narrower distal bowel, producing small bowel obstruction. Such a stone,
if opaque, can be seen on plain radiography and, if not, can be appreciated by sonography. Stools are
not acholic, since the cholecystoenteric fistula allows bile access to the intestinal lumen.

39. Which of the following statement(s) about gallstone ileus is/are not true?
A. The condition is seen most frequently in women older than 70.
B. Concomitant with the bowel obstruction, air is seen in the biliary tree.
C. The usual fistula underlying the problem is between the gallbladder and the ileum.
D. When possible, relief of small bowel obstruction should be accompanied by definitive repair of the
fistula since there is a significant incidence of recurrence if the fistula is left in place.
E. Ultrasound studies may be of help in identifying a gallstone as the obstructing agent.
Answer: C

DISCUSSION: It is true that gallstone ileus occurs mostly in elderly women and should always be suspect
when small bowel obstruction presents in this age group. The great majority of cases of gallstone ileus
are preceded by a spontaneous fistula occurring between the gallbladder and duodenum, allowing
gallstones to enter the intestinal tract, which can potentially block the terminal ileum. Finding air
within the biliary tree should always arouse suspicion of the possibility of this diagnosis when it is
associated with a radiographic pattern of small bowel obstruction. The initial part of the operative
approach to this disease is to relieve the bowel obstruction by performing an enterotomy just proximal
to the point of obstruction to remove the stone. Where possible, definitive repair of the fistula should
be undertaken to avoid recurrent obstruction and to obviate the possible recurring complications of
cholangitis. Percutaneous drainage of bile collections combined with endoscopic papillotomy may be
sufficient treatment for external and internal biliary fistulas but is never an allowable approach in the
presence of gallstone ileus with small bowel obstruction. Relief of the obstruction is mandated in this
setting.

40. Which of the following lesions are believed to be associated with the development of carcinoma of
the gallbladder?
A. Cholecystoenteric fistula.
B. A calcified gallbladder.
C. Adenoma of the gallbladder.
D. Xanthogranulomatous cholecystitis.
E. All of the above.
Answer: E

DISCUSSION: The prevalence of carcinoma of the gallbladder in patients who have or have had a
cholecystoenteric fistula is believed to be 15%. The prevalence of carcinoma in a calcified, or
“porcelain,” gallbladder is reported to range from 12.5% to 61%. It is generally accepted that adenoma
of the gallbladder is a precancerous lesion that presents as a polypoid lesion. Xanthogranulomatous
cholecystitis is a rare form of chronic cholecystitis believed to be associated with a higher incidence of
cancer. This form of cholecystitis is also important because, grossly, it may mimic cancer of the
gallbladder.

41. The preferred treatment for carcinoma of the gallbladder is:


A. Radical resection that includes gallbladder in continuity with the right hepatic lobe and regional
lymph node dissection.
B. Radiation therapy.
C. Chemotherapy.
D. Combined treatment involving surgical therapy, chemotherapy, and radiation.
E. None of the above.
Answer: E
DISCUSSION: Radical resection, radiation therapy, and chemotherapy have been effective only
anecdotally. Most believe that the dismal prognosis of carcinoma of the gallbladder does not justify
anything more than palliative treatment. About 88% of patients are dead within a year of diagnosis,
and only about 4% are alive after 5 years, regardless of the type of treatment. Those whose surgeon
was unaware of the presence of the tumor at the time of cholecystectomy (approximately 12% of
cases) are most likely to survive long term. There are insufficient data to support conclusively the
proposition that the patient with unexpected carcinoma found on histologic examination should
undergo reoperation with intent for radical excision. There also are indirect suggestions that the
prognosis of gallbladder carcinoma may be improving, but it is not clear if this is spontaneous or due to
either earlier diagnosis or surgical management.

42. Which of the following statement(s) about pancreatic embryonic malformations is/are correct?
A. Pancreas divisum can be a cause of gastrointestinal bleeding.
B. Heterotopic pancreatic tissue predisposes to pancreatic adenocarcinoma.
C. Annular pancreas may cause gastrointestinal obstruction in children or in adults.
D. Relative obstruction to the flow of pancreatic juice through the minor papilla appears to be the
cause of pancreatitis in some patients with pancreas divisum.
Answer: CD

DISCUSSION: The clinically recognized embryonic malformations of the pancreas include heterotopic
pancreas, pancreas divisum, and annular pancreas. Heterotopic pancreatic tissue most often takes the
form of a firm nodule of variable size in the stomach, duodenum, small bowel, or Meckel's
diverticulum. The typical complications of heterotopic pancreas include intestinal obstruction,
ulceration, or hemorrhage. Pancreas divisum is an anatomic variant that results from failure of fusion
of the two primordial pancreatic duct systems. In pancreas divisum the major portion of the pancreas
is drained via the duct of Santorini through the minor duodenal papilla. Relative stenosis of the minor
duodenal papilla can cause pancreatitis. Pancreas divisum is not associated with gastrointestinal
bleeding. Annular pancreas results when histologically normal pancreatic tissue completely or partially
encircles the second portion of the duodenum. Varying degrees of duodenal obstructive symptoms may
be observed in both children and adults with this condition.

43. The pancreas occupies a retroperitoneal position in the upper abdomen. Which statement(s) is/are
correct?
A. The superior mesenteric vein and the splenic vein join to form the portal vein posterior to the neck
of the pancreas.
B. The uncinate process of the pancreas extends posterior to the inferior vena cava.
C. The tail of the pancreas extends to the left of the aorta, toward the splenic hilum.
D. The head of the pancreas is jointly supplied by arterial blood from the celiac axis and the superior
mesenteric artery.
Answer: ACD

DISCUSSION: The pancreas occupies a retroperitoneal position in the upper abdomen, extending
obliquely from the duodenal C loop to a more cephalad position where the pancreatic tail abuts the
hilum of the spleen. The portion of the pancreas anterior to the confluence of the superior mesenteric
vein, splenic vein, and portal vein is designated the neck of the gland. The uncinate process extends
posterior to the superior mesenteric vein and approaches the superior mesenteric artery. The head of
the pancreas is intimately associated with the second portion of the duodenum, and these two
structures are jointly supplied by two arterial arcades known as the anterior and posterior
pancreaticoduodenal arteries, which originate as branches of the celiac axis and superior mesenteric
artery.

44. Both endocrine and exocrine tissue comprise the pancreas. Which statement(s) is/are true?
A. The islets of Langerhans total 1 million per gland and drain their secretions via intercalated duct
cells through the ampulla of Vater.
B. Islet alpha cells produce glucagon.
C. Islet sigma cells produce somatostatin.
D. The acini and ductal systems constitute the exocrine portion of the pancreas.
Answer: BD

DISCUSSION: The endocrine portion of the pancreas is served by the islets of Langerhans, which number
1 million islets per gland. The islets of Langerhans drain their endocrine secretions into the
bloodstream. Insulin-producing beta cells comprise the majority of the islet population. Alpha cells
produce glucagon and constitute approximately 20% to 25% of the total islet cell number. Delta cells of
the islets produce somatostatin. The acini and ductal systems constitute the exocrine portion of the
pancreas. The acinar cells contain zymogen granules in their narrow, centrally located apical portion.
The pancreatic duct system includes intercalated duct cells along the ductal pathway, terminating in
the main excretory duct of the pancreas.
45. Pancreatic exocrine secretory products include a bicarbonate-rich electrolyte solution as well as
digestive enzymes. Which of the following statement(s) is/are true?
A. Cholecystokinin (CCK) is the most potent endogenous stimulant of pancreatic enzyme secretion.
B. The chloride and bicarbonate concentrations of pancreatic juice vary and depend on the secretory
flow rate.
C. Secretin is the most potent endogenous stimulant of pancreatic water and electrolyte secretion.
D. The peptidases synthesized by acinar cells are released into the pancreatic duct system in active
form.
Answer: ABC

DISCUSSION: CCK is the most potent endogenous stimulant of pancreatic enzyme secretion. The
pancreatic acinar cells respond to CCK with release of their zymogen granules into the ductal system.
Peptidases are released in inactive form, later to be activated by contact with duodenal enterokinase
and activated trypsin. Secretin is the most potent endogenous stimulant of pancreatic water and
electrolyte secretion. The concentrations of the anions bicarbonate and chloride vary and are largely
dependent on the secretory flow rate stimulated by secretin.

46. Which of the following parameters is/are not included in the Ranson's prognostic signs useful in the
early evaluation of a patient with acute pancreatitis?
A. Elevated blood glucose.
B. Leukocytosis.
C. Amylase value greater than 1000 U per dl.
D. Serum lactic dehydrogenase (LDH) greater than 350 IU per dl.
E. Alanine aminotransferase greater than 250 U per dl.
Answer: CE

DISCUSSION: Several prognostic systems have been demonstrated to predict the severity of pancreatitis
accurately. Two Ranson prognostic criteria have been developed: one each, for pancreatitis that is not
due to gallstones and pancreatitis that is. The systems have minor differences. In both of the Ranson
systems elevated blood glucose, leukocytosis, and elevations of serum LDH have proved to have
prognostic importance. The degree of amylase elevation is not one of the parameters, nor is the degree
of ALT elevation.
47. Standard supportive measures for patients with mild pancreatitis include the following:
A. Intravenous fluid and electrolyte therapy.
B. Withholding of analgesics to allow serial abdominal examinations.
C. Subcutaneous octreotide therapy.
D. Nasogastric decompression.
E. Prophylactic antibiotics.
Answer: A

DISCUSSION: Standard therapy for all patients with mild acute pancreatitis should include intravenous
fluid resuscitation, electrolyte replacement, and analgesics. Nasogastric decompression is typically
reserved for patients with significant ileus who are at risk for emesis and aspiration. Subcutaneous
therapy with octreotide, the octapeptide analog of somatostatin, has not been proven to influence the
outcome in patients with mild pancreatitis. Prophylactic antibiotics are not used for mild pancreatitis.
Antibiotics are reserved for patients with severe pancreatitis (defined as greater than three Ranson
prognostic signs with associated CT evidence of pancreatic or peripancreatic necrosis).

48. Which of the following statements about chronic pancreatitis is/are correct?
A. Chronic pancreatitis is the inevitable result after repeated episodes of acute pancreatitis.
B. Patients with chronic pancreatitis commonly present with jaundice, pruritus, and fever.
C. Mesenteric angiography is useful in the evaluation of many patients with chronic pancreatitis.
D. Total pancreatectomy usually offers the best outcome in patients with chronic pancreatitis.
E. For patients with disabling chronic pancreatitis and a dilated pancreatic duct with associated
stricture formation, a longitudinal pancreaticojejunostomy (Peustow procedure) is an appropriate
surgical option.
Answer: E

DISCUSSION: Chronic pancreatitis is a clinical entity that includes recurrent or persistent abdominal
pain with evidence of exocrine and endocrine pancreatic insufficiency. While chronic pancreatitis may
result from repeated episodes of acute pancreatitis, not all patients with recurring acute pancreatitis
progress to chronic pancreatitis. The most common causes of chronic pancreatitis include alcohol
abuse, hyperparathyroidism, congenital anomalies of the pancreatic duct, pancreatic trauma, and
cystic fibrosis. The most useful radiographic tests in patients with suspected chronic pancreatitis are
CT and endoscopic retrograde cholangiopancreatography (ERCP). Mesenteric angiography has no role in
the evaluation of most patients with chronic pancreatitis. Patients with disabling chronic pancreatitis
who require operative intervention are candidates for a longitudinal pancreaticojejunostomy (Peustow
procedure) if pancreatography demonstrates a dilated pancreatic duct. Total pancreatectomy is rarely
performed because of the significant problems associated with labile insulin sensitivity, steatorrhea,
and weight loss.

49. Which of the following statements about pancreatic ascites is/are correct?
A. Patients typically present with painful ascites, reflecting the release of toxic pancreatic enzymes
into the peritoneal cavity.
B. The standard evaluation of a patient with new-onset ascites includes abdominal paracentesis. In
cases of pancreatic ascites, the peritoneal fluid contains high concentrations of both amylase and
protein.
C. Pancreatic ascites can follow an episode of acute pancreatitis.
D. Patients with pancreatic ascites may fail to improve with nonoperative therapy and require surgical
procedures. At abdominal exploration an acceptable approach to the pancreatic duct disruption
involves suture ligation with omental patching.
Answer: BC

DISCUSSION: Pancreatic ascites typically occurs because of a pancreatic duct disruption, most
commonly involving alcohol abuse and resultant acute pancreatitis. In pancreatic ascites, pancreatic
exocrine secretions exit a pancreatic duct disruption and drain anteriorly into the peritoneal cavity.
Patients typically present with painless massive ascites, as the pancreatic enzymes that extravasate
into the peritoneal cavity are typically nonactivated. The diagnosis of pancreatic ascites is best made
by paracentesis, in which the analysis of the ascites fluid reveals it to be high in amylase (more than
1000 U. per dl.) and high in albumin (more than 3 gm. per dl.). Nonoperative treatment is initially
indicated in most patients with pancreatic ascites. Should nonoperative therapy fail, surgical therapy is
directed to closure of the pancreatic duct disruption. Preoperative pancreatography is useful in
directing surgical therapy. Distal pancreatic duct disruption may be treated with distal pancreatectomy
or with Roux-en-Y pancreaticojejunostomy. Pancreatic leaks in the more proximal aspects of the gland
are treated with Roux-en-Y pancreaticojejunostomy. Suture ligation of the pancreatic duct with
omental patching is not considered appropriate therapy for pancreatic duct disruptions.

50. Which of the following statements about adenocarcinoma of the pancreas is/are correct?
A. It is the fifth most common cause of cancer death in the U.S.
B. Most cases occur in the body and tail of the pancreas, making distal pancreatectomy the most
commonly performed resectional therapy.
C. For cancers of the head of the pancreas resected by pancreaticoduodenectomy, prognosis appears to
be independent of nodal status, margin status, or tumor diameter.
D. The most accurate screening test involves surveillance of stool for carbohydrate antigen (CA 19–9).
Answer: A

DISCUSSION: Adenocarcinoma of the pancreas is newly diagnosed in approximately 28,000 patients in


the United States every year. It is the fifth most common cause of cancer death in the United States,
exceeded only by lung, colorectal, breast, and prostate cancer. The majority of cases of
adenocarcinoma of the pancreas occur in the head of the gland, and if resectable, are treated via
pancreaticoduodenectomy. Recent studies have shown that factors favoring long-term survival after
pancreaticoduodenectomy for adenocarcinoma of the head of the pancreas include negative nodal
status, negative margin status, small tumor diameter, and diploid DNA content. No accurate screening
tests for adenocarcinoma of the pancreas are currently available. The best serologic test appears to be
the CA 19–9, which is elevated in the majority of patients with adenocarcinoma of the head of the
pancreas. Unfortunately, the test is not sufficiently sensitive or specific, and further screening tests
are needed.

51. A 35-year-old woman presents with episodes of obtundation, somnolence, and tachycardia. An
insulinoma is suspected based on a random serum glucose test value of 38 mg. per dl. Which of the
following statements is/are true?
A. The most important diagnostic study for insulinoma is an oral glucose tolerance test.
B. It may be helpful to perform ERCP in an effort to localize the tumor.
C. Most patients with insulinoma present with extensive disease, rendering them only rarely resectable
or curable.
D. An important component of the preoperative evaluation in patients with presumed insulinoma
involves confirming elevated C-peptide or proinsulin levels and screening for anti-insulin antibodies.
Answer: D

DISCUSSION: Insulinoma is the most common endocrine tumor of the pancreas. Insulinoma is associated
with Whipple's triad, which consists of (1) symptoms of hypoglycemia at fasting; (2) documentation of
blood glucose levels of less than 50 mg. per dl.; and (3) relief of symptoms following administration of
glucose. The most reliable method for diagnosing insulinomas is a monitored fast. Neither an oral or an
intravenous glucose tolerance test is indicated in the majority of patients being evaluated for
insulinoma. Support for the diagnosis of insulinoma can come from documenting elevated C peptide
and proinsulin levels. Screening for anti-insulin antibodies is indicated to rule out the possibility of
surreptitious insulin administration. Tumor localization is typically performed with CT, endoscopic
ultrasonography, or angiography. ERCP is not indicated for evaluation of most pancreatic endocrine
tumors, as the tumors only rarely communicate with the main pancreatic duct system. As many as 90%
of patients with insulinoma have benign solitary pancreatic adenomas amenable to surgical cure.

52. Which of the following statements about gastrinoma (Zollinger-Ellison syndrome) is/are correct?
A. As many as 75% of gastrinoma patients have sporadic disease; 25% have gastrinoma associated with
multiple endocrine neoplasia type 1 (MEN 1).
B. Extrapancreatic gastrinomas are common, and exploration should include careful assessment of the
duodenum and peripancreatic lymph nodes.
C. Diarrhea may be a prominent presenting feature of some patients with gastrinoma.
D. Before elective operation acid-reducing medications such as omeprazole should be administered.
Answer: ABCD

DISCUSSION: Gastrinoma patients typically present with peptic ulceration of the upper gastrointestinal
tract and abdominal pain. As many as 50% of patients may have diarrhea, which may be a prominent
feature in some cases. Approximately 25% of gastrinoma patients have the disease associated with the
MEN-1 syndrome, whereas 75% have a sporadic variety of the disease. Recent evidence indicates that
extrapancreatic gastrinomas are common. Careful attention must be paid to the duodenum and
peripancreatic lymph nodes at the time of abdominal exploration. Before elective operation it is
imperative that the gastric acid hypersecretion be controlled. The control of gastric hypersecretion is
best performed by the administration of one of the substituted benzimidazoles, such as omeprazole or
lansoprazole.

53. With regard to the control of pancreatic exocrine function, which of the following statement(s)
is/are correct?

a. Cholecystokinin, a hormone released from the duodenal mucosa, is the predominant stimulus for
pancreatic enzyme secretion
b. Gastrin is a major stimulant for pancreatic bicarbonate secretion
c. Secretin is released from the duodenum upon mucosal acidification and stimulates pancreatic
bicarbonate secretion
d. Acetylcholine, released from pancreatic nerves, stimulates enzyme secretion
Answer: a, c, d
Enzyme secretion is regulated primarily through hormonal and neural factors. The enteric hormone
cholecystokinin, released from endocrine cells in the duodenal mucosa, is the predominant regulator
and stimulates acinar cells through specific membrane-bound receptors. Acetylcholine strongly
stimulates acinar cells when released from postganglionic fibers of the pancreatic plexus and acts in
synergy with CCK to potentiate enzyme secretion. Secretin weakly stimulates acinar cell secretion and
potentiates the effect of cholecystokinin on the acinar cells.
Bicarbonate is formed from carbonic acid by the enzyme carbonic anhydrase. Secretin, the major
stimulant for bicarbonate secretion, is released from the duodenal mucosa in response to a duodenal
luminal pH of less than 3.0. Cholecystokinin only weakly stimulates bicarbonate secretion, whereas it
potentiates secretin-stimulated bicarbonate secretion. Gastrin and acetylcholine are weak stimulants
of bicarbonate secretion.

54. In the performance of a pancreaticoduodenectomy (Whipple procedure), the superior mesenteric


vein is an important landmark. Which of the following statements is/are true with regard to the
superior mesenteric vein?

a. Small venous branches enter the superior mesenteric vein anteriorly as it courses beneath the neck
of the pancreas
b. The superior mesenteric vein joins the splenic vein at the superior border of the pancreas to form
the portal vein
c. Small venous branches enter the superior mesenteric vein laterally as it courses beneath the neck of
the pancreas
d. The superior mesenteric vein courses anterior to the neck of the pancreas
Answer: b, c

The venous drainage of the pancreas and duodenum follows the arterial supply. The anterior and
posterior venous arcades drain the head; the body and tail drain into the splenic vein. All venous
effluent from the pancreas ultimately drains into the portal vein which is formed by the confluence of
the superior mesenteric vein and the splenic vein at the superior border of the pancreas. The anterior
and posterior venous arcades in the head of the pancreas drain directly into the suprapancreatic portal
vein. The anteroinferior pancreaticoduodenal arcades drain with the right gastroepiploic vein to form a
common venous trunk with the right colic vein. This trunk is known as the gastrocolic trunk and enters
the superior mesenteric vein at the inferior border of the neck of the pancreas. The posteroinferior
venous arcade empties directly into the superior mesenteric vein. The veins of the head drain laterally
into the superior mesenteric and portal veins. There are no venous tributaries entering the superior
mesenteric vein anteriorly. For this reason, it is safe to dissect the neck of the pancreas directly
anterior to the superior mesenteric and portal veins when performing a pancreaticoduodenectomy.

55. Pancreas divisum results from incomplete fusion of the ventral pancreatic duct with the dorsal
pancreatic duct during embryologic development. Which of the following statements correctly
describes pancreas divisum?

a. The body and tail of the pancreas drain via an accessory ampulla distal to the ampulla of Vater. The
uncinate process drains via the ampulla of Vater
b. The entire pancreatic ductal system drains via the ampulla of Vater
c. The entire pancreatic ductal system drains via an accessory ampulla proximal to the ampulla of
Vater
d. The body and tail of the pancreas are absent. The uncinate process drains via the ampulla of Vater
Answer: c

In 90% of individuals, the main pancreatic duct, or duct of Wirsung, runs the entire length of the
pancreas and joins the common bile duct to empty into the duodenum at the ampulla of Vater. The
pancreatic duct is 2 to 3.5 mm in diameter and contains 20 secondary branches, which drain the tail,
body, and uncinate process. The drainage of the lesser duct, or duct of Santorini, is variable. The
lesser duct commonly drains the superior portion of the head of the pancreas. It empties separately
into the second portion of the duodenum through the lesser papilla located 2 cm proximal to the
ampulla of Vater. Pancreas divisum results from an incomplete fusion of the ventral pancreatic duct
with the dorsal duct during fetal development and is present in 5% of patients. In this anomaly, the
lesser duct drains the entire pancreas via an accessory ampulla located proximal to the ampulla of
Vater. Inadequacy of this pattern of drainage can result in chronic pain.

56. Which of the following statements is/are correct with regard to the blood supply of the pancreas?

a. The inferior pancreaticoduodenal artery, a branch of the celiac artery, divides into anterior and
posterior branches to supply the pancreatic head
b. The body and tail of the pancreas are supplied by branches of the splenic artery
c. The superior pancreaticoduodenal artery is a branch of the gastroduodenal artery
d. The body and tail of the pancreas are supplied by branches derived from the left renal artery
Answer: b, c

The pancreas receives its blood supply from a variety of major arterial sources. In the head of the
pancreas, there are arcades in the anterior and posterior surfaces, which generally collateralize. These
arcades arise from branches of the gastroduodenal and the superior mesenteric arteries. Just distal to
the first portion of the duodenum, the gastroduodenal artery becomes the superior
pancreaticoduodenal artery, which divides into anterior and posterior branches. The inferior
pancreaticoduodenal artery is the first branch of the superior mesenteric artery and divides into
anterior and posterior branches.
The body and tail of the pancreas are supplied by the splenic artery. The splenic artery arises from the
celiac trunk and courses along the superior surface of the pancreas to the spleen. Approximately ten
branches of the splenic artery supply the body and tail of the pancreas.

57. Orally administered glucose provokes a greater insulin response than an equivalent amount of
intravenously administered glucose. The incremental response to ingested glucose is due to the effects
of which of the following hormones?

a. Gastric inhibitory peptide


b. Somatostatin
c. Pancreatic polypeptide
d. Secretin
Answer: a

Orally administered glucose stimulates a greater insulin response than an equivalent amount of
intravenous glucose through the release of enteric hormones that potentiate insulin secretion. This
effect is known as the enteroinsular axis. Gastric inhibitory polypeptide (GIP) appears to be an
important regulator of this effect, although other gut peptides, such as glucagon-like peptide I (GLP-1),
may contribute to this effect as well. Nutrients that regulate insulin secretion include amino acids,
such as arginine, lysine, and leucine, and free fatty acids. Hormones that stimulate insulin secretion
include glucagon, GIP, and cholecystokinin, whereas somatostatin, amylin, and pancreastatin are
inhibitory. Insulin is also stimulated by sulfonylurea compounds, which act independently of the
glucose concentration and form the basis of treatment of type II, or insulin-independent, diabetes.

58. The islets of Langerhans contain four major endocrine cell types that secrete which of the following
hormones?

a. Insulin, somatostatin, glucagon, secretin


b. Insulin, somatostatin, cholecystokinin, pancreatic polypeptide
c. Insulin, somatostatin, glucagon, pancreatic polypeptide
d. Insulin, secretin, glucagon, cholecystokinin
Answer: c

Within the pancreas are small nests of cells that are responsible for the secretion of hormones that
control glucose homeostasis. These nests are called islets of Langerhans and constitute 2% of the
pancreatic mass. The islets contain an average of 3000 cells and range in diameter from 40 to 900 mm.
The islets are composed of four major cell types—alpha (A), beta (B), delta (D), and PP or F cells,
which secrete glucagon, insulin, somatostatin, and pancreatic polypeptide, respectively. The B cells
are centrally located within the islet and constitute 70% of the islet mass, whereas the PP, A, and D
cells are located at the periphery of the islet. They constitute roughly 15%, 10%, and 5% of the islet cell
mass, respectively.

59. A 50-year-old man develops acute pancreatitis due to alcohol abuse. Hyperamylasemia resolves by
the third day after admission. By the eighth hospital day, the patient is noted to have recurrent fever
(38.5°C), progressive leukocytosis (18,500 WBC/mm3), and tachypnea. The most appropriate
management includes which as the next step?

a. Laparotomy with pancreatic debridement


b. CT guided aspiration of peripancreatic fluid collections
c. ERCP with sphincterotomy and placement of biliary stent
d. Intravenous amphotericin B
Answer: b

The common causes of pancreatic abscesses are infected pancreatic pseudocysts and necrotizing
pancreatitis. The diagnosis is suggested by persistent fever, leukocytosis, and a palpable abdominal
mass. Bacteremia and systemic toxicity are late clinical features. Percutaneous aspiration with positive
cultures is the definitive preoperative test, facilitated by CT scanning or ultrasound-guidance to
suspicious peripancreatic fluid collections. When diagnosed, the treatment of choice is wide surgical
débridement with removal of all infected and revitalized tissues. Generous drainage is mandatory. One
of the major sources of morbidity and mortality in this situation is the late development of mycotic
visceral pseudoaneurysms, particularly involving the splenic circulation. These may be complex
management problems, requiring angiographic embolization or other innovative treatment strategies.

60. The patient in the above question is treated by observation for 8 weeks. He continues to be
symptomatic with epigastric pain. A repeat abdominal CT scan reveals a persistent 6 cm pseudocyst in
the region of the body of the pancreas. The pseudocyst is unilocular and demonstrates a well-defined
rim of fibrous tissue. The gastric antrum is displaced anteriorly. Using CT guidance, 300 ml of fluid is
aspirated from the lesion which is shown to be collapsed radiographically. No further intervention is
performed. What is the risk of pseudocyst recurrence after simple aspiration?

a. 80–85%
b. 60–65%
c. 40–45%
d. 20–25%
Answer: d

Generally, a pancreatic pseudocyst can be observed for a period of weeks or months in an effort to
allow for spontaneous resolution. Percutaneous ultrasound-or CT-directed aspiration or drainage
catheter placement is an initial treatment option. Simple aspiration is performed if the initial aspirate
is sterile; if the aspirate is infected, a catheter or open drainage procedure is appropriate.
Determination of pancreatic ductal anatomy is important. Contrast injection into the pseudocyst at the
time of aspiration should be considered to assess the possibility of pancreatic ductal communication
and obstruction, or multiple cysts. The pseudocyst recurrence rate after simple aspiration is about 20%
to 25%.

61. In prospective, randomized trials which of the following agents or therapeutic measures has/have
been demonstrated to accelerate recovery from acute pancreatitis?

a. Peritoneal lavage
b. Anticholinergic blockade
c. Octreotide
d. H2 receptor blockade
e. None of the above
Answer: e
A variety of pharmacologic agents that directly or indirectly reduce acinar cell enzyme release or
ductal secretion have undergone clinical evaluation for the treatment of acute pancreatitis—generally
with unimpressive results. Among the first were anticholinergic drugs. Despite extensive experience
over many years, no objective data have emerged to support their use. Clinical trials of glucagon and
calcitonin based on the same principle have produced a similar lack of supportive data. More recently,
a somatostatin analog has been subjected to clinical trials for patients with acute pancreatitis.
Somatostatin inhibits pancreatic enzyme and bicarbonate secretion by preventing the normal release of
cholecystokinin, secretin, and other gut peptides. Despite the theoretical appeal, it has not been
possible to demonstrate that somatostatin alters the natural history or prognosis of simple acute
pancreatitis, although it does diminish pancreatic secretion.
Peritoneal lavage as a specific therapy for acute pancreatitis was proposed after experimental studies
demonstrated improved survival in animals with fulminant pancreatitis. The concept was appealing in
that activated proteases and other vasoactive substances identifiable in peritoneal aspirates from
patients with pancreatitis would be removed, rather than systemically absorbed. Unfortunately,
clinical trials using this approach have produced disappointing results, and the eventual overall
mortality rate appears unchanged.

62. Which of the following medical procedures has/have been associated with an increased risk of post-
procedure acute pancreatitis?

a. Common bile duct exploration


b. Endoscopic retrograde cholangiopancreatography
c. Coronary bypass grafting
d. Distal gastrectomy
Answer: a, b, c, d

Many surgical procedures in the upper abdomen are associated with postoperative pancreatitis. The
incidence of acute pancreatitis after gastric resection ranges from 0.6% to 1.23%. After biliary tract
surgery, particularly after common bile duct exploration itself, acute pancreatitis occurs with an
incidence of 0.5% to 3%. Direct manipulation or retraction of the pancreas or pancreatic duct appears
to be the most common cause. About 1% of patients develop acute pancreatitis after endoscopic
retrograde cholangiopancreatography (ERCP). This is a predictable event, and the risk can be
minimized by limiting the pressure used for contrast injection of the pancreatic duct. Acute
pancreatitis also occurs in patients after coronary artery bypass surgery and a variety of other
procedures remote from the pancreas. Although pancreatitis in this circumstance is thought to result
from ischemia, hypotension is not always noted. The systemic consequences of activation of the
inflammatory system may contribute to changes in microvascular blood flow.

63. A 42-year-old male develops acute pancreatitis in the setting of acute alcohol abuse. One week
after onset of symptoms, computed tomography of the abdomen reveals a pancreatic phlegmon and
associated pseudocyst. Which of the following factors, if present, would decrease the likelihood of
spontaneous resolution of the pseudocyst?

a. Size greater than 5 cm


b. Diffuse calcification of the pancreatic gland
c. Multilocularity
d. Location in the pancreatic tail
Answer: a, b, c

Initial management of pancreatic pseudocysts is based on symptoms. If the patient is asymptomatic


and the cyst is small (< 5.0 cm) it can be safely observed as many of these will resolve over a period of
weeks. Concurrent chronic alcoholic pancreatitis (by history or as indicated by pancreatic
calcification), pseudocyst size greater than 5 cm, the presence of a multilocular or debris-filled
pseudocyst cavity, and chronicity (longer than 6 weeks) are all factors that are associated with a lower
probability of spontaneous resolution.

64. Which of the following is/are prognostic signs reported by Ranson to predict outcomes associated
with acute pancreatitis?

a. Age greater than 60 years


b. Hematocrit decrease of 105 within 48 hours of hospital admission
c. Serum amylase value greater than 4 times upper limit of normal
d. Serum glucose greater than 200 mg/dL on admission
e. Ca2+ level less than 8 mg/dL within 48 hours of hospital admission
Answer: b, d, e

Ranson prognostic signs include:


ON ADMISSION
Age above 55 years
White blood cell count above 16,000/µL
Glucose level above 200 mg/dL
Lactase dehydrogenate level above 350 IU/L
SGOT value above 250 IU/L

AFTER 48 HOURS
Hematocrit decrease of 10%
Blood urea nitrogen level increase of 5 mg/dL
Ca2+ level below 8 mg/dL
PaO2 level below 60 mmHg
Base deficit value above 4 mEq/L
Fluid sequestration greater than 6 L

65. A 36-year-old woman is admitted to a the hospital with upper abdominal pain, hyperamylasemia,
elevation of serum alkaline phosphatase and ultrasound evidence of cholelithiasis. With intravenous
hydration and analgesia, symptoms rapidly resolved. After 48 hours, serum amylase and alkaline
phosphatase values had returned to normal and physical examination revealed lessening tenderness in
the right upper quadrant of the abdomen. Appropriate management consists of which of the following
as the next step?

a. Cholecystectomy and intraoperative cholangiography before hospital discharge


b. Elective cholecystectomy at approximately 8 weeks
c. Endoscopic sphincterotomy before discharge followed by cholecystectomy at approximately 8 weeks
d. Observation
Answer: a

A patient who has simple cholelithiasis and an episode of acute pancreatitis is usually treated
nonoperatively until clinical resolution of the pancreatitis occurs. The rate of recurrent biliary
pancreatitis is as high as 34% to 56% within 6 weeks; therefore, an aggressive operative approach is
appropriate. Cholecystectomy is often performed after the resolution of acute pancreatitis but before
hospital discharge. Common bile duct instrumentation in this setting has a substantially increased risk
of recurrent acute pancreatitis.
66. For the patient in the preceding question, symptomatic recurrence at 3 weeks after aspiration is
confirmed ultrasonographically. Endoscopic retrograde pancreatography does not demonstrate
communication of a major pancreatic duct with the pseudocyst. Appropriate management includes
which of the following?

a. Pancreatectomy to include the pseudocyst


b. Cystgastrostomy
c. Repeat aspiration followed by injection of sodium morrhuate into the pseudocyst cavity
d. Pancreatic debridement followed by external drainage
Answer: b

The operative treatment for pseudocysts depends on the underlying cause of the cyst, as well as the
size, location, and maturity of the pseudocyst wall. Whenever possible, the status of the pancreatic
duct should be assessed preoperatively, preferably by ERCP. Operative drainage can be either external
or internal. External drainage is chosen in the presence of infection or an immature capsule. The
disadvantages of external drainage include the risk of pancreatic fistula formation and a pseudocyst
recurrence. External drainage has been associated with a higher mortality rate, probably because it is
used in patients at higher risk, especially those with sepsis, pancreatic abscesses, or ruptured
pseudocysts.
The type of internal drainage procedure selected depends on the location of the pseudocyst and
whether or not there is associated pancreatic ductal pathology. Cystogastrostomy is the simplest and
safest alternative if the pseudocyst is appropriately adjacent to the posterior wall of the stomach.
Cystojejunostomy using a Roux-en-Y or loop jejunostomy may also be appropriate, depending on the
location and specific anatomy of the pseudocyst. Pancreatic resection is associated with the lowest
recurrence rate (3%), but is limited to pseudocysts occurring in the tail of the pancreas.

67. With regard to acute pancreatitis: which of the following statements is/are correct?

a. The majority of patients presenting with acute pancreatitis of biliary type are female
b. The majority of patients presenting with acute pancreatitis of alcoholic type are female
c. The most common cause of acute pancreatitis in the United States is alcohol use
d. Patients with alcohol-induced pancreatitis tend to be older than those with biliary-induced disease
Answer: a, c
In autopsy series, the evidence for past acute pancreatitis averages 0.31%. Variations among
populations are highly dependent on social factors such as ethanol use and on environmental and
hereditary determinants such as the incidence of gallstones. Acute pancreatitis may occur at any age
but is most common in adults between 30 and 70 years of age. In general, patients with gallstone-
induced pancreatitis are older (age 40 to 60 years), whereas those with alcohol-associated pancreatitis
are younger (age 30 to 40 years). The sex distribution of acute pancreatitis depends on the clinical
cause of the disease, with women representing 68% of patients with gallstone-associated pancreatitis.
Conversely, when alcohol is the primary association, men account for most patients.
Clinical associations with acute pancreatitis can be divided into three broad categories-biliary stones,
ethanol, and others. Biliary tract stone disease and ethanol-induced pancreatitis account for most
cases of acute pancreatitis reported worldwide. The particular distribution of causes reflects the
source of the patient population evaluated. In a summary of 18 different reports of acute pancreatitis
in the United States with a combined total of 7147 patients, 53% of patients were believed to have
ethanol-induced disease, whereas 28% had proven biliary stones. In contrast, of 1539 patients reported
on from Great Britain, 52% had gallstones, 7% were ethanol related, and 34% had no identifiable cause.

68. Which of the following statement(s) relating to chronic pancreatitis is/are correct?

a. In the United States, the most common cause of chronic pancreatitis is alcohol abuse
b. Approximately 50% of chronic alcoholics develop chronic pancreatitis
c. Clinically significant chronic pancreatitis develops on average after five years of alcohol abuse in
men
d. The risk of alcohol-induced chronic pancreatitis can be decreased by consumption of a high-protein
diet
Answer: a

In the United States, alcohol consumption is the major cause of chronic pancreatitis: with
approximately 70% of cases attributable to this factor. Most patients with symptomatic chronic
pancreatitis have consumed large volumes of alcohol daily for a prolonged period of time. The average
daily intake of alcohol is 150 to 175 g with the mean duration of alcoholism before recognition of
pancreatitis being 18 years for men and 11 years for women. The incidence of chronic pancreatitis on
autopsy studies of chronic alcoholics is 50 times the rate of non-drinking controls. Only 10% of
alcoholics develop chronic pancreatitis—suggesting that factors other than long-term alcohol exposure
may also influence susceptibility. In both experimental and clinical studies, the risk of alcohol-induced
chronic pancreatitis is increased by a high-protein, high-fat diet.
69. The most appropriate test to confirm a clinical diagnosis of early chronic pancreatitis is which of
the following?

a. Serum amylase determination


b. Calculation of urinary amylase clearance
c. Measurement of para-aminobenzoic acid absorption
d. Endoscopic retrograde cholangiopancreatography
Answer: d

Routine tests of blood or serum are not helpful in making a diagnosis of chronic pancreatitis. Although
serum amylase levels are almost always elevated in acute pancreatitis—amylase levels may be normal,
elevated, or subnormal in chronic pancreatitis. Determination of urinary amylase secretion and
calculation of urinary amylase clearance does not improve sensitivity or specificity. Indirect tests of
pancreatic function which measure absorption of nutrients that first require pancreatic digestion are
not helpful in early cases of chronic pancreatitis. Clinically detectable malabsorption is absent until
90% of exocrine function is lost. Because of this, indirect tests of pancreatic function do not detect
early disease. In addition, false positive tests may occur in other disease states associated with
malabsorption (Crohn’s disease, sprue, postgastrectomy states, or in association with diabetes
mellitus, cirrhosis, or renal disease. ERCP has become widely recognized as the most sensitive and
reliable method for diagnosing chronic pancreatitis. Sensitivity approaches 90% with equal specificity.

70. A 52-year-old male, known to be alcoholic, is evaluated because of chronic abdominal pain. The
clinical diagnosis of chronic pancreatitis is supported by ERCP findings of pancreatic ductal ectasia with
alternating areas of stricture and dilatation. Several pancreatic ductal stones are also noted. With
chronic pain as the operative indication, the most appropriate procedure would be:

a. 80% distal pancreatectomy with splenectomy


b. Longitudinal pancreaticojejunostomy
c. Distal pancreatectomy with end pancreaticojejunostomy
d. Total pancreatectomy
Answer: b

When patients with chronic pancreatitis have pancreatic ductal dilatation (greater than 8 mm. ductal
decompression using longitudinal pancreaticojejunostomy may be employed for relief of pain. The
finding that pancreatic ductal hypertension exists in patients with painful chronic pancreatitis and that
surgical decompression reduces intrapancreatic pressure to normal provides the rationale for this
operation. The anterior surface of the pancreas is exposed through the lesser sac. The entire
pancreatic duct is opened from the pancreatic tail to a point 1 cm from the duodenum. A side-to-side
anastomosis is then performed between the opened pancreatic duct and a loop of jejunum.
Splenectomy is not necessary. In-hospital mortality rates of less than 5% have been widely reported.
Approximately 80% of patients report complete or substantial improvement of pain following
longitudinal pancreaticojejunostomy.

71. For the patient in the preceding question, the most appropriate long-term management is which of
the following?

a. Endoscopic stenting of the distal common bile duct


b. Choledochoduodenostomy
c. Pancreaticoduodenectomy (Whipple procedure)
d. Percutaneous transhepatic drainage of the common hepatic duct
Answer: b

Operative management of patients with stricture of the common bile duct associated with chronic
pancreatitis is justified to treat symptoms and to prevent development of biliary cirrhosis. Operative
indications include progressive jaundice, cholangitis, liver biopsy evidence of biliary cirrhosis,
persistent elevation of alkaline phosphatase at greater than three times normal, and progressive
stricture demonstrated by radiologically progressive dilatation of extrahepatic and intrahepatic biliary
ducts. Both choledochoduodenostomy and choledochojejunostomy are excellent operative choices.

72. Which of the following is the most common cause of obstructive jaundice in patients with chronic
pancreatitis?

a. Adenocarcinoma of the head of the pancreas


b. Choledocholithiasis
c. Fibrotic stricture of the common bile duct
d. Pancreatic pseudocyst formation
Answer: c
Biliary complications involving the common bile duct can occur in chronic pancreatitis because of the
intimate association of that structure with the head of the pancreas. In two-thirds of individuals, the
common bile duct traverses the pancreatic parenchyma and in an additional 25%, the common bile duct
lies within a groove along the posterior surface of the pancreas. Fibrosis associated with chronic
pancreatitis can encase and compress the common bile duct. Common bile duct stenosis is relatively
common in chronic pancreatitis, occurring in approximately 10% of cases observed long-term.
Cholangiography typically reveals a long, gradually tapering stricture conforming to the intrapancreatic
portion of the common bile duct. In contrast, malignant strictures usually result in abrupt termination
of the biliary duct. The proximal suprapancreatic portion of the bile duct is variably dilated.

73. Alcohol-induced and hereditary chronic pancreatitis are the two most common etiologies observed
in North American patients. Most of the remaining patients fall into which of the following categories?

a. Chronic pancreatitis secondary to hyperparathyroidism


b. Chronic pancreatitis caused by protein-calorie malnutrition
c. Chronic pancreatitis secondary to congenital pancreatic ductal obstruction
d. Idiopathic chronic pancreatitis
Answer: d

After alcohol-induced and hereditary disease, idiopathic chronic pancreatitis is the most common cause
of calcifying pancreatitis in North American patients. This designation is given to those cases without a
recognizable cause. Idiopathic pancreatitis accounts for about 15% of the cases and has two peaks in
incidence, suggesting that differing underlying causes may exist. The first peak occurs in young
adulthood and the second has an occurrence at approximately 60 years of age.

74. Which of the following statements regarding prognosis in chronic pancreatitis is/are correct?

a. Patients with chronic pancreatitis have decreased long-term survival compared with the general
population
b. Patients with chronic pancreatitis exhibit no excess mortality relative to the general population
c. Excess mortality in patients with chronic pancreatitis is related to cancers of the aerodigestive
system, complications of diabetes, and complications of cirrhosis
d. Excess mortality in patients with chronic pancreatitis is due to development of adenocarcinoma of
the pancreas and to the complications of recurrent pancreatitis
Answer: a, c

Patients with chronic pancreatitis have decreased long-term survival compared to the general
population. An excess of mortality of 30% over 20 years has been estimated. Less than 20% of deaths
are directly attributable to pancreatitis or its complications. Excess mortality is related to
extrapancreatic complications of alcoholism and smoking, including cancers of the aerodigestive
system, complications of diabetes, and complications of cirrhosis.

75. Which of the following is the most common clinical manifestation of chronic pancreatitis?

a. Epigastric pain with radiation to the hypogastrium


b. Diabetes mellitus
c. Steatorrhea
d. Epigastric pain with radiation to the upper lumbar vertebrae
Answer: d

Pain is a predominant symptom complex in most patients with chronic pancreatitis. Chronic pancreatic
pain is usually localized to the epigastrium with radiation to the back in the region of the upper lumbar
vertebrae. Discomfort may be exacerbated by eating and is usually alleviated by abstinence from food
and by bending forward. Malabsorption and weight loss, clinical manifestations of steatorrhea, are only
observed when greater than 90% of exocrine tissue has been destroyed. Clinical signs of malabsorption
are a late manifestation of chronic pancreatitis. Although abnormal glucose tests can be demonstrated
in 50% to 70% of patients with chronic Pancreatitis: overt diabetes mellitus is present in only 30% to
40%. Endocrine deficits are usually progressive. If individual patients are repetitively tested,
progressive deterioration is often observed.

76. For the patient in the preceding question, appropriate management includes which of the
following?

a. Distal pancreatectomy
b. Cystjejunostomy
c. Percutaneous drainage
d. Primary radiotherapy and chemotherapy
Answer: a

The proper treatment is surgical removal of the tumor; aggressive pancreatic resection is appropriate.
It is crucial to avoid mistaking a mucinous cystic tumor for a pancreatic pseudocyst. Internal drainage
of a malignant mucinous cystic tumor results in catastrophic tumor dissemination and should never be
performed. With appropriate treatment, all patients with histologically benign tumors should be cured;
for tumors demonstrating malignant change, 5-year survival after surgery is about 60%.

77. A 72-year-old man develops jaundice and is demonstrated to have a 2.5 mass in the pancreatic
head by computed tomography. There are no signs of unresectability on CT examination. Fine needle
aspiration cytology is positive for adenocarcinoma. Which of the following intraoperative findings
would indicate unresectability?

a. Fibrotic reaction in the body and tail of the pancreas


b. Microscopic tumor cells in perigastric lymph nodes on frozen section
c. Inability to develop an avascular plane anterior to the superior mesenteric vein
d. Cholelithiasis
Answer: b, c

During performance of pancreaticoduodenectomy, the lesser sac is opened widely through the
gastrocolic omentum. This maneuver allows inspection of the body and tail of the gland to determine
the extent of the tumor involvement and allows examination of lymph nodes along the superior and
inferior body of the pancreas and around the celiac axis. Enlarged nodes in these areas should undergo
biopsy and be submitted for frozen-section examination, since tumor in these areas is beyond the
bounds of standard pancreaticoduodenectomy and constitutes a contraindication to resection. If there
is no evidence of lymphadenopathy, a dissection between the anterior surface of the portal vein and
the posterior surface of the neck of the pancreas is performed. Ordinarily, only thin areolar tissue lies
between the pancreas and the portal vein, and a communication behind the neck of the pancreas can
be established. If there is hard tissue intervening and such communication cannot be established, this
implies invasion of the anterior surface of the portal vein and signals unresectability by standard
methods.

78. A 67-year-old male presents with complaints of itching, dark urine, and epigastric pain. Physical
examination reveals jaundice. Initial laboratory tests show total bilirubin of 6.5 mg/dL, alkaline
phosphatase elevated at 3 the upper limit of normal, and mild elevations in serum transaminases.
Appropriate management includes which diagnostic test next?

a. Abdominal ultrasonography
b. Computed tomography of the abdomen
c. Magnetic resonance imaging of the abdomen
d. Endoscopic retrograde cholangiography
Answer: a

Standard transcutaneous ultrasonography is the appropriate first test in the evaluation of the patient
with jaundice, because the presence of a dilated common bile duct or intrahepatic bile ducts is
essentially diagnostic of extrahepatic biliary obstruction. This finding directs the physician to a search
for the cause of the obstruction. If the bile ducts are not dilated, mechanical obstruction is unlikely
and the diagnostic thrust should move toward hepatocellular disease. Ultrasonography is also the best
test to determine whether gallstones are present; this is extremely important because
choledocholithiasis is one of the conditions most likely to cause jaundice in the elderly population.

79. Which of the following statements regarding ductal adenocarcinoma of the pancreas is/are correct?

a. For ductal adenocarcinoma, 60–70% of tumors arise in the head of the gland, 15% in the body, and
10% in the tail, the remainder are diffuse
b. Fifty percent of pancreatic adenocarcinomas involve the gland diffusely at the time of diagnosis
c. For ductal adenocarcinomas, tumors of the body and tail are usually larger at the time of diagnosis
than those arising in the head of the gland
d. Pancreatic adenocarcinomas occur with equal frequency within the head, body, and tail of the gland
Answer: a, c

Sixty to 70% of pancreatic ductal adenocarcinomas occur in the head of the gland. About 15% reside in
the body of the gland, another 10% are in the tail, and the remaining 5% to 15% are diffuse. The
predilection of pancreatic cancer to develop in the head of the gland is unexplained, but has the
practical consequence that tumors in the head are diagnosed earlier because they cause obstructive
jaundice whereas tumors in the body and tail tend to be more advanced at the time of symptomatic
presentation. Tumors in the body and tail are typically larger at the time of diagnosis (average, 7 to 8
cm) than in the head (average, 4 to 5 cm).
80. The most common cause of death in the postoperative period following pancreaticoduodenectomy
is which of the following?

a. Myocardial infarction
b. Intraperitoneal hemorrhage
c. Pulmonary embolism
d. Pneumonia
Answer: b

Pancreaticoduodenectomy is a formidable operation, and until recently, average operative mortality


was reported to approximate 20%. In the past few years, several centers have reported large series
with operative mortalities lower than 5%.
The most dreaded complication of pancreaticoduodenectomy is disruption of the
pancreaticojejunostomy, which occurs in about 10% of patients. Anastomotic breakdown may lead to
the development of an upper abdominal abscess or may present as a external pancreatic fistula. In its
most virulent form, disruption leads to necrotizing retroperitoneal infection which may erode major
arteries and veins of the upper abdomen, including the portal vein or its branches or the stump of the
gastroduodenal artery. Impending catastrophe is often preceded by a small herald bleed from the drain
site. Such an event is an indication to return to the operating room to widely drain the
pancreaticojejunostomy and to repair the involved blood vessel. Open packing of the wound may be
necessary in controlling diffuse necrosis and infection. On rare occasions, completion pancreatectomy
is required to control sepsis. Intraperitoneal hemorrhage is the most common cause of death from
pancreaticoduodenectomy.

81. For the above patient, ultrasonography reveals dilated extrahepatic and intrahepatic bile ducts and
the absence of gallstones within the gallbladder. A 2 cm mass within the pancreatic head is visualized.
Computed tomography of the abdomen is performed. Which of following CT findings indicate probable
unresectability?

a. Common bile duct diameter of 2 cm


b. Ascites
c. Infiltration of the transverse colonic mesentery
d. Dilatation of the main pancreatic duct to 1.5 cm
Answer: b, c
CT scans provide the best available radiologic information to determine whether or not a pancreatic
neoplasm is resectable, but they cannot be considered absolutely definitive in this regard. Only about
half of pancreatic tumors that appear to be confined to the pancreas on CT scan are found to be
resectable in the operating room. CT scanning is more accurate in the diagnosis of unresectability. CT
findings that indicate that the tumor is unlikely to be surgically curable include vascular invasion,
enlarged lymph nodes outside the boundaries of resection, ascites, distant metastases (usually liver),
and distant organ invasion (usually colon). When a CT scan shows distant metastases or extensive local
invasion, the positive predictive value of the technique is high; some 90% of such patients have
unresectable disease at laparotomy.

82. Which of the following have been shown to be risk factors for development of adenocarcinoma of
the pancreas?

a. Cigarette smoking
b. Coffee drinking
c. Adult-onset diabetes mellitus
d. Chronic coumadin usage
e. Prior gastrectomy
Answer: a, e

Most cases of pancreatic cancer have no obvious predisposing host factors. The most consistently
observed risk factor for pancreatic cancer is cigarette smoking. Most studies estimate that smoking
results in a two-to three-fold increase in risk of developing pancreatic cancer. Alcohol consumption has
been implicated in some case-control studies of pancreatic cancer, but the overall evidence is
inconsistent and alcohol is not likely to be a major factor in the development of the disease. Although
considerable public interest was focused in the past few years on coffee consumption as a risk factor
for pancreatic cancer, evidence linking coffee consumption to pancreatic cancer is not compelling.
Abnormal glucose tolerance is present in about 80% of patients with pancreatic cancer, if carefully
sought. Although diabetes and pancreatic cancer occur together far more frequently than would be
expected by chance, persons with long-standing diabetes are not at increased risk of developing
pancreatic cancer. Chronic pancreatitis is a significant risk factor for the development of pancreatic
cancer. It appears that all forms of chronic pancreatitis are associated with an increased risk of
pancreatic cancer, suggesting that it is the pancreatitis and not the injuring agent which is responsible
for the augmented cancer risk. Studies indicate that patients who have previously undergone gastric
resection may be from three to seven times more likely to develop pancreatic cancer than a control
population.
There have been several reports of familial clustering of pancreatic cancer. Recent epidemiological
studies suggest that about 7% of pancreatic cancer patients have a positive family history of the
disease. For most cases, however, no hereditary basis for pancreatic cancer has been identified.

83. Which of the following surgical procedures has the lowest incidence of recurrent jaundice when
used in the context of unresectable carcinoma of the head of the pancreas?

a. Choledochoduodenostomy
b. Cholecystojejunostomy
c. Cholecystoduodenostomy
d. Choledochojejunostomy
Answer: d

When jaundiced patients undergo exploration in the hopes of resection but unresectable disease is
found, biliary bypass should be performed. The jejunum is typically chosen as a conduit in preference
to the duodenum because duodenal obstruction may occur as the tumor becomes more advanced.
There has been much discussion over the use of the bile duct or the gallbladder for biliary
decompression. Operative mortality and mean survival (about 6 months) do not differ between patients
with cholecystojejunostomy and choledochojejunostomy. Recurrent jaundice is more common after
cholecystojejunostomy. Because recurrent jaundice constitutes a failure of palliation, the use of the
common duct for biliary bypass is preferable in most patients. There are circumstances, however, in
which it may be more appropriate to use the gallbladder. Such instances include patients with poor
performance status, cases in which the tumor is bulky and invades the porta hepatis, or when
periductal varices have developed as a result of portal vein thrombosis. The suitability of the
gallbladder as a biliary conduit must be proven intraoperatively. If, on aspiration, the gallbladder
contains colorless fluid, the cystic duct may be assumed to be obstructed, and the gallbladder should
be removed and not used for bypass. If there is green bile in the gallbladder, patency of the cystic duct
should be proved by cholangiography before a bypass is performed.

84. A 45-year-old woman is evaluated for epigastric and back pain. Physical examination is normal.
Computed tomography of the abdomen reveals a 8 cm cystic lesion in the region of the tail of the
pancreas. The cyst demonstrates internal septations and papillary projections from its walls. Which of
the following diagnoses is most likely in this patient?

a. Pancreatic lymphoma
b. Retroperitoneal liposarcoma
c. Pancreatic pseudocyst
d. Pancreatic mucinous cystadenoma
Answer: d

Mucinous cystic neoplasms account for about 2% of pancreatic exocrine tumors. Most patients with
mucinous cystic tumors present with abdominal pain or an abdominal mass. There may be associated
weight loss, steatorrhea, or diabetes. The diagnosis is best made by CT scanning and ultrasonography,
which demonstrate a mass containing fluid-filled structures and internal septations. Occasionally, it is
possible to see the papillary tumor excrescences on the cyst walls.
The tumor occurs six times as often in females as in males. About 80% of the tumors are located in the
body and tail of the pancreas. They present as large (average, 10 cm), soft, and somewhat irregular
tumors. Microscopically, the cysts are lined by columnar epithelium which contains mucin. Although
most of the cells may appear benign histologically, most tumors larger than 3 cm contain areas of
premalignant or malignant change and all mucinous cystic tumors should be considered to have
malignant potential.

85. A 45-year-old woman develops upper gastrointestinal hemorrhage. Evaluation by upper endoscopy
reveals three ulcers in the second portion of the duodenum. Bleeding is controlled using an endoscopic
heat probe. Further investigation reveals a serum gastrin value of 240 pg/mL. Which of the following
would support the presumptive diagnosis of gastrinoma?

a. An increase of 320 pg/mL in serum gastrin upon intravenous infusion of secretin


b. Gastric acid analysis demonstrating fasting acid secretion of 3 mEq/h
c. Enlarged gastric rugae on upper gastrointestinal contrast study
d. An increase of 150 pg/mL in serum gastrin upon intravenous infusion of cholecystokinin
Answer: a, c

The indications for the measurement of gastrin include the presence of peptic ulcer disease, patients
with prolonged undiagnosed diarrhea, patients within MEN-1 families and patients with prominent
gastric rugal folds on upper GI series. In most patients with gastrinoma, the fasting serum gastrin level
is elevated above 200 pg/ml. Gastrin values over 1000 pg/ml are virtually diagnostic of gastrinoma.
However, fasting hypergastrinemia alone is not sufficient for the diagnosis of gastrinoma. Gastric acid
analysis is an important test in the evaluation of patients with suspected gastrinoma, as it can
differentiate between ulcerogenic causes of hypergastrinemia and nonulcerogenic causes of
hypergastrinemia. The diagnosis of gastrinoma is supported by a basal acid output above 15 mEq/hour
in nonoperated patients.
Following documentation that hypergastrinemia is associated with excessive acid secretion,
provocative testing using secretin should be performed to differentiate between gastrinoma, antral G
cell hyperplasia/hyperfunction, and the other causes of ulcerogenic hypergastrinemia. The secretin
stimulation test is carried out in the fasting state by obtaining peripheral serum samples for gastrin in
the basal period, administering secretin (2 units/kg body weight) as an intravenous bolus, and
obtaining serum samples for gastrin at five minute intervals for 30 minutes. An increase in the gastrin
level of more than 200 pg/ml above the basal level is supportive of the diagnosis of gastrinoma.

86. For the patient in the preceding question, an insulin/glucose ratio of 0.5 was documented at 28
hours of fasting. Symptoms of mental obtundation developed concurrently and were reversed by oral
glucose administration. Endoscopic ultrasonography demonstrated a 1.2 cm mass in the head of the
pancreas. Appropriate management consists of which of the following?

a. Surgical enucleation of the tumor


b. Total pancreatectomy
c. Long-term octreotide administration
d. Primary radiotherapy
Answer: a

The treatment of insulinoma is surgical in nearly all cases. Insulinomas are found evenly distributed
within the pancreas, with approximately one-third being located in the head and uncinate process,
one-third in the body of the gland, and one-third in the tail of the gland. Ninety percent of patients
will be found to have benign solitary adenomas amenable to surgical cure. Small benign insulinomas
not in close proximity to the main pancreatic duct may be removed by enucleation, independent of
their location within the gland. In the body and tail of the Pancreas: insulinomas greater than 2 cm in
diameter, and those in close proximity to the pancreatic duct are most commonly excised by distal
pancreatectomy. Large insulinomas deep in the head or uncinate process of the pancreas may not be
amenable to local excision, and may require pancreaticoduodenectomy.
87. A 35-year-old woman is evaluated for seizure disorder, mental obtundation, and personality
change. Physical examination is normal. Fasting serum glucose is 44 mg/dL. Other serum values are
normal. Subsequent investigations should include which of the following?

a. Oral glucose tolerance test


b. Determination of fasting insulin/glucose ratios
c. Assay of serum C-peptide levels
d. Determination of serum prolactin levels
Answer: b, c

A common mistake made in the evaluation of a patient with suspected insulinoma is to commence the
evaluation with an oral glucose tolerance test. Instead, insulinoma is most reliably diagnosed using the
technique of a monitored fast. During a monitored fast, blood for glucose and insulin determinations is
sampled every four to six hours, and at the time of symptom occurrence. Hypoglycemic symptoms
typically occur when glucose levels are less than 50 mg/dl, with concurrent serum insulin levels often
being greater than 25 µU/ml. Additional support for the diagnosis of insulinoma comes from the
calculation of the insulin to glucose ratio (I:G ratio) at different time points during the monitored fast.
Normal individuals will have I:G ratios less than 0.3, while patients with insulinoma typically
demonstrate I:G ratios greater than 0.4 after a prolonged fast. Other measurable beta cell products
synthesized in excess in patients with insulinoma include C peptide and proinsulin. Elevated levels of C
peptide and proinsulin are typically found in the peripheral blood in patients with insulinoma. The
possibility of surreptitious insulin or oral hypoglycemic agent administration should be considered in all
patients with suspected insulinoma. C peptide and proinsulin levels will not be elevated in patients
self-administering insulin. Additionally, patients self-administering either bovine or porcine insulin may
demonstrate anti-insulin antibodies in circulating blood.

88. The most common location(s) for development of gastrinoma is/are which of the following?

a. Pancreas to the right of the superior mesenteric vein


b. Pancreatic body and tail
c. Gastric antrum
d. Duodenum
Answer: a, d

The majority of gastrinomas have been identified to the right of the superior mesenteric vessels within
the head of the pancreas or the duodenum. Intraoperative ultrasonography should be available to assist
in tumor localization. In addition, intraoperative upper endoscopy may be helpful by allowing
transillumination of the duodenal wall and identification of small duodenal gastrinomas. At
exploration, any suspicious peripancreatic lymph nodes are excised and submitted for frozen section.
Primary tumors located within the substance of the pancreas that are small (< 2 cm) and well
encapsulated may be carefully enucleated. Pancreatic tumors without defined capsules or situated
deep in the pancreatic parenchyma may require partial pancreatic resection. In the absence of an
identifiable pancreatic or duodenal tumor, a longitudinal duodenotomy may be performed at the level
of the second portion of the duodenum to allow for eversion of the duodenum in a search for duodenal
microgastrinomas. Primary gastrinomas identified within the duodenal wall are resected locally, with
primary closure of the duodenal defect.

89. Neoplastic hypersecretion of the hormone vasoactive intestinal peptide is associated with which of
the following features?

a. Hypokalemia, hypochlorhydria, diarrhea


b. Hyperglycemia, necrolytic rash, hypoaminoacidemia
c. Constipation, gallstones, hyperglycemia
d. Hyperkalemia, necrolytic rash, diarrhea
Answer: a

Patients characteristically present with intermittent severe diarrhea, typically of a watery nature,
averaging 5 liters/day. Malabsorption and steatorrhea are not common. Hypokalemia results from the
fecal loss of large amounts of potassium (up to 400 meq/day), and low serum potassium levels may be
associated with muscular weakness, lethargy, and nausea. Most patients are hypochlorhydric or
achlorhydric. Half of the patients have some degree of hyperglycemia and hypercalcemia, while
cutaneous flushing can be observed in a minority of patients. The diagnosis of VIPoma is typically made
after excluding other more common causes of diarrhea. The active agent in the VIPoma syndrome is
usually vasoactive intestinal polypeptide (VIP), with a minority of patients having elevations of other
candidate mediators such as peptide histidine-isoleucine (PHI) or prostaglandins.

90. A patient with biochemically confirmed gastrinoma undergoes computed tomography for tumor
localization. CT reveals a 2 cm mass in the head of the pancreas and multiple nodules within right and
left lobes of the liver. Appropriate management includes which of the following?
a. Omeprazole administration
b. Radiotherapy
c. Pancreaticoduodenectomy
d. Proximal gastric vagotomy
Answer: a

Gastrinoma patients whose localization and staging studies are indicative of unresectable hepatic
metastases should undergo percutaneous or laparoscopically-directed liver biopsy for histologic
verification. If unresectable gastrinoma is confirmed, then open surgical exploration is not performed
and the patient is maintained on long-term omeprazole therapy. Virtually all patients can be rendered
achlorhydric with appropriate dose adjustment of omeprazole. Noncompliant patients who refuse to
take appropriate doses of omeprazole and who develop complications related to their ulcer diathesis
may require total gastrectomy for management. Total gastrectomy removes the end organ (parietal
cell mass) and was once the procedure of choice for gastrinoma. Today its use in gastrinoma patients
has markedly declined.

91. The following statement(s) is/are true concerning the widely accepted French or Couinaud’s
nomenclature for liver anatomy.

a. The liver is divided into eight discrete segments based on portal pedicle branches and hepatic
venous drainage
b. This anatomy is particularly useful in allowing less than lobar segmental anatomical resections that
minimize blood loss and loss of hepatic reserve
c. Enumeration of the system begins from right to left
d. Segments II and III are synonymous with the left lateral segment based on English nomenclature
Answer: a, b, d

In the now widely accepted French (Couinaud’s) nomenclature, the liver can be divided into eight
discrete segments based on portal pedicle branches and hepatic venous drainage. Enumeration of the
segments begins left to right, beginning with segment I, the caudate lobe. The left lateral sector
consists of a superior segment II and an inferior segment III and is synonymous with the left lateral
segment in older terminology. The major advantage to this detailed segmental anatomy, which is based
on discrete portal pedicle branches, is to accurately locate individual lesions in the hepatic substance
by preoperative imaging and intraoperative ultrasound and to allow the possibility of less than lobar
segmental anatomical resections that minimize blood loss and functional loss of hepatic reserve.

92. In the patient described above, which of the following are important operative steps in the
performance of a right hepatic lobectomy?

a. The use of an ultrasonic dissector is essential for division of the hepatic parenchyma
b. If temporary portal inflow occlusion is used (Pringle maneuver), it is not necessary to reestablish
blood flow during the course of the parenchymal division
c. The greater omentum may be used to buttress the transected liver edge
d. Control of the main right hepatic vein should eliminate all forms of venous drainage
Answer: c

The steps involved in a right hepatic lobectomy involve adherence to the tenet of optimal operative
exposure and control of vascular inflow and outflow. In select circumstances, control of the vena cava
may be desired. Either the individual portal structures can be identified and ligated early in the course
of the procedure, or simply the entire portal triad can be circled with an umbilical tape tourniquet in
preparation for the Pringle maneuver. If temporary portal inflow occlusion is used, intermittent 10 to
20 minute intervals of clamping with 3 to 5 minutes to reestablish blood flow is recommended. The
division of the hepatic parenchyma begins with scoring of Glisson’s capsule with cautery or knife and
proceeds with division of the hepatic surface using either blunt dissection by finger fracture, the blunt
edge of an instrument or suction tip, or using an ultrasonic dissector. Individual vessels and bile ducts
are cauterized, sutured, or clipped in rapid succession from anterior to posterior. The hepatic veins are
encountered in the hepatic substance near the vena cava and are carefully clamped and suture ligated
to complete the resection. In addition, there are also several posterior accessory veins (up to 10 in
number) which drain the medial aspect of the right lobe and empty directly into the right anterior
surface of the IVC.

93. Intraoperative ultrasound is now commonly used by the hepatic surgeon. Which of the following
statement(s) is/are true concerning intraoperative ultrasound and hepatic surgery?

a. An intraoperative ultrasound offers no advantage to conventional transcorporial ultrasound in


detection of hepatic lesions
b. Portal structures can be differentiated from hepatic veins by the extension of Glisson’s capsule
surrounding these structures
c. It is difficult on ultrasound to differentiate a vascular structure from a mass
d. The short hepatic veins are difficult to detect with intraoperative ultrasound
Answer: b

Over the past 10 years, detailed anatomic description of the hepatic veins, portal pedicles, and the
inferior vena cava have been possible through the use of intraoperative ultrasound. Cooperation
between radiologists and hepatic surgeon with the use of intraoperative ultrasound has allowed the
identification of lesions during surgery that were not visible by conventional transcorporial ultrasound
or CT scanning. Beginning superiorly at the inferior vena cava, the confluence and course of each of
the hepatic veins can easily be determined. More inferiorly, the main right and left portal pedicles can
be seen coursing transversely in the transverse scissura. Portal structures can easily be differentiated
from hepatic veins by the hyperechoic extensions of Glisson’s capsule which surround these structures.
When a circular structure is encountered, a mass or metastasis may be suspected. Scanning away from
the mass may reveal a tubulovascular shape which has been imaged and cross sectioned. Flattening of
the circular mass by external compression with the ultrasound probe will also differentiate a vascular
structure from a solid mass.

94. The following statement(s) is/are true concerning hepatic anatomical nomenclature.

a. In the traditional English system, the right lobe is divided into anterior and posterior segments by an
intersegmental line with no topographic landmarks or interparenchymal septi
b. The caudate lobe in the French or Couinaud’s nomenclature is referred to as segment I
c. The right lobe of the liver by English nomenclature is subdivided in the French system into segments
V–VIII
d. In the English system, the left lobe of the liver is divided into the medial segment and lateral
segment by the falciform ligament
Answer: a, b, c, d

Until recently, anatomic descriptions in the English literature began with the major divisions of liver
into a right and left lobe separated by a vertical line drawn from the gallbladder fossa to the inferior
vena cava. The left lobe is further divided by the falciform ligament into a medial segment and lateral
segment. The right lobe is further divided into an anterior and posterior segment by an intersegmental
line which has no reliable topographical landmarks and no interparenchymal septi to allow easy
identification. The French nomenclature also known as Couinaud’s nomenclature enumerates the
segments of the liver beginning with segment I or the caudate lobe. Segments II, III, and IV make up
most of the “English” left lobe, while segments V through VIII represent the English nomenclature right
lobe.

95. A 57-year-old man with a history of Duke’s C colon cancer is being evaluated for a rising CEA.
Which of the following statement(s) is/are correct concerning the use of CT scanning for this
indication?

a. Conventional CT scanning will detect lesions well below 1 cm in size


b. CT arterio-portography involves immediate CT scanning after direct injection into both the common
hepatic artery and superior mesenteric artery
c. A double helical (spiral) CT scan may eliminate the need for invasive angiography
d. Magnetic resonance imaging of the liver will add little to the workup of this patient
Answer: b, c, d

CT scanning has been used increasingly to screen for hepatic and other intra-abdominal or
retroperitoneal lesions. Conventional CT scanning includes 0.5–1 cm axial images of the liver after oral
administration of barium and bolus injection of intravenous contrast. Although resolution has improved,
hepatic lesions below 1 cm in size or lesions that are isodense with hepatic parenchyma may be missed.
Resolution of hepatic lesions has been greatly enhanced by the combination of visceral angiography and
CT scanning, known as CT arterio-portography (CTAP). Immediate CT scanning after injection of
contrast directly into the common hepatic artery may identify small hepatic lesions which usually show
increased density relative to the surrounding hepatic parenchyma. CT arterio-portography also includes
direct injection of contrast into the splenic or superior mesenteric arteries, with CT imaging during the
portal venous phase of this injection. Hepatic lesions supplied by the hepatic artery thus appear as
discrete hypodense lesions surrounded by normal hepatic parenchyma enhanced by portal venous
contrast. Recently, double helical (spiral) CT scanning has become available and shows considerable
promise to complement or replace CTAP for preoperative imaging. This technique allows total hepatic
imaging in both the arterial and arterial/venous phases after a single rapid bolus injection of
intravenous contrast during a single breath hold by the patient. It is possible to visualize the portal
structures and hepatic veins on a single scan and give a high resolution of small hepatic lesions. In
addition, three-dimensional reconstructions can be created to further delineate hepatic parenchyma
and demonstrate a CT constructed hepatic arteriogram. This technique may completely replace the
need for invasive arteriography to characterize the blood supply to the liver prior to hepatic resection
or after hepatic transplantation. Magnetic resonance imaging of the liver has results similar to CT
scanning, but to date has not demonstrated improvements sufficient to justify the increased cost
associated with the technique.

96. A solitary 6 cm lesion is identified in the right hepatic lobe in the patient described above. Which
of the following statement(s) is/are true concerning the initial operative management?

a. To facilitate mobilization and assessment with intraoperative ultrasound, complete mobilization


including dividing the left and right triangular ligaments would be necessary
b. In dividing the right triangular ligament, care must be taken to avoid injury to accessory right
hepatic veins draining directly into the vena cava
c. Unless a considerable length of hepatic vein is found outside the hepatic parenchyma, early hepatic
vein ligation should be avoided
d. Ligation of the portal arterial structures is always necessary before proceeding with hepatic
lobectomy
Answer: a, b, c

For major hepatic resections and for complete intraoperative ultrasound, complete mobilization of the
liver will be required. After detachment of the hepatic flexure of the colon and division of the
falciform ligament, both the left and right triangular ligaments must be sharply taken down to fully
mobilize the liver. During division of the right triangular ligament, care must be taken to avoid injury
to the right diaphragm, the right adrenal gland and adrenal vein, the right phrenic vein, and several
moderate-size accessory right hepatic veins draining directly into the vena cava. After mobilization,
digital and bimanual palpation is performed and intraoperative ultrasound may be performed.
Dissection of the porta hepatis is performed by many hepatic surgeons to identify the main bifurcations
of the hepatic artery, bile duct, and portal vein. This allows individual ligation of unilateral branches
of each of these structures during hepatic lobectomy but prior to parenchymal dissection. An
alternative approach has been recently described where the main portal structures are left undisturbed
and branches to a given lobe are ligated during parenchymal transection. Hemorrhage can be
minimized by intermittent portal inflow occlusion by clamping or compression of the portal triad
(Pringle maneuver). There has been considerable debate over early versus late isolation and ligation of
a given hepatic vein during lobectomy since the extraparenchymal component of the hepatic vein may
be quite short or absent. Since hemorrhage in this location may be difficult to control, a safe strategy
is to always avoid early isolation of a given hepatic vein or to attempt isolation only when a
considerable length of vein is found on mobilization of the respective triangular ligament.
97. Which of the following statement(s) is/are true concerning the arterial venous anatomy of the
liver?

a. Most commonly, the right, left, and middle hepatic veins join the inferior vena cava as a separate
trunk
b. Most frequently, the entire length of each hepatic vein is within the parenchyma of the liver
c. A replaced right hepatic artery may be placed in jeopardy during performance of a
pancreaticoduodenectomy
d. There is little collateral arterial circulation between the right and left hepatic lobes
Answer: b, c

There are three major hepatic veins which carry blood from the central veins of the hepatic substance
to the inferior vena cava (IVC). In two-thirds of patients, there is a single large right hepatic vein which
joins the right anterior wall of the IVC and a middle and a left hepatic vein which converge one-to-two
cm from the IVC and enter the left anterior wall of the IVC as a single vessel. In one-third of patients,
each major hepatic vein joins at the same horizontal level of the IVC as a separate trunk. In some
patients, there is a short but definable extraparenchymal segment of one or more of the hepatic veins
at the confluence with the IVC. More frequently, the entire length of the hepatic veins is
intraparenchymal, which may preclude early, safe hepatic venous isolation during hepatic resection.
There is considerable variability in the origin and course of the right and left hepatic arteries. The
most common finding (55% of patients) is a transverse common hepatic artery from the celiac trunk
which gives off the gastroduodenal, right gastric, and supraduodenal arteries and courses obliquely in
the left anterior aspect of the hepatoduodenal ligament as a proper hepatic artery. After giving off the
cystic artery to the gallbladder, there is then a fairly low trifurcation into a single right, middle, and
left hepatic arteries. Knowledge of the most common variations is extremely importance since
inadvertent division may occur during gastric, pancreatic, and hepatobiliary procedures. There may be
a replaced or accessory left hepatic artery which arises from the left gastric artery and courses
transversely in the lesser omentum. With nearly equal frequency, there is a replaced or accessory right
hepatic artery from the superior mesenteric artery near its origin which courses posterior or through
the head of the pancreas obliquely along the right posterior border of the hepatoduodenal ligament.
Although original anatomic descriptions deny the existence of collateral vessels to the opposite hepatic
lobe, image perfusion studies after ligation of main or replaced hepatic arteries have clearly
demonstrated the presence of collateral flow to the deprived lobe.

98. The liver plays a vital role in carbohydrate metabolism and regulation of blood glucose. The
following statement(s) is/are true concerning carbohydrate metabolism by the liver.

a. Glycogen, a complex polymer of glucose, is synthesized by the hepatocyte in a remarkably energy


efficient process
b. Glucagon stimulates glycogenesis
c. Glycolysis, the process by which glucose is converted to two molecules of pyruvate, occurs in the
liver mitochondria
d. If glycogen stores become depleted, the liver is capable of synthesizing new glucose by the process
of gluconeogenesis, which is stimulated by insulin
Answer: a

Serum glucose is tightly regulated by the liver despite wide fluctuations in dietary ingestion. The liver
can take up as much as 100 g/day of glucose and convert it to glycogen by the process of glycogenesis.
The liver can also release glucose into the blood by glycogenolysis, the breakdown of glycogen, or by
gluconeogenesis, the formation of new glucose from substrates such as alanine, lactate, glycerol or
dietary amino acids. Hormones play a key role in hepatic regulation of glucose metabolism. Insulin, for
example, stimulates glycogenesis, and glucagon stimulates glycogenolysis and gluconeogenesis.
Gluconeogenesis is also enhanced by fasting, critical illness and periods of anaerobic metabolism.
Glycogen is a complex polymer of glucose. Liver cells can store up to 8% of their weight as glycogen.
The first step in glycogen storage is the transport of glucose through the hepatocyte plasma
membrane. About 90% of portal venous glucose is removed from the blood by liver cells through
carrier-facilitated diffusion. The rate of glucose transport is enhanced by insulin. Once in the
hepatocyte, glucose and ATP are converted by the enzyme glucokinase to glucose-6-phosphate (G6P),
the first intermediate in the synthesis of glycogen. Because complete oxidation of one molecule of G6P
generates 37 molecules of ATP, and storage only uses one molecule of ATP, the overall efficiency of
glucose storage in glycogen is a remarkable 97%. Glycolysis is the pathway by which glucose is
converted to two molecules of pyruvate and occurs in the cytoplasm in contrast to the citric acid cycle
which occurs in the mitochondria.

99. Transport of substances from the blood into the hepatocyte occurs through the sinusoidal
membrane. The following statement(s) is/are true concerning this plasma membrane.

a. The high lipid content of this phospholipid bilayer allows lipid-soluble molecules to enter the cell by
simple diffusion
b. Carrier proteins within the phospholipid bilayer bind to a solute in blood and by conformational
change allow it to be transported into the cell
c. Large glycoprotein molecules of the sinusoidal membrane known as receptors always transport the
binding ligand into the cell
d. The transmission of a signal to the interior of the cell by receptor-ligand binding which generates
intracellular second messengers is known as signal transduction
Answer: a, b, d

The hepatocyte plasma membrane consists of a phospholipid bilayer in which hydrophobic fatty acid
tails are oriented to the interior membrane and hydrophilic phospholipid head groups are oriented to
the exterior (sinusoidal or cytoplasmic) membrane. Within this phospholipid bilayer are proteins which
serve either structural functions or metabolic functions. The hepatocyte sinusoidal plasma membrane is
heavily studded with microvilli to increase the absorptive area in contact with sinusoidal blood. The
cell membrane, by virtue of its high lipid content, allows lipid-soluble molecules to enter the cell by
simple diffusion. Polar molecules must enter cells via membrane transport proteins. Channel proteins
allow molecules to diffuse simply into cells without binding, whereas carrier proteins first bind the
solute and, by conformational change, allow it to be transported into the cell. The glucose carrier in
hepatocytes is an example of carrier-facilitated diffusion. The sinusoidal membrane is studded with
receptors, which are large glycoprotein molecules that span the plasma membrane lipid bilayer. A
ligand-binding site of this receptor molecule projects into the space of Disse. When appropriate ligand-
receptor binding occurs, the entire ligand may be internalized for intracellular degradation or biliary
transport, or the ligand may transmit a signal to the interior of the hepatocyte by a number of
intracellular second messenger systems, a process known as signal transduction. Such second
messengers include cAMP, inositol triphosphate, and diacylglycerol. Each of these structurally simple
chemicals can amplify cell membrane events and bring about major changes in cellular physiology.

100. The liver is an important site of protein metabolism. Which of the following statement(s) is/are
true concerning protein metabolism by the liver?

a. Amino acids are taken up by hepatocytes by active transport mechanisms and are generally stored
long-term for later synthetic activity
b. Under certain conditions the amine group is removed from the amino acids in the liver and the
carbon chain used for carbohydrate, lipid, or nonessential amino acid synthesis
c. The most important route of detoxification of ammonia formed as the result of deamination of
amino acids is via excretion of ammonia into the urine
d. Proteins synthesized by the liver include albumin, transferrin, fibrinogen, and apolipoproteins
e. Albumin is a sensitive indicator of hepatic synthetic function
Answer: b, d

Essentially all of the end products of dietary protein digestion are amino acids, which are absorbed by
the enterocytes into the portal circulation in ionized states. Amino acids are taken up by hepatocytes
by one of several active transport mechanisms. Amino acids are not stored in the liver but are rapidly
used in the production of plasma proteins, purines, heme proteins, and hormones. Under certain
conditions, the amine group is removed from the amino acids, and the carbon chain is used for
carbohydrate, lipid, or nonessential amino acid synthesis. The ammonia formed as the result of
deamination of amino acids is detoxified by one of two routes. The most important pathway involves
conversion of ammonia to urea by enzymes of the Krebs-Henseleit cycle, found only in the liver. A
second route of ammonia metabolism involves deamination of L-glutamine by the kidney, with
excretion of ammonia into the urine.
Essentially all albumin, fibrinogen, and apolipoproteins are derived from the liver which can add up to
50 grams of protein to the plasma per day. Of total hepatic protein synthesis, 75% is destined for
export in plasma. Albumin, an important plasma protein synthesized in the liver, has a long half-life in
plasma of about 19 days. This long half-life makes albumin an insensitive indicator of hepatic synthetic
function.

101. The following statement(s) is/are true concerning hepatic blood flow.

a. Although constituting only 2.5% of total body weight, the liver receives 25% of the cardiac output
b. Hepatic blood flow is equally derived from the portal vein and hepatic artery
c. The liver serves as a physiologic blood reservoir either releasing blood back into the systemic
circulation at times of acute blood loss or in situations of volume overload serving as a site of extra
blood storage
d. An important function of the liver is to filter particulate debris which is performed by phagocytic
Kupffer cells which line the hepatic sinusoidal endothelium
Answer: a, c, d

The liver constitutes about 2.5% of the total body weight but receives 25% of the cardiac output. Total
hepatic blood flow is 100 to 130 ml/min/kg. About two-thirds of total hepatic blood flow is derived
from the portal vein and one-third from the hepatic artery. The liver also serves as a physiologic blood
reservoir. About 25% to 30% of the liver volume is accounted for by blood, and in cases of acute blood
loss up to 30%, or as much as 300 ml of the hepatic blood volume can be released into the systemic
circulation without adverse effects on liver function. Conversely, in the case of right heart failure or
other causes of systemic volume overload, as much as one liter of extra blood can be stored in the liver
before passive congestion and liver injury occur. The hepatic sinusoids are lined by an endothelium
punctuated with pores that allow proteins and other particles to diffuse out of the vascular tree and
into proximity with hepatocytes. This extreme permeability of the liver allows rapid exchange of a
diverse number of nutrients, hormones and environmental agents between the blood and the
hepatocyte. The liver also acts as a filter for particulate debris, which enters the portal circulation
through intestinal capillaries. Particles such as bacteria are ingested by Kupffer cells by the process of
phagocytosis. Kupffer cells line the hepatic sinusoidal endothelium where formed blood elements and
matter may be in direct contact with these phagocytic cells.

102. The liver synthesizes key metabolites essential for the production of fuel substrates for other
organs. These key metabolites include:

a. Glucose-6-phosphate (G6P)
b. Acetyl CoA
c. Pyruvate
d. Oxaloacetate
Answer: a, b, c

Hepatic processes in the liver are essential for the production of fuel substrates for other organs. The
liver, by virtue of its terminal position in the portal system, is the organ that must regulate intestinally
absorbed nutrients for tissue consumption or storage. The liver accomplishes its task by synthesizing
three key metabolites: -glucose-6-phosphate, pyruvate and acetyl CoA. G6P can be stored as glycogen
or converted into glucose, pyruvate, or ribose-5-phosphate (a nucleotide precursor). Pyruvate can be
converted into lactate, alanine (and other amino acids), and acetyl CoA, or it can enter the
tricarboxylic acid cycle. Acetyl CoA is converted to HMG-CoA (a cholesterol and ketone body precursor)
or citrate (for fatty acid and triglyceride synthesis), or it is degraded to carbon dioxide and water for
energy.

103. The following statement(s) concerning hepatic bile formation/secretion is/are true.

a. The adult human liver secretes less than 1000 cc of bile daily
b. Most bile is secreted by hepatocytes (canalicular bile)
c. Primary bile acids include cholic acid, chenodeoxycholic acid, and deoxycholic acid
d. The enterohepatic circulation is tremendously efficient in reabsorption of intestinal bile acids
e. Bile acids are the primary determinant of bile flow
Answer: b, d, e

The adult human liver secretes about 1.5 liters of bile daily. Eighty percent of this volume is secreted
by the hepatocytes (canalicular bile) and 20% is secreted by the bile duct epithelial cells (ductular
bile). Solutes constitute about 3% of bile. The major solutes are conjugated bile acids, phosphatidyl
choline, cholesterol, protein and bilirubin. Bile acids are the main determinant of bile production, and
canalicular bile flow is traditionally divided into bile acid-dependent and bile acid-independent
components. Primary bile acids are synthesized from cholesterol in the liver and in humans consist of
cholic acid and chenodeoxycholic acid. Secondary bile acids are formed in the intestinal lumen by
bacterial dehydroxylation and consist of deoxycholic acid and lithocholic acid derived from cholic acid
and chenodeoxycholic acid, respectively. Essentially all primary and secondary bile acids are
conjugated with the amino acids glycine or taurine. The human liver synthesizes 300 to 400 mg per day
of bile acids from cholesterol, or about 10% of the total bile salt pool. Normally intestinal bile acids are
efficiently (about 95%) taken up by the enterohepatic circulation. Luminal bile acids are transported by
carrier proteins in the distal ileum and appear in the portal venous effluent. The hepatocyte extracts
more than 95% of portal venous bile acids for resecretion into the bile.

104. The following statement(s) is/are true containing lipid metabolism in the liver.

a. Hepatic mitochondrial hydrolysis of fatty acids is a tremendous source of ATP


b. Significant hepatic storage of triglyceride or fatty infiltration can cause hepatic fibrosis or necrosis
c. Approximately 90% of cholesterol synthesis occurs in the liver
d. Most cells in the body are capable of phospholipid synthesis, therefore the liver plays a minimal role
in this process
Answer: a, c

The liver has a number of important functions in the metabolism of lipids: 1) the synthesis of
apolipoproteins, 2) the degradation of fatty acids into energy substrates, 3) the synthesis of
triglycerides from carbohydrates and proteins, and 4) the synthesis of cholesterol and phospholipids
from fatty acids. The mitochondrial hydrolysis of fatty acids is a source of large quantities of ATP. The
conversion of stearic acid to CO2 and H2O, for instance, generates 136 ATP molecules and
demonstrates the highly efficient storage of energy in fat. In times of unrestrained lipolysis, such as
starvation, uncontrolled diabetes or other conditions of triglyceride mobilization from adipose tissue,
the ability of the liver to perform beta-oxidation may be inadequate. Under these circumstances,
significant hepatic storage of triglycerides or fatty infiltration of the liver may occur. Triglyceride
storage by itself does not appear to be a cause of hepatic fibrosis or necrosis, but fatty infiltration may
be a marker for derangement of normal processes by alcohol or drug toxicity, diabetes, chronic
parenteral nutrition, or morbid obesity.
Cholesterol is an important regulator of membrane fluidity and is a substrate for bile acid and steroid
hormone synthesis. Cholesterol may be available by dietary intake or by de novo synthesis. In
mammals, about 90% of new cholesterol is synthesized by the liver from its precursor, acetyl CoA.
Dietary cholesterol intake suppresses endogenous synthesis by inhibiting the rate-limiting enzyme in
cholesterol by a synthetic pathway, HMG-CoA reductase. There are three major classes of
phospholipids synthesized by the liver: the lecithins, the cephalins, and the sphingomyelins. Although
most cells in the body are capable of some phospholipid synthesis, the liver produces 90%.

105. Hepatic biotransformation is defined as the intracellular metabolism of endogenous and exogenous
organic compounds. Which of the following is/are enzyme families responsible for hepatic bile
transformation?

a. Cytochromes P-450
b. UDB-glucuronyl transferases
c. Glutathione-transferases
d. Sulfotransferases
Answer: a, b, c, d

The liver contains enzyme systems that can expose functional groups such as hydroxyl ions and alter
the size and solubility of a wide variety of organic and inorganic compounds by conjugation with small
polar molecules. The general strategy of the liver is to convert hydrophobic, potentially toxic
compounds into hydrophilic conjugates that can then be excreted into bile or urine. There are four
general enzyme families responsible for hepatic bile transformation. The cytochromes P-450 catalyze
reactions such as oxidation, hydroxylation, sulfoxide formation, oxidative deamination,
dealcoholization and dehalogination. Such reactions allow further phase II conjugation with polar
groups such as glucuronate, glutathione and sulfate. Glucuronidation is the conjugation of UDB-
glucuronic acid to a wide variety of xenobiotics by either esther or ether linkages. The glutathione
transferases and sulfotransferases play a role in conjugation of P-450 derivatives. However, the
glucuronyl transferase system is the predominant mechanism.
106. The following statement(s) is/are true concerning the differential diagnosis between an amoebic
and a pyogenic liver abscess.

a. The clinical presentation is often clearly distinguishable


b. A history of travel or origin from a high risk area might suggest an amebic liver abscess
c. Routine liver chemistries frequently can distinguish pyogenic from amoebic liver abscess
d. Serologic testing for the presence of antibody to entamoeba histolyctica is the only specific and
sensitive way to confirm the diagnosis of amoebic liver abscess
e. Distinguishing pyogenic from hepatic abscesses preoperatively is not important since surgical
drainage is imperative for both
Answer: b, d

Distinguishing amoebic from pyogenic liver abscess can be a diagnostic challenge. It is of major
importance, however, because effective medical therapy with metronidazole can obviate the need for
either percutaneous or surgical drainage in most cases of amoebic abscess. The clinical presentation
for both conditions with acute onset of fever, abdominal pain, and altered liver function tests are
almost identical. Important features such as travel to or origin from a high risk area is particularly
important for amebic liver abscess. Routine liver chemistries and radiographic studies can rarely
distinguish between amoebic and pyogenic liver abscesses. Specific serologic tests for the presence of
antibody to E. histolytica are specific and sensitive for amoebic hepatic abscess being positive in 95% of
the cases, and therefore, are key in distinguishing the two infections.

107. A patient is found to develop evidence of hepatitis approximately eight weeks after receiving
blood transfusions during a surgical procedure. Which of the following statement(s) is/are true?

a. The virus responsible is most likely hepatitis C


b. A chronic carrier state will ultimately develop in most patients
c. There is no role for interferon in the treatment of chronic hepatitis C viral infection
d. Chronic infection with hepatitis C is not associated with an increased risk of developing
hepatocellular carcinoma
Answer: a, b

Hepatitis C virus is a virus that is responsible for more than 90% of post-transfusion hepatitis and most
sporadic non-A, non-B hepatitis throughout the world. The most common identifiable sources of
acquisition of hepatitis C virus are prior transfusion of blood or blood-derived products or a history of
intravenous illicit drug use. The usual incubation period of post-transfusion hepatitis C viral infection is
5 to 10 weeks. An initial elevation of liver enzymes may be associated with little or no clinical
disturbance. In some patients, acute hepatitis C viral infection does not progress to chronic infection,
however, chronic hepatitis C viral infection develops in up to 70% of patients with post-transfusion
hepatitis C infection with many progressing to cirrhosis. Hepatitis C does not appear to alter life
expectancy at least in the first 15 years of infection. However, once cirrhosis and end stage liver
disease develop, the clinical syndrome is indistinguishable from other forms of chronic liver disease
with a predisposition to the development of hepatoma. Interferon alpha is the only FDA approved
therapy for chronic hepatitis C viral infection. There is some evidence that early administration of
interferon in acute hepatitis C viral infection may reduce the risk of progression to the chronic state.
As yet, there is no evidence that interferon alters the natural history of chronic hepatitis C viral
infection or changes the incidence.

108. A surgeon is suspected of having contacted hepatitis B virus via needle stick. Which of the
following statement(s) is/are true concerning his diagnosis and outcome?

a. Incubation of hepatitis B virus is about two weeks


b. Jaundice is the first serologic indicator of hepatitis B infection
c. The patient has about a 10% chance of developing a chronic carrier state
d. All susceptible household or sexual contacts of the surgeon should receive hepatitis B viral vaccine
e. The surgeon should receive hepatitis B immunoglobulin as soon as possible after the accidental
needle stick
Answer: c, d, e

Hepatitis B viral infection is insidious. The incubation period of the virus is about eight weeks. The first
serum indicator of infection by hepatitis B virus is detection of the serum hepatitis B surface antigen
(HBsAg) which may proceed the onset of jaundice. In most cases, hepatitis B infection is self-limited
and does not progress to chronic hepatitis. However, some 10% of patients with acute hepatitis B viral
infection, whether it is clinical or subclinical, will develop a chronic carrier state. The carrier state is
defined by the presence of HBsAg in serum for longer than six months. The best method of treatment
of hepatitis B viral infection is primary prevention by vaccination. All susceptible household or sexual
contacts of a person with a positive serum test for HBsAg should be advised to receive a full course of
hepatitis B viral vaccine. Passive prophylaxis with hepatitis B immunoglobulin should be provided to any
susceptible contact in whom there is recent potential parenteral exposure such as an accidental needle
stick.

109. The following statement(s) is/are true concerning the diagnosis and treatment of hydatid cysts.

a. Percutaneous aspiration is an important aspect of diagnosis and treatment of a hydatid cyst


b. CT scan will oftentimes show the classic findings of a cystic liver lesion with a calcific rim
c. At operation, care must be taken to protect the operative field from spillage of the cyst fluid
d. The use of a scoleocide has become obsolete with current surgical techniques
Answer: b, c

Hydatid cysts are most commonly the result of infection with the tape worm, Echinococcus granulosis.
Routine laboratory tests in patients with hydatid cysts are normal or nonspecifically abnormal.
Although routine chest or abdominal radiographs may show a mass with a calcific rim, sonography and
CT scan are the favored means of imaging hydatid cysts. The presence of calcifications and daughter
cysts within the parent cyst suggests Echinococcus. Percutaneous needling of a hydatid cyst is unwise
unless precautions against anaphylaxis are undertaken. A cyst’s fluid is often under pressure, and
needling may precipitate rupture with the potential for anaphylaxis or intraperitoneal seating. The
classic treatment of hydatid cysts is operative. The surgical aim is to remove the cyst or cysts without
dissemination of the organism. At operation, the cyst is drained of fluid through a cannula after
carefully protecting the operative field from fluid leakage. If the aspirate is clear a parasiticidal fluid
(ethyl alcohol or 20% sterile saline) is injected into the cyst to kill any adherent scoleces. The cyst
contents and the pericystic wall is then removed with careful surgical dissection.

110. Which of the following statement(s) is/are true concerning treatment of pyogenic liver abscess?

a. Antibiotic therapy alone may be advisable in patients with multiple small abscesses
b. Percutaneous drainage provides comparable results to surgical drainage in patients with unilocular
large abscesses
c. Sufficient antibiotic coverage for most hepatic abscesses includes coverage for gram-positive aerobic
bacteria only
d. In patients with a primary biliary origin for the hepatic abscess, treatment must also be addressed at
underlying biliary pathology such as choledocholithiasis or biliary ductal obstruction
Answer: a, b, d
The preferred treatment of most patients with hepatic abscesses is broad-spectrum antibiotic coverage
and drainage. A number of studies have demonstrated for most patients with large unilocular abscesses
that percutaneous catheter drainage is as effective as surgical drainage. Bacteria that predominate in
pyogenic liver abscesses are gram-negative aerobes, streptococcal species, and anaerobes. Therefore,
broad-spectrum antibiotic coverage is necessary. Antibiotic coverage alone may be advisable in
occasional patients who have multiple small abscesses not accessible to percutaneous or surgical
drainage. Since many of these patients have an underlying biliary pathology as the source of the
hepatic abscess, correcting this underlying pathology, for example, establishing biliary drainage
surgically or nonoperatively is important.

111. Which of the following statement(s) is/are true concerning acute, fulminant hepatic failure?

a. The most frequent cause of acute hepatic failure world-wide is hepatitis B infection
b. Higher grades of encephalopathy are associated with a worse prognosis
c. Hypoglycemia is a common complication of all liver diseases
d. Liver transplantation would appear indicated in all patients with hepatic coma secondary to acute
liver failure
Answer: b

The diagnosis of acute (fulminant) hepatic failure is based on the development of encephalopathy
within eight weeks of the onset of symptoms. The overall prognosis is poor, but the hepatic lesions are
potentially reversible, and recovery can lead to restoration of normal liver function. The most frequent
cause of acute hepatic failure world-wide is non-A, non-B viral hepatitis. A variety of other viral agents
and hepatotoxins can also cause this condition.
No reliable criteria predict outcome and response to treatment. Higher grades of encephalopathy
(depth of coma) on admission are associated with the worst prognosis. Management should include
general supportive measures and specific treatment for hepatic encephalopathy, cerebral edema,
electrolyte and metabolic disturbances, infection, and pain. Hypoglycemia is an unusual complication
of most liver diseases except in patients with acute hepatic failure or hepatic neoplasms. The
enormous reserve capacity of the liver accounts for the rarity of hypoglycemia except as a preterminal
event. Bleeding is also a frequent cause of death in patients with acute hepatic failure secondary to
depressed liver synthesis of clotting factors and qualitative or quantitative platelet disorders. The lack
of a definitive medical treatment for acute hepatic failure makes liver transplantation seem attractive
especially for patients with little or no chance of recovering normal liver function. Perhaps the most
significant drawback to widespread acceptance of liver transplantation for acute hepatic failure is the
lack of criteria reliability to predict which patients are likely to benefit from operation. Patients with
mild to moderate degrees of coma are likely to recovery spontaneously without the need for liver
transplantation while rapid deterioration and neurologic status to grade III or grade IV coma are
associated in some centers with a mortality of 95%.

112. Which of the following statement(s) is/are true concerning the natural history and clinical
features of alcoholic cirrhosis?

a. In patients with compensated cirrhosis, the probability of survival at 10 years approaches 50%
b. The development of clinical evidence of hepatic decompensation reduces five year survival to less
than 20%
c. Continued consumption of alcohol worsens prognosis
d. The risk of death after variceal hemorrhage depends more on the severity of underlying liver disease
than the type of therapy
Answer: a, b, c, d

Recent studies have analyzed the natural history of cirrhosis as a function of the degree of hepatic
decompensation at the time of diagnosis. A high proportion of patients with compensated cirrhosis
remain well for many years after diagnosis. In these studies the probability of remaining compensated
10 years after diagnosis was 42%, and survival probability of compensated patients was 47%. The
prognosis worsened considerably once patients developed clinical evidence of hepatic decompensation
(ascites, jaundice, encephalopathy, or gastrointestinal hemorrhage). Among these patients, the
probability of five-year survival was only 16%. The risk of death from variceal hemorrhage depends
much more of the severity of the underlying liver disease than on the type of therapy. It would also
appear from natural history studies that continued alcohol consumption does affect survival. In one
study, the overall five-year survival is 63% for abstainers versus 40.5% for those who continued to drink.
Continued alcohol consumption may have less of an effect on survival than the intensity of alcohol
consumption. Furthermore, the degree of hepatic compensation at the time of inclusion into the study
may have also been an important factor.

113. Which of the following statement(s) is/are true concerning the morphologic and histologic findings
of cirrhosis?
a. Micronodular cirrhosis is a pattern typical of chronic alcoholic liver disease
b. Mallory bodies and megamitochondria are typical findings of alcoholic cirrhosis
c. Bile leaks caused by rupture of bile ducts with extravasation of bile into portal triads is a common
finding in post-necrotic cirrhosis secondary to hepatitis
d. Large regenerating nodules separated by coarse irregular scars in piecemeal parenchyma necrosis is
common in liver disease secondary to chronic active hepatitis
Answer: a, b, d

Morphologic classification of cirrhosis includes micronodular, macronodular, and mixed forms.


Micronodular cirrhosis is characterized by uniform nodules and scars. The nodules are usually less than
3 mm in diameter and are typically associated with Laennec or nutritional cirrhosis in alcoholics. Post-
necrotic cirrhosis is characterized by large regenerating nodules separated by coarse irregular broad as
well as thin scars. This pattern is frequently seen in patients with viral hepatitis. Biliary cirrhosis is
characterized by a coarsely granular macronodular liver. This condition results from long-standing
cholestasis secondary to obstruction of intrahepatic or extrahepatic bile ducts. The most distinctive
feature of large duct obstruction is the presence of bile leaks caused by rupture of bile duct with
extravasation of bile into portal triads. Portal cirrhosis, which is typically observed in alcoholics, can
generally be distinguished histologically by the presence of several specific hepatocellular alterations
such as Mallory bodies and megamitochondria.

114. The following statement(s) is/are true concerning the management of ascites associated with
chronic liver disease.

a. Spontaneous bacterial peritonitis is an insignificant complication


b. Large volume paracentesis is unsafe due to excessive volume loss from the intervascular space
c. Peritoneovenous shunting is a trivial surgical procedure with minimal perioperative morbidity and
mortality
d. Transjugular intrahepatic portosystemic shunts (TIPS) can effectively treat ascites in patients
refractory to conventional medical therapy
Answer: d

The onset of ascites usually indicates the presence of advanced liver disease. Cirrhotic ascites is
usually straw colored, clear, or greenish. Spontaneous bacterial peritonitis occurs as a complication of
cirrhotic ascites in up to 10% of patients. Spontaneous bacterial peritonitis is defined as infected ascitic
fluid without a demonstrable other site of infection. This is a serious complication with reported in-
hospital mortality rates of 60% to 90%. The rational approach of therapy for ascites includes sodium and
fluid restriction, the use of diuretics, and the use of therapeutic paracentesis. Several studies have
shown that repeated paracentesis in stable cirrhotic patients may be safe and effective as medical
therapy and shortens the length of hospitalization. Single, large volume paracentesis has been reported
to be effective and safe. Up to 10 liters of ascites can be removed in one hour if salt-poor albumen is
administered simultaneously. In a small percentage of patients, surgical implantation of a
peritoneovenous shunt may be advisable. The principal indication for use of peritoneovenous shunt is
to stabilize ascites that is refractory to conventional medical therapy and therapeutic paracentesis.
Despite the simplistic nature of the device, postoperative mortality and morbidity rates of 20% to 60%,
respectively have been reported. Precipitation of disseminated intravascular coagulopathy, variceal
hemorrhage, or hepatic failure may complicate this procedure. Transintrahepatic portosystemic shunts
(TIPS) have been demonstrated to control ascites in one study in over 90% of patients with ascites
refractory to medical management. However, patients with poor hepatic reserve in this study all died
if orthotopic liver transplantation was not performed. This data suggests that TIPS is effective for
refractive ascites in patients with good to moderate hepatic reserve but poor risk cirrhotics require
orthotopic liver transplantation to correct this problem.

115. Which of these statement(s) is/are true concerning the etiologic factors in the development of
cirrhosis?

a. Viral hepatitis of any type (A, B, or non-A, non-B) can all progress to cirrhosis
b. Acetaminophen can cause acute liver failure and necrosis but will not lead to cirrhosis
c. Alcohol exerts toxic effects on the liver via reactive intermediates such as acetaldehyde
d. Long-standing congestive heart failure can lead to cirrhosis secondary to centrilobular congestion,
hemorrhage, and necrosis
Answer: c, d

Liver cells are sensitive to a variety of physical, microbiologic, and chemical agents, all of which may
produce cellular injury. The eventual development of cirrhosis is determined by the nature and
severity of the cellular injury and the liver’s ability to regenerate. Most infectious hepatitides are viral
in origin. There is no documentation that hepatitis A progresses to cirrhosis, however hepatitis B and
non-A, non-B hepatitis do demonstrate a propensity to become chronic with the development of
cirrhosis. Chemical hepatotoxicity include direct and indirect actions. Acetaminophen under normal
circumstances is detoxified chiefly by conjugation with glucuronic acid or sulfate. Saturation of
glucuronic pathway with large doses of acetaminophen results in progressive depletion of intracellular
glutathione stores, accumulation of toxic intermediate, and eventual cell necrosis which may progress
to either acute liver failure or chronic cirrhosis. Alcohol can affect liver cell function in a number of
ways. Like many hepatotoxins, the toxic effects of alcohol are caused indirectly by reactive
intermediates. Acetaldehyde is the principal reactive compound generated by alcohol metabolism. The
hepatotoxicity of acetaldehyde is related to its binding two major constituents of cellular membranes
altering membrane integrity and enzymatic function frequently to the detriment of the cell. Early
morphologic changes of long-standing cardiac decompensation and right-sided heart failure are central
lobular congestion, hemorrhage, and necrosis combined with phlebosclerosis of central veins and scars
connecting centrizonal areas (cardiac cirrhosis).

116. Important spontaneous portosystemic collaterals which develop in the face of portal hypertension
include:

a. The hemorrhoidal veins


b. Left renal vein
c. The paraumbilical venous plexus
d. The coronary, short gastric, and paraesophageal veins
Answer: a, b, c, d

If portal pressure is elevated, spontaneous portosystemic collaterals develop in an attempt to


decompress the portal system. Such collaterals increase venous return to the heart and increase
cardiac output. In humans, the most important collaterals develop as tributaries of the coronary, short
gastric, and paraesophageal veins; intercostal, esophageal, and azygous veins; the superior, middle,
and inferior hemorrhoidal veins; and the peraumbilical plexus. Retroperitoneal veins and veins draining
to the left renal vein from the splenic, adrenal, and gonadal veins may also serve as sites of the
development of venous collaterals.

117. Which of the following statement(s) is/are true concerning the pathophysiology of variceal
hemorrhage?

a. All patients with portal hypertension will develop esophageal varices


b. All patients with esophageal varices eventually bleed
c. Variceal size can predict the incidence of variceal hemorrhage
d. Control of acid secretion by H2 blockade can decrease the incidence of rebleeding after esophageal
hemorrhage
e. None of the above
Answer: e

About two-thirds of patients with portal hypertension develop varices; of these only two-thirds
subsequently experience variceal hemorrhage. A number of factors are important in the pathogenesis
of variceal hemorrhage. These include portal pressure, intravariceal pressure, variceal size and
structure, and other factors. Variceal size alone is not predictive of variceal hemorrhage. Evidence
conflicts about whether erosive esophagitis is a cause of variceal rupture. Control of acid reflux by H2
blockade has not been shown to decrease the incidence of rebleeding after esophageal hemorrhage.

118. Which of the following statement(s) is/are true concerning the use of transjugular intrahepatic
portosystemic shunts (TIPS) in the treatment of variceal bleeding?

a. This procedure effectively creates an end-to-side portocaval shunt


b. Procedure-related mortality is generally in excess of 20%
c. TIPS has been used successfully as a pretransplant procedure to reduce portal pressure
d. The placement of a TIPS is not associated with the development of encephalopathy
Answer: c

Transjugular intrahepatic portosystemic shunts (TIPS) refer to an implantable, expandable metal stent
placed radiologically through the hepatic parenchyma to establish a track between branches of the
hepatic and portal veins. TIPS results in similar hemodynamics as a side-to-side portal systemic shunt.
There is firm clinical data that TIPS provides effective control of acute variceal hemorrhage and portal
hypertension regardless of the etiology of the underlying liver disease or the degree of hepatic
decompensation. TIPS has also been used for preoperative portal decompression to facilitate orthotopic
liver transplantation. Pretransplant TIPS should reduce portal pressure thereby reducing operative time
and blood loss. The major complications of TIPS include encephalopathy and stenosis or occlusion of
this stent. Encephalopathy occurs in 10% to 20% of patients after TIPS. This complication appears to
correlate with increasing age of the patient and increased shunt diameter and shunt flow.

119. Hepatic encephalopathy is a common systemic manifestation of chronic liver disease. Which of the
following statement(s) is/are true concerning this condition?
a. Blood ammonia levels correlate well with the stage of hepatic encephalopathy
b. Alterations in central nervous system neurotransmitters such as the neurotransmitter g-
aminobutyric acid (GABA) have been proposed in the pathogenesis of hepatic encephalopathy
c. Lactulose can be used to decrease intestinal ammonia absorption
d. Patients can be expected to have an increased sensitivity to benzodiazepines
Answer: b, c, d

Hepatic encephalopathy, a poorly explained neuropsychiatric syndrome, characterized by diverse


neurologic abnormalities, is the pathologic evidence of nonspecific structural changes in neurons, and a
variable prognosis. Several hypotheses to explain the pathogenesis of hepatic encephalopathy have
been proposed. Ammonia has been widely implicated in the pathogenesis of hepatic encephalopathy
despite conflicting evidence. Blood ammonia levels correlate poorly with the stage of encephalopathy,
however, one of the mainstays of treatment is measures to decrease ammonia absorption from the gut
including the oral administration of lactulose. Another hypothesis has implicated false
neurotransmitters in the pathogenesis of hepatic encephalopathy. Evidence suggests that activation of
the GABA system may be important in the pathogenesis of hepatic encephalopathy. The GABA receptor
binds several classes of ligands including GABA and drugs such as benzodiazepines. For reasons that are
unclear, hepatic failure appears to increase the brain density of GABA receptors. This observation may
explain the increased sensitivity to benzodiazepines and other inhibitory neurotransmitters observed
with patients with chronic liver disease.

120. Which of the following statement(s) is/are true concerning the management of gastroesophageal
variceal hemorrhage?

a. Vasopressin decreases portal pressure through the process of splanchnic vasoconstriction


b. Somatostatin is as effective as vasopressin but without the cardiac side effects
c. Balloon tamponade provides good long-term control of bleeding esophageal varices
d. Endoscopic sclerotherapy is more effective than conservative medical therapy in the treatment of
bleeding esophageal varices
e. Sclerotherapy, although excellent for the control of bleeding short-term, does not prolong overall
survival
Answer: a, b, d

A number of invasive and noninvasive therapies exist for the treatment of bleeding esophageal varices.
Vasopressin is frequently used in the treatment of acute variceal hemorrhage and acts by decreasing
portal venous pressure or flow through splanchnic vasoconstriction. Vasopressin alone has been
reported to temporarily control variceal hemorrhage in 50% to 75% of patients, however, it can be
associated with cardiac and peripheral extremity cutaneous ischemia. Somatostatin also acts as a
vasoconstrictor to reduce splanchnic flow, with trials demonstrating similar efficacy in controlling
acute hemorrhage when compared to vasopressin but without the cardiac side effects. Balloon
tamponade is generally used for the temporary control of acute variceal hemorrhage unresponsive to
vasopressin or sclerotherapy. Initial control of acute variceal hemorrhage occurs in about 80% of
patients, but bleeding recurs promptly on deflation of the balloons in over 50%. Endoscopic
sclerotherapy has become the primary treatment for bleeding esophageal varices. A number of clinical
trials demonstrate that emergent sclerotherapy is able to halt variceal bleeding that fails to respond to
more conservative measures, with no increase in frequency or severity of complications. The long-term
survival of patients treated with sclerotherapy continues to be debated. Metaanalysis has been
performed on the data from seven randomized clinical trials evaluating the effect of repeated
sclerotherapy on long-term survival. This analysis demonstrates that sclerotherapy reduces the number
of deaths by 25% therefore supporting the use of sclerotherapy as an effective means of prolonging
survival in patients who have experienced variceal hemorrhage.

121. Which of the following statement(s) is/are true concerning the surgical management of bleeding
esophageal varices.

a. A side-to-side portacaval shunt may be associated with the development of hepatofugal blood flow
b. Selective shunts preserve prograde (hepatopedal) blood flow while decompressing esophageal
varices or reducing portal pressure
c. The presence of intractable ascites is a contraindication to the Warren shunt
d. If the patient is considered a liver transplant patient, an interposition mesocaval shunt is a suitable
alternative
Answer: a, b, c, d

Although portosystemic shunts are the most effective therapy for preventing recurrent variceal
hemorrhage, they are associated with the increased incidence of encephalopathy. A number of types of
shunts have been described, primarily to avoid the consequences of complete diversion of portal blood
flow or to simplify the operation. End-to-side portocaval shunts are hemodynamically unique in that all
portal flow is diverted and the hepatic limb of the portal vein is ligated, thus preventing hepatofugal
blood flow from the liver. With lateral side-to-side shunts, the hepatic limb of the portal vein remains
patent. A greater compensatory increase in hepatic arterial flow occurs when the portal vein serves as
an outflow track and the liver extracts oxygen and metabolites from the blood exiting through the
patent limb of the portal vein (hepatofugal). The goal of selective shunts is to preserve prograde
(hepatopedal) portal flow to the liver while selectively decompressing gastroesophageal varices
(Warren shunt) or reducing portal pressures sufficiently to prevent variceal hemorrhage (small-
diameter shunts). Most surgeons with a special interest in this field attempt to construct a selective
shunt when the operation is elective, unless contraindications are present. The distal splenorenal shunt
of Warren and the small-diameter interposition portocaval shunt of Sarfeh are the most common
selective shunts used. The Warren shunt is an ascitogenic operation and therefore the presence of
ascites that is difficult to control medically is a contraindication of this operation. An interposition
mesocaval shunt is frequently preferred in emergent situations because the shunt is relatively safe to
construct and promptly halts variceal hemorrhage. If future hepatic transplantation is contemplated,
this shunt offers the advantages in that it does not involve dissection of the area of the
hepatoduodenal ligament.

122. Which of the following statement(s) is/are true concerning the results of portosystemic shunting?

a. When comparing shunts with nonshunting procedures, only minor differences in long-term survival
are reported, but the mode of death usually changes
b. Distal splenorenal shunts are associated with the development of less hepatic encephalopathy
c. Survival statistics following distal splenorenal shunt in good risk patients (Child’s Class A) are in
excess of 80%
d. None of the above
Answer: a, b, c

Multiple randomized comparisons of distal splenorenal shunt with other types of portosystemic shunts
have been reported. Operative mortality and long-term prognosis are similar, but patients undergoing
distal splenorenal shunts develop less encephalopathy. Less encephalopathy is also seen with the
selective Sarfeh shunt and after nonshunting procedures, such as sclerotherapy or devascularization.
When comparing shunts with nonshunting procedures, only minor differences in long-term survival are
reported, but the mode of death usually changes. With nonshunting procedures, a greater proportion of
patients die of recurrent hemorrhage; after construction of a shunt, a greater proportion die of hepatic
failure. Class A patients undergoing distal splenorenal shunts can be predicted to have an actuarial
survival including operative mortality in excess of 80%. This is a substantial improvement over previous
experience with portocaval shunts and is likely better than the results that can be obtained with
hepatic transplantation.
123. Which of the following statement(s) is/are true concerning radiologic examinations used for the
assessment of hepatic neoplasms?

a. Magnetic resonance imaging is considered the test of choice for distinguishing hemangiomas from
other mass lesions
b. Dynamic CT scanning is the dominant imaging modality for routine screening and diagnosis
c. Radionucleotide imaging plays an important role in modern screening and detection for liver lesions
d. CT angio-portography (CTAP) is the gold standard for early detection of metastatic hepatic lesions
Answer: a, b, d

In the Western world, the test most commonly used for screening, detection, and diagnosis of hepatic
neoplasms are dynamic bolus-enhanced CT scan, MRI, and ultrasound. Dynamic CT scanning remains
the dominant imaging modality for routine screening and diagnosis because it is sensitive, widely
available, provides helpful anatomic information and allows the evaluation of other intraabdominal
structures in the detection of extrahepatic disease. The most sensitive test for the detection of liver
masses is CTAP, however, like angiography is generally reserved for preoperative evaluation of
resectability and not for screening. Hepatic MRI imaging is another useful tool in the detection and
diagnosis of the liver lesions, with sensitivities equal to that of CT. MRI with T1 and T2-weighted
images and gadilinium enhancement can aid in the differential diagnosis of a mass. For example, MRI is
now considered the test of choice for distinguishing hemangiomas from other mass lesions.
Radionucleotide imaging is less sensitive and specific than CT, MRI, or ultrasound, and has a high
percentage of false-positive and false-negative results. Therefore, nuclear medicine scans play a
limited role in modern screening and detection, but they can help differentiate discrete masses. 

124. A 45-year-old woman undergoes an ultrasound because of vague right upper quadrant pain and
epigastric fullness. A 7 cm cystic lesion is detected. Which of the following statement(s) is/are true
concerning the patient’s diagnosis and management?

a. Simple aspiration is indicated for treatment and diagnosis


b. Bile stained fluid suggests underlying biliary pathology
c. The cyst is likely lined by cuboidal epithelium
d. Laparoscopic unroofing of the cyst can provide satisfactory treatment
Answer: c, d
A symptomatic solitary hepatic cyst may cause vague right upper quadrant discomfort or pain, a
sensation of epigastric fullness or heaviness, and early satiety, however, most cases are asymptomatic.
Complications are rare but include hemorrhage into the cyst, secondary bacterial infection, or
obstructive jaundice from compression of extrahepatic ducts. In the absence of complications,
laboratory abnormalities are uncommon. Cysts are somewhat more common in females, are more
common in the right lobe and are often multilocular rather than unilocular. They are lined with
cuboidal epithelium resembling bile duct epithelium and are filled with fluid that may be clear,
mucoid, bloody, or bilious.
If the patient has no symptoms and the cyst was discovered incidentally and there is no evidence of
infection or malignancy, one may observe the patient. Neither percutaneous aspiration nor surgery is
indicated. Cysts nearly always recur after simple aspiration. Treatment of symptomatic cysts is
surgical. Indications for surgery include symptoms, rupture, hemorrhage, or infection. Asymptomatic,
uninfected simple cysts are best treated by excision, if possible. Larger cysts may be inroofed with free
peritoneal drainage unless there is a history of hemorrhage or evidence of biliary communication. A
laparoscopic approach to the unroofing of cysts has recently been reported with excellent success. If
this cyst communicates with the biliary system (grossly by cholangiography) the leak may be oversewn
or the cyst drained by a Roux-en-Y cystojejunostomy.

125. Which of the following statement(s) is/are true concerning biopsy techniques for hepatic masses?

a. A fine needle aspiration (FNA for cytology is contraindicated for patients with hypervascular masses)
b. Percutaneous biopsy should be performed only if results may obviate the need for exploratory
laparotomy
c. Needle track seeding of tumor is not a risk associated with percutaneous biopsy
d. Laparoscopy and biopsy play little role in the management of liver lesions
Answer: b

Biopsy of a liver mass can be done percutaneously (with or without CT or ultrasound guidance),
laparoscopically, or at laparotomy. The biopsy may be done for cytology only (FNA) or for histology
(larger-core biopsy). Guided FNA has an overall sensitivity of 77% to 94% and may allow a distinction
between primary and secondary malignancy. The risks associated with needle biopsy include bleeding,
infection, needle track seeding of tumor, and sampling error. Hypervascular masses, coagulopathy, and
ascites are contraindications to percutaneous core biopsy, however, FNA biopsy is generally considered
safe under these circumstances. In evaluation of any liver mass, percutaneous biopsy should be
performed only if it can reasonably be expected to obviate the need for exploratory laparotomy. Biopsy
of suspected primary metastatic malignancy with clinical indications of unresectability may spare the
patient an unnecessary laparotomy. Laparoscopy with biopsy may also be used to evaluate liver masses
and to possibly avoid laparotomy in patients considered to be borderline resectable.

126. A 55-year-old woman presents with vague right upper quadrant pain and a palpable liver.
Laboratory tests are normal and a noncontrast CT scan (patient has a history of contrast allergy)
reveals an 8 cm right hepatic mass. Which of the following statement(s) is/are correct concerning the
patient’s diagnosis and management.

a. A gadilinium-enhanced MRI would be indicated to define the extent of the lesion and confirm the
diagnosis of hemangioma
b. A fine needle aspiration should be performed regardless of radiographic workup
c. Hepatic embolization is the treatment of choice
d. The lesion should be resected because of concern for malignant degeneration
Answer: a

Cavernous hemangiomas of the liver are the most common benign hepatic tumor and are detected in
some 2% to 7% of autopsies. Overall, hemangiomas are exceeded only by hepatic metastases as the
most common hepatic tumor. Cavernous hemangiomas consist histologically of cystically dilated,
endothelium-lined vascular spaces. They occur in all ages but are observed more commonly in females.
Hemangiomas are not premalignant. Less than half of affected patients have symptoms. Those that
have symptoms usually have large masses. Symptoms usually occur including vague right upper
quadrant discomfort, pain, fullness, and early satiety. Physical examination may be notable for
hepatomegaly, mass or bruit. There are no laboratory abnormalities in patients with hemangiomas. The
most useful radiologic test for diagnosing hemangiomas are MRI, CT and tagged red blood cell scanning.
These tests have largely replaced angiography. CT with vascular contrast often demonstrates a
diagnostically characteristic enhancement pattern. Gadilinium-enhanced MRI has recently been shown
to be sensitive and specific in the diagnosis of hemangioma and has better resolution than tagged red
blood cell scans. FNA biopsy of suspected hemangiomas can be performed, however this procedure
should be avoided if the diagnosis is secure using noninvasive procedures. Given the natural history of
hemangiomas and its low risk of rupture, observation is indicated for asymptomatic patients, especially
for lesions smaller than 4 cm. Surgical excision is the only consistently effective treatment for
symptomatic masses and should be performed if the lesion is localized and accessible with an
acceptable operative risk. Embolization is indicated only for unresectable lesions and is only modestly
effective.

127. Which of the following statement(s) is/are true concerning the pathogenesis of hepatocelluar
carcinoma?

a. Hepatocellular carcinoma is associated with infection with hepatitis A, hepatitis B, and hepatitis C
b. The risk of developing hepatocellular carcinoma related to hepatitis B viral infection is related to
the chronicity of the infection
c. Hepatocellular carcinoma only develops in cirrhosis associated with hepatitis
d. Alfatoxin B1 is a potent carcinogen in the development of hepatocellular carcinoma
Answer: b, d

Hepatocellular carcinoma has three well-known epidemiologic associations: hepatitis B infection,


cirrhosis, and various hepatotoxins, most notably aflatoxin B1 (a microtoxin from the fungus Aspergillus
flavus). Hepatocellular carcinoma is strongly associated with hepatitis B viral infection but there is no
link between the neoplasm and the hepatitis A viral infection. There does appear to be some link
between hepatocellular carcinoma and hepatitis C virus. Cirrhosis, predominantly macronodular, is a
frequent result of hepatitis B viral infection and is a risk factor for hepatocellular carcinoma.
Nevertheless, the micronodular form, which is more common in early alcoholic cirrhosis, will progress
to hepatocellular carcinoma in about 2% to 3% of cases. Thus, hepatocellular carcinoma can develop in
patients with hepatitis B viral infection and cirrhosis, hepatitis B viral infection alone, and with
cirrhosis from any cause alone.

128. A 38-year-old woman with a 17 year history of oral contraceptive use presents with right upper
quadrant pain. A CT scan demonstrates a 4 cm lesion in the right lobe of the liver. Which of the
following statement(s) is/are true concerning the patient’s diagnosis and management?

a. The lesion is likely premalignant


b. A 99mTc sulfur colloid scan will distinguish this benign lesion from a malignant hepatoma
c. The lesion, although benign, may be associated with life-threatening hemorrhage
d. The lesion would be expected to be hypervascular on angiographic study
Answer: c, d

Hepatic adenomas are clearly linked with the use of oral contraceptives. The likelihood of developing a
hepatic adenoma appears to be related to the duration and dosage of estrogen, and is greater at ages
above 30 years. The majority (75%) of these lesions occur in the right lobe of the liver. Although
adenomas may be a symptomatic, most patients have symptoms, usually of abdominal pain (as many as
50%), and 10% to 33% of patients present with acute signs and symptoms secondary to bleeding or
rupture with intraperitoneal hemorrhage.
No radiologic test is specific for adenomas. CT is the most useful preliminary test and often reveals
areas of hemorrhage and necrosis. Angiography may add to CT findings by demonstrating the
hypervascular tumor with a peripheral blood supply. 99mTc sulfur colloid scans show a cold spot
thereby distinguishing adenoma from focal nodular hyperplasia but not from other solid masses. For
asymptomatic patients or patients with minimal symptoms, surgery is the treatment of choice given
the tumors have the tendency to bleed and a small chance of coexisting malignancy.

129. Which of the following statement(s) is/are true concerning focal nodular hyperplasia (FNH)?

a. The lesion predominantly affects young women


b. The lesion is associated with the use of oral contraceptives and other estrogens
c. Radionucleotide scanning can be useful in the specific diagnosis of FNH
d. Excisional biopsy is indicated in almost all cases because of the risk of bleeding
Answer: a, c

Focal nodular hyperplasia (FNH) should not be confused with a hepatic adenoma. Although FNH
predominantly affects young women, it is also found in men and children. Unlike adenomas, there is no
clear relationship between oral contraceptives and the development of FNH. FNH is most commonly
asymptomatic and does not have a propensity to bleed or undergo malignant change. Histologically,
FNH contains normal-appearing hepatocytes, bile ducts, and Kupffer cells in distinction to adenomas.
Radionucleotide imaging can be useful in diagnosing FNH because FNH is the only lesion that contains
Kupffer cells and therefore appears isodense rather than a filling defect. Treatment of asymptomatic
patients is conservative when the diagnosis is clear. If there is doubt regarding the diagnosis, then
excisional biopsy is indicated for small, easily removable lesions.

130. Which of the following statement(s) is/are true concerning the prognosis of patients with hepatic
metastases and colorectal carcinoma?

a. Over half of these patients will survive one year without treatment
b. Five year survivals following hepatic resection for an isolated metastasis is in excess of 25%
c. Survival beyond five years after resection suggests a high probability of cure
d. Survival rates are improved with a margin of resection greater than 1 cm
e. The size of a liver metastasis is not a significant factor in predicting recurrence if adequate margins
can be obtained
Answer: b, c, d, e

Studies consistently report five-year survival rates averaging 25% for hepatic resection for colorectal
metastases. Those who survive beyond five years seem to do well with only an additional 5% dying of
recurrence within the next five years. Median survival of patients with untreated metastases is on the
order of three to 10 months, with only 20% surviving past one year. Overall survival is significantly
improved with surgical margins greater than 1 cm with decreased survival in patients with positive
margins or margins less than 1 cm in size. The number of metastasis is a less consistent but statistically
significant factor. Patients with four or more metastases have a poorer prognosis. As for the size of the
metastasis, it is not a significant factor except that a larger total liver volume of metastasis requires a
larger hepatic resection. Larger size may preclude adequate margins and indicate longer development
of time with an increased likelihood of micrometastases.

131. Which of the following statement(s) is/are true concerning the treatment and prognosis of
hepatocellular carcinoma?

a. The fibrolamellar variant of hepatocellular carcinoma has a distinctly better prognosis than other
forms of the disease
b. Patients with untreated hepatocellular carcinoma rarely survive a year
c. Multiagent chemotherapy is extremely effective in hepatocellular carcinoma and should be
considered for the treatment for most patients
d. Hepatic artery ligation or embolization has been demonstrated to be highly effective for
hepatocellular carcinoma
Answer: a, b

Hepatocellular carcinoma has a tendency for local and vascular invasion. Untreated, patients have a
dismal prognosis with a median survival of only three to four months after diagnosis with only rare
survival beyond one year. The fibrolamellar variant of hepatocellular carcinoma has a distinctly better
prognosis with a five year survival after resection of 50% to 60% versus 25% for other varieties. Surgical
resection offers the only chance for cure for primary hepatic malignancy, and survival is better if
tumors are small and asymptomatic. The role of orthotopic liver transplantation for hepatocellular
carcinoma continues to evolve. For patients with severe hepatic dysfunction or larger or centrally
located tumors or bilobar tumors, this may be the preferred approach. Extrahepatic disease including
lymph node involvement is a contraindication to resection or transplantation. No single-or multiple-
agent chemotherapy is particularly effective in treating hepatocellular carcinoma. The rationale for
treating vascular liver tumors with arterial obstruction is the fact that the nutritional blood supply of
these tumors comes primarily from the hepatic artery. Hepatic artery ligation or embolization alone or
in combination with regional chemotherapy has been shown to be ineffective for liver cancer. In
addition, hepatic artery ligation has a high complication rate and cannot often be used in patients with
compromised liver function from cirrhosis.

132. The following statement(s) is/are true concerning the relationship of the biliary tree and the
hepatic artery and portal vein.

a. The common hepatic and common bile duct lie immediately anterior to the portal vein
b. The cystic artery, which usually arises from the right hepatic artery, crosses behind the hepatic duct
in most cases
c. A replaced right hepatic artery arising from the superior mesenteric artery system will run to the
right of the common bile duct
d. The arterial supply of the extrahepatic biliary ducts is derived from major trunks running along the
medial and lateral walls of the common duct at the 3 o’clock and 9 o’clock position
Answer: a, b, c, d

The common hepatic duct passes downward in the superior and lateral portions of the hepatoduodenal
ligament and lies in front of the portal vein and to the right of the hepatic artery. The cystic artery
which in most cases arises from the right hepatic artery usually crosses behind the hepatic duct (84%)
of cases but is sometimes anterior to that structure (16%) of cases. A number of anatomic variations of
the arterial supply of the liver can be seen. In patients in which the right hepatic artery arises from the
superior mesenteric artery system, the "replaced" right hepatic artery usually runs to the right of the
bile duct and portal vein. The arteries to the extrahepatic biliary ducts anastomose freely within the
duct walls. The ductal arterial supply is derived primarily from the gastroduodenal and right hepatic
arteries with major trunks running along the medial and lateral walls of the common duct at the 3
o’clock and 9 o’clock positions.
133. An understanding of the anatomy of the extrahepatic biliary tree is essential in performing biliary
tract surgery. Which of the following statement(s) is/are true concerning biliary ductal anatomy?

a. The majority of patients have the "classic" anatomical description


b. The common hepatic duct unites with the cystic duct to form the common bile duct
c. An accessory right hepatic duct occurs in 5% of patients
d. A common channel or "Y" configuration of the distal bile duct and pancreatic ducts occur in
approximately 70% of patients
Answer: b, c, d

The classic anatomical description of the extrahepatic bile ducts and their arteries is present in only
one-third of individuals. The left hepatic duct usually has a longer extrahepatic course than does the
right hepatic duct. The common hepatic duct is formed by the union of the right and left hepatic ducts
close to the emergence from the liver. The duct passes downward in the superior portion of the
hepatoduodenal ligament and lies in front of the portal vein and to the right of the hepatic artery. The
common hepatic duct unites with the cystic duct to form the common bile duct. An accessory right
hepatic duct occurs in 5% of cases. The cystic duct passes downward, backward, and to the left in the
hepatoduodenal ligament and usually unites with the main hepatic duct at an acute angle. Its course
and mode of insertion into the common duct is highly variable. The common bile duct is formed by the
union of the common hepatic and cystic ducts and is usually about 7 to 9 cm long. The junction of the
distal common bile duct and pancreatic duct at the ampulla can take one of three configurations. In
approximately 70% of patients, there is a common channel of the bile and pancreatic ducts, thus a "Y"
configuration. In approximately 20%, the common channel is nonexistent, and in another 10%, the two
ducts enter the duodenum via separate openings.

134. A 35-year-old female presents with typical biliary colic symptoms, however her sonogram shows
no gallstones. Which of the following statement(s) is/are true concerning her diagnoses?

a. Chronic acalculous cholecystitis or gallbladder dyskinesia is seldom associated with classic biliary
colic symptoms
b. The most specific test for diagnosing gallbladder dyskinesia is CCK-enhanced cholescintigraphy with
assessment of gallbladder ejection fraction
c. An ejection fraction greater than 75% is considered abnormal and indicative of gallbladder dyskinesia
d. Cholecystectomy is not indicated for chronic acalculous cholecystitis
Answer: b
Motility abnormalities of the gallbladder and cystic duct present with symptoms suggesting gallstones.
The most common presentation for patients with gallbladder motility disorders such as chronic
acalculous cholecystitis or gallbladder dyskinesia is recurrent biliary-type pain. Currently, the most
specific test for diagnosing gallbladder dyskinesia is CCK-enhanced cholescintigraphy with assessment
of gallbladder ejection fraction. CCK is infused intravenously 15 to 30 minutes after ejecting an
analogue of 99MTC imminodiacetic acid and calculating the ejection fraction of the isotope by the
contracting gallbladder. An ejection fraction of less than 35% is considered abnormal and
cholecystectomy may be indicated. Most patients will have relief of symptoms following
cholecystectomy.

135. The following statement(s) is/are true concerning the embryology of the biliary tree.

a. The primordial anlagen of the liver and biliary tract arises from the entoderm
b. Superior and inferior caudal buds form as the hepatic diverticulum develops
c. The development of the liver is a separate process from that of the gallbladder and distal biliary
tree
d. The biliary tree develops in association with the dorsal pancreas
Answer: a, b

The primordial anlagen of the liver, extrahepatic bile ducts, gallbladder, and ventral part of the
pancreas develop as a thickened area of entoderm on the ventral surface of the caudal portion of the
foregut where it joins the midgut. Superior and inferior caudal buds form as the hepatic diverticulum
grows out into the ventral mesogastrium. The solid mass of endodermal cells spreading with this
cephalic bud forms the right and left lobes of the liver. The superior growth of the cranial portion of
the hepatic diverticulum, which extends from the duodenum to the liver, results in the formation of
the hepatic, common hepatic, and common bile ducts. The caudal portion of the hepatic diverticulum
develops into the gallbladder and cystic duct. The common bile duct is attached to the ventral aspects
of the duodenum and is in close contact with the ventral pancreatic bud.

136. The following statement(s) is/are true concerning biliary motor function.

a. The contracted sphincter of Oddi impairs bile flow into the duodenum and directs it into the
gallbladder
b. In the postprandial state about 70% of hepatic bile flows into the gallbladder before reaching the
duodenum
c. During the interdigestive period, only a small fraction of gallbladder bile enters the duodenum
d. Gallbladder emptying during fasting is associated with phase III of the interdigestive migrating motor
complex (MMC)
e. After a meal, the gallbladder empties by a steady tonic contraction thought to be due to release of
endogenous motilin from the mucosa of the small intestine
Answer: a, b, c

As bile is secreted from the liver, it flows through the hepatic ducts into the common hepatic duct and
continues through the common bile duct into the duodenum. With an intact and contracted sphincter
of Oddi, bile flows directly into the gallbladder where it is concentrated and stored. In the postprandial
state, about 70% of hepatic bile flows into the gallbladder before reaching the duodenum and entering
the enterohepatic cycle. During the interdigestive phase, 90% of bile from the liver enters the
gallbladder, while only a small fraction of gallbladder bile enters the duodenum. Gallbladder emptying
during fasting is associated with phase II of the interdigestive migrating motor complex (MMC). Motilin
may account for this stimulatory effect since plasma elevations of motilin seem to correlate with the
onset of phase II waves. Following a meal, the gallbladder empties by a steady tonic contraction
thought to be due to release of endogenous cholecystokinin (CCK) from the mucosa of the small
intestine.

137. The following statement(s) is/are true concerning the anatomy of the gallbladder.

a. The gallbladder lies between the right, left, and quadrate hepatic lobes or hepatic segments IV and
V
b. The cystic duct contains the spiral valve of Heister which serves an important valvular function for
the gallbladder
c. The cystic artery arises from the right hepatic artery in 95% of cases
d. The cystic artery crosses anterior to the hepatic duct in the majority of cases
Answer: a, c

The gallbladder is a pear-shaped organ bound to a fossa on the right inferior surface of the liver
located between the right, left, and quadrate hepatic lobes or hepatic segments IV and V. The
gallbladder can be divided into four areas: the fundus, body, infundibulum, and neck. The body of the
gallbladder extends from the fundus into the tapered portion, or neck, which curves backward and
upward toward the transverse fissure of the liver and terminates in the cystic duct. The cystic duct
lumen contains a thin mucosal septum, the spiral valve of Heister; the valve may make catheterization
to the cystic duct difficult but does not have true valvular function. The arteries of the gallbladder are
derived from the cystic branch of the hepatic artery which in 95% of cases originates from the right
hepatic artery. From its origin, the cystic artery usually crosses behind the hepatic duct (84% of cases)
but is sometimes anterior to that structure. The cystic artery proceeds to the neck of the gallbladder,
where it divides into anterior and posterior divisions that supply the corresponding areas of the
gallbladder. The cystic veins empty into the right branch of the portal vein indirectly into the liver.

138. The gallbladder plays an important role in altering bile composition by absorption and secretion.
Which of the following statement(s) is/are true concerning this mucosal function?

a. The absorption of water by the gallbladder can result in concentration of the solute components of
bile from 2-to 10-fold
b. Gallbladder mucosal absorption can occur by both active and passive mechanisms
c. Cyclic adenosine monophosphate (cAMP) stimulates NaCl-coupled transport and may also influence
tight junction permeability
d. Secretory products of the gallbladder include bicarbonate and glycoproteins
Answer: a, b, d

The gallbladder rapidly absorbs water and solutes from bile and concentrates the solute components 2-
to 10-fold. The gallbladder has an active mucosa and is able to absorb water and solutes against
significant concentration gradients. Water absorption is linked to the transport of ions. The two major
mechanisms of absorption are active and passive. In passive absorption, sodium and chloride enter the
gallbladder epithelial cells because of electrochemical gradients. This results in an osmotic gradient,
and water flows into the cell. Intracellular sodium is extruded across the basolateral membrane into
the lateral intercellular spaces by active transport. The active transport of sodium against an
electrochemical gradient is associated by an Na+-K+-ATPase pump. Cyclic AMP may inhibit NaCl-
coupled transport and may also influence tight junction permeability. Other peptides, such as secretin,
glucagon, and gastric inhibitory peptide, have been shown to inhibit absorption. Secretion by the
gallbladder occurs either by inhibiting net ion and fluid absorption or with stimulation of bicarbonate
secretory mechanisms. Gallbladder epithelium may also secrete mucin and nonmucin glycoproteins
which may play a role in gallstone formation.
139. Abnormalities of the sphincter of Oddi have been recently recognized to cause symptoms which
are referable to the biliary tree or pancreas. The following statement(s) is/are true concerning
sphincter of Oddi motor function.

a. The sphincter’s basal resting pressure is 10 to 15 mm Hg above duodenal pressure


b. Contraction of the sphincter occurs with CCK stimulation
c. Vagal stimulation results in relaxation of the sphincter
d. Manometry of the sphincter of Oddi may be performed at the time of ERCP to characterize basal
pressure, amplitude, frequency of contraction, and direction of propagation of contractile waves
e. Stenosis of the sphincter of Oddi is characterized by abnormally elevated basal pressure on sphincter
of Oddi manometrics
Answer: a, c, d, e

The sphincter of Oddi is about 4 to 6 mm in length. The sphincter’s basal resting pressure is about 13
mm Hg above duodenal pressure. The sphincter exhibits phasic contractions at a frequency of 4 per
minute and a duration of 8 seconds. The regulation of bile flow is primarily controlled by the sphincter
and not by the surrounding smooth muscle of the duodenum. Relaxation of the sphincter occurs with
CCK stimulation leading to diminished amplitude of phasic contractions and reduced basal pressure,
allowing increased passive flow of bile into the duodenum. Parasympathetic stimulation also causes
intermittent relaxation of the sphincter, and sympathetic splanchnic stimulation causes increased
pressure. Abnormalities of the sphincter of Oddi may cause symptoms which are referable to the biliary
tree or pancreas. Manometry of the sphincter of Oddi may be performed at the time of ERCP to
characterize its basal pressure, the amplitude and frequency of contractions, and the direction of
propagation of contractile waves. Stenosis of the sphincter of Oddi is characterized by abnormally
elevated basal pressure (> 40 mm Hg) whereas dyskinesia is characterized by abnormalities of other
manometric parameters.

140. A 32-year-old woman with symptomatic gallstones wishes to discuss nonsurgical options for her
gallstones. Which of the following statement(s) are true?

a. The best commercially available oral dissolution agent, ursodeoxycholic acid, is associated with a
complete dissolution rate of less than 50%
b. If the gallstones dissolve, there is minimal risk of gallstone recurrence
c. Contact dissolution is applicable regardless of stone type
d. Extracorporial shock wave lithotripsy (ESWL) in combination with oral dissolution agents is an
appropriate technique for most patients and can result in complete stone fragment clearance in over
90% of patients by one year
Answer: a

Prior to the introduction of laparoscopic cholecystectomy, there was tremendous enthusiasm for a
number of nonsurgical techniques for managing gallstones. Dissolution of existing gallstones with
pharmacologic agents has been addressed by several national cooperative studies. Ursodeoxycholic acid
is the best, safest, and most effective commerically available drug currently available. However, it still
has a rate of complete dissolution of only 40%. It is also estimated that only 10% of patients will be
suitable candidates for this therapy. Furthermore, gallstone recurrence is a major problem. Actuarial
life table analysis indicates that the risk of gallstone recurrence in patients who have undergone
dissolution of gallstones with oral bile acid therapy is 50% by five years.
The addition of extracorporial shock wave lithotripsy increases the efficiency of gallstone clearance
and in selected patients complete fragment clearance can be obtained in over 90% of patients by one
year. However, optimal results can be obtained only by setting relatively strict criteria for inclusion. In
applying such criteria, less than 20% of patients in the United States would be considered eligible for
ESWL. Similar problems with gallstone recurrence have been observed with this technique. Finally,
contact dissolution primarily with the ether solvent methyl tert-butyl ether (MTBE) is extremely
effective in dissolving cholesterol gallstones. This technique, however, will work only in stones which
are composed of cholesterol and the patient must have a demonstrably patent cystic duct before
considering this treatment. As with oral dissolution and lithotripsy, gallstone recurrence will remain a
problem with this technique.

141. A 48-year-old woman presents with several hours of acute right upper quadrant pain, low grade
fever, and nausea and vomiting. Which of the following statement(s) is/are true concerning her
diagnosis and management?

a. A mild elevation of her bilirubin (< 3 mg/dl) would strongly suggest a common bile duct stone
b. A positive bile culture can be expected in virtually 100% of patients with this scenario
c. Laparoscopic cholecystectomy is clearly contraindicated
d. Appropriate antibiotic coverage should include coverage for gram-negative aerobes
Answer: d

Acute cholecystitis occurs in about 10% to 20% of patients with symptomatic gallstones. Although the
clinical manifestations of biliary colic and acute cholecystitis may overlap and clinical distinction is
often difficult, persistence of pain beyond a matter of hours and fever would strongly suggest acute
cholecystitis. The primary events in the development of acute cholecystitis are chemical in nature with
bacterial infection playing a minor role in the genesis of the disease. In normal healthy subjects
without gallstones, incidence of positive bile cultures is essentially zero. In contrast, between 30 and
70% of patients with the clinical diagnosis of acute cholecystitis will have positive bile cultures. The
incidence of positive bile cultures who undergo cholecystectomy increases significantly with age.
Septic complications continue to be a significant source of morbidity after cholecystectomy for acute
cholecystitis. These septic complications can best be prevented by the judicious use of appropriate
antimicrobial agents. The goal of antimicrobial therapy should be establishment of adequate serum and
tissue levels of antibiotic rather than selection of an antibiotic that is excreted into the bile. Given the
bacteriology that is typical in patients with uncomplicated cholecystitis, an appropriate antibiotic
regimen should provide for adequate coverage of gram-negative aerobes. Although technically more
difficult, laparoscopic cholecystectomy can be completed safely in the majority of patients with acute
cholecystitis. Significant experience and good judgment, however, is essential in insuring optimal
results.
Laboratory data are often nonspecific with acute cholecystitis. Mild jaundice may be present in up to
20% of patients and is typically due to inflammation as opposed to bile duct obstruction secondary to
stones.

142. Laparoscopic cholecystectomy has become the procedure of choice for the management of
symptomatic gallstones. Which of the following statement(s) is/are true concerning laparoscopic
cholecystectomy?

a. Injury to biliary tree occurs more frequently with laparoscopic cholecystectomy than open
cholecystectomy
b. Laparoscopic cholecystectomy is contraindicated in patients with acute cholecystitis
c. Bile duct injuries are more likely to occur during the surgeon’s early operative experience with the
procedure
d. Previous upper abdominal surgery is an absolute contraindication to laparoscopic cholecystectomy
Answer: a, c

Laparoscopic cholecystectomy was first reported in 1988 and in recent years has supplanted open
cholecystectomy as the treatment of choice in the management of calculous biliary tract disease. As
experience with the technology increases, recommendations regarding contraindications for
laparoscopic cholecystectomy have evolved. Absolute contraindications include the inability to perform
laparoscopic cholecystectomy because of inadequate training or equipment, poor candidate for general
anesthesia, uncontrolled coagulopathy, peritonitis, or suspected gallbladder carcinoma. Factors
previously considered relative contraindications have been identified but with appropriate skill and
judgment do not preclude the laparoscopic technique. These include acute cholecystitis, morbid
obesity, previous upper abdominal surgery, cirrhosis, portal hypertension, and pregnancy. Laparoscopic
cholecystectomy for acute cholecystitis is clearly more difficult and while the conversion rate of
elective laparoscopic cholecystectomy to an open procedure is less than 5% in the setting of acute
cholecystitis, the rate is approximately 20–30%.
The incidence of bile duct injury in large surveys in this country and abroad suggest that the incidence
of bile duct injury with open cholecystectomy is less than 0.2%. Initially much higher, (approaching
1%), several series of laparoscopic cholecystectomy have documented that injuries to the bile ducts
currently occur at the rate of 0.2–0.6% of cases. There is a definite learning curve associated with
performing laparoscopic cholecystectomy with most bile duct injuries likely to occur during a surgeon’s
early operative experience with this procedure.

143. Which of the following statement(s) is/are true concerning gallstone ileus?

a. The diagnosis may be suggested by plain abdominal radiograph


b. Primary surgical management consists of relief of obstruction and cholecystectomy
c. Gallstone ileus accounts for less than 5% of all causes of intestinal obstruction
d. Typical patients are elderly with long-standing gallstone disease
Answer: a, c, d

Cholecystoenteric fistulas are the most common form of gallbladder perforation accounting for 40% of
all cases. Depending on the size of the fistulous communication, a gallstone may pass through this
tract. If the stone is large enough (> 2 cm), it may become lodged in a portion of the gastrointestinal
tract and cause mechanical small bowel obstruction. This condition, called gallstone ileus, is relatively
rare and accounts for fewer than 5% of cases of intestinal obstruction. The diagnosis of gallstone ileus
may be suggested by the presence of intrahepatic biliary air in the setting of a small bowel obstruction
seen on abdominal radiography. Patients with gallstone ileus are best managed as if they had a
mechanical small bowel obstruction with aggressive fluid resuscitation, broad spectrum antibiotics, and
early laparotomy. The primary goals at laparotomy are correction of the obstruction and removal of
the offending gallstone. Since many of the patients are elderly and ill, cholecystectomy and takedown
of the biliary enteric fistula may not be appropriate. Enterolithotomy alone, without cholecystectomy,
has an associated mortality of 5% in contrast to 15% mortality rate in patients who undergo both
procedures at the same time.

144. The pathogenesis of cholesterol gallstones is multifactorial. A number of key processes, however,
appear to interact closely in the formation of cholesterol gallstones. These include:

a. Cholesterol supersaturation
b. Gallbladder stasis
c. Accelerated nucleation
d. High bilirubin concentration
Answer: a, b, c

The development of gallstones is a complex multifactorial process. Although a number of factors may
contribute to gallstone formation, a number of steps are considered critical in the formation of
gallstones. These include cholesterol supersaturation whereby the amount of cholesterol present is in
excess of the ability of the micelles and vesicles to transport cholesterol in a soluble state. In addition,
the process by which cholesterol crystals form and agglomerate is referred to as nucleation. It is likely
that the process of nucleation is altered by a number of biliary proteins or other compounds which may
promote or inhibit the nucleation process. Gallbladder stasis provides the proper environment for
gallbladder crystal precipitation and stone formation. The stagnant pool of bile within the gallbladder
may facilitate both cholesterol supersaturation, the nucleation process, as well as altering the
gallbladder’s absorptive or secretory functions. Although bilirubin has been suggested to play a small
role in cholesterol gallstone formation, perhaps in either nucleation or solubilization of cholesterol,
excessive production of bilirubin as seen in hemolytic disorders is associated primarily with the
formation of pigment gallstones.

145. Which of the following statement(s). is/are true concerning the diagnosis of biliary tract disease?

a. Nonvisualization of the gallbladder on oral cholecystogram is diagnostic of biliary calculous disease


b. Ultrasonography has a diagnostic accuracy and sensitivity for cholelithiasis in excess of 95%
c. Ultrasonography is the preferred test to distinguish chronic from acute cholecystitis
d. Hepatobiliary scintigraphy is primarily indicated to confirm the clinical diagnosis of acute
cholecystitis
Answer: b, d
A number of radiographic studies can be used to diagnose cholelithiasis. Visualization of gallstones on
plain abdominal radiographs is possible in about 20% of patients whose stones are grossly calcified.
Traditionally, oral cholecystogram has been the gold standard for the diagnostic evaluation of patients
with suspected gallstones. Although the accuracy for oral cholecystography has been reported to be as
high as 95%, several conditions preclude satisfactory examination, including acute illness; poor patient
compliance; inability to absorb the tablets as the result of emesis, malabsorption, or diarrhea; and
jaundice or hepatic dysfunction. Abdominal ultrasonography has therefore become the preferred test
for evaluation of patients with suspected gallstones. Most large series suggest that diagnostic accuracy
and sensitivity for cholelithiasis exceeds 95%. In addition to detecting gallstones, information is
provided concerning the size and shape of the gallbladder, gallbladder wall thickness, presence of
pericholcystic fluid, or a sonographic Murphy’s sign. The latter has been suggested to have an 85%
accuracy rate in patients with acute cholecystitis. Nonetheless, while abdominal sonography is most
helpful in identifying the presence or absence of gallstones, it is of limited use in distinguishing chronic
from acute cholecystitis. Hepatobiliary scintigraphy provides information on the patency of the bile
ducts, including the cystic duct. Recognition that cystic duct obstruction is the sine qua non of acute
cholecystitis, failure of the gallbladder to visualize during hepatobiliary scintigraphy is highly
suggestive of acute cholecystitis. Although a number of false positive exams can occur, nonvisualization
of the gallbladder during hepatobiliary scanning in the appropriate clinical setting is highly diagnostic
of acute cholecystitis.

146. Which of the following statement(s) is/are true concerning the solubilization of cholesterol in bile?

a. Cholesterol is highly soluble in both serum and bile


b. Mixed micelles are the primary transport mechanism for biliary cholesterol
c. Most cholesterol found in bile is the result of excretion from serum
d. Biliary vesicles are composed primarily of biliary phospholipid
Answer: d

Bile is secreted by the liver and is composed primarily of water, electrolytes, and organic solutes. Bile
salts, cholesterol, and phospholipids are the main solutes found within bile and account for about 80%
of the dry weight of bile. Most of the cholesterol found in bile is synthesized de novo in the liver.
Cholesterol is an organic molecule that is virtually insoluble in an aqueous medium such as bile.
Therefore, mechanisms for maintaining cholesterol in solution have been evolved. For decades, the
mixed micelle which is composed of the amphiphatic bile salts and phospholipid was considered the
primary carrier of bile. More recently, it has been demonstrated that up to 70% of the total amount of
cholesterol normally found in gallbladder bile is transported and solubilized in the vesicular form. Bile
vesicles are composed primarily of phospholipid of which in the human, lecithin accounts of 90% of the
phospholipid content.

147. Appropriate options for management of common bile duct stones identified at laparoscopic
cholecystectomy include:

a. Conversion to open cholecystectomy and common duct exploration


b. Transcystic duct dilatation and exploration
c. Laparoscopic choledochotomy
d. Complete the laparoscopic cholecystectomy with postoperative ERCP and stone removal
Answer: a, b, d

Common bile duct stones found unexpectedly during laparoscopic procedures presents the surgeon with
a different set of choices from that of stones found during an open procedure where one would
invariably proceed to common duct exploration. The practice of converting to an open procedure for
the purposes of common duct exploration is acceptable and should be considered based on the
surgeon’s experience and expertise and other options available. Transcystic duct dilatation and
exploration with stone removal is being performed in increasing numbers and appears to be safe and
effective. Although experience with laparoscopic choledochotomy and common duct exploration has
been reported with limited experience, this technique is probably not appropriate for the average
surgeon. Many surgeons, depending on their clinical situation, may opt to complete the laparoscopic
removal of the gallbladder and proceed to postoperative management of the stone using endoscopic
technique.

148. Risk factors associated with development of gallstones include:

a. Increasing age
b. Obesity
c. Rapid weight loss
d. Cirrhosis
e. Diabetes mellitus
Answer: a, b, c, d, e
Incidence of gallstones varies widely throughout the world} In the United States, about 10% of the
population has cholelithiasis. Although the physiologic explanation for increasing incidence of gallstone
disease in the elderly is unclear, epidemiologic studies have demonstrated a relationship between
increasing age and the prevalence of cholelithiasis in both males and females. Gallstone disease is a
much more common in women than men, however with the higher incidence of cholelithiasis in women
persisting until the seventh or eighth decade of life, at which time the incidence approaches 20% in
both men and women. Clinical evidence suggests a two to three-fold increase in the incidence of
cholelithiasis among morbidly obese patients compared to age matched normal weight subjects.
Interestingly, rapid weight loss has also been associated with secretion of cholesterol saturated bile
and increased incidence of gallstones. Patients with diabetes mellitus have a two-fold increase of
gallstone disease when compared to non-diabetics. Although the influence of alcohol on gallstone
formation remains poorly defined, autopsy studies suggests that the incidence of cholelithiasis is
significantly increased in patients with alcoholic cirrhosis.

149. Which of the following statement(s) is/are true concerning a 35-year-old woman found to have
asymptomatic gallstones?

a. The patient should undergo attempts at medical dissolution


b. Cholecystectomy should only be performed if the laparoscopic technique is an available option
c. The patient has less than 10% chance of developing significant symptoms over the next five years
d. The risk of gallbladder cancer with stones detected at such a young age, warrants cholecystectomy
Answer: c

The optimal management of individual patients with asymptomatic gallstones has been debated for
years. Most evidence, although still somewhat controversial, would indicate that the natural history of
asymptomatic gallstones is benign and that early or prophylactic cholecystectomy, either laparoscopic
or otherwise, is rarely indicated. It is suggested that less than 10% of patients with asymptomatic
gallstones will develop significant symptoms over a five year period. Similarly, medical dissolution with
oral agents, extracorporial biliary lithotripsy, or contact dissolution, is not indicated in these patients.
The risk of gallbladder cancer is so low as to not warrant cholecystectomy in asymptomatic patients.

150. In which of the following clinical situations is pigment rather than cholesterol gallstones a more
common problem?
a. Alcoholic cirrhosis
b. Morbid obesity
c. Diabetes mellitus
d. Hereditary spherocytosis
e. Biliary tract infection
Answer: a, d, e

World wide, pigment stones are the most common type of calculus found in the gallbladder. In the
United States, although cholesterol calculi predominate, pigment stones constitute about 30% of all
gallbladder stones. Pigment stones are characterized by a high concentration of bilirubin and a low
cholesterol content. The high incidence of pigment gallstones in patients with hemolytic disorders
probably results from the excessive loads of bilirubin being presented to the liver for excretion.
Infection has been thought to be a key factor in the pathogenesis of pigment gallstones in that
unconjugated bilirubin, produced by bacterial deconjugation, is insoluble in water, and combines with
calcium in bile to produce a calcium bilirubin matrix. Specific alterations in bile acid metabolism have
been proposed to explain the increased incidence of pigment gallstones found in cirrhotics. Gallstones
associated with morbid obesity and rapid weight loss appear to be primarily due to the increased
hepatic secretion of cholesterol leading to a cholithogenic state.

151. The following statement(s) is/are true concerning the mode of spread of gallbladder cancer.

a. The most common mode of spread for gallbladder cancer is to the liver by hematogenous spread
b. Lymphatic drainage of the gallbladder can extend from the cystic duct lymph node to periportal
lymph nodes and the celiac and superior mesenteric lymph nodes
c. Gallbladder cancers rarely extend past the cystic duct
d. Direct extension to the liver, particularly liver segments IV and V, is the most common mode of
spread for gallbladder cancer
e. Most patients with gallbladder cancer present with a very limited extent of disease
Answer: b, d

The mode of spread of gallbladder cancer can be predicted by the lymphatic and venous drainage of
the gallbladder. Venous drainage is into the venules that drain directly into the adjacent liver. The
most common spread of gallbladder cancer is through direct extension into the liver, particularly liver
segments IV and V. The lymphatic drainage of the gallbladder is to the cystic duct lymph node, to
periportal lymph nodes, and then to celiac and superior mesenteric lymph nodes. These tumors can
also spread into and around the cystic duct and may extend into the common bile duct, causing biliary
obstruction. Besides direct extension and lymphatic spread of the tumor, distant metastases is possible
but is less commonly seen. A staging system has been developed which takes into consideration
lymphatic and venous drainage of the gallbladder. The most advanced stage, stage V, includes distant
spread, either into the liver or adjacent organs, which unfortunately is the case at the time of
presentation for most patients.

152. A 55-year-old woman undergoes cholecystectomy for symptomatic gallstones. The final pathology
returns one week after the operation showing gallbladder carcinoma. Which of the following
statement(s) is/are true concerning this patient’s prognosis and management?

a. If the gallbladder carcinoma is limited to the mucosa and/or submucosa, cholecystectomy alone is
adequate treatment
b. If the carcinoma involves deeper layers of the gallbladder wall, the prognosis will be significantly
worse
c. If reoperation is performed, the chance of finding residual disease will be low
d. If a full thickness tumor had been recognized at the time of cholecystectomy, an "extended
cholecystectomy" may have been appropriate
Answer: a, b, d

Gallbladder cancers with the best prognosis are those that are found incidentally at the time of
cholecystectomy for symptomatic gallstone disease. This point emphasizes the importance of opening
all gallbladders at the time of cholecystectomy so that any suspicious lesions can be immediately
examined histologically. When gallbladder cancer is limited to mucosa and submucosa,
cholecystectomy is adequate treatment and has a good prognosis, with up to 100% five year survival.
When the cancer involves the deeper layers of the gallbladder wall, the prognosis is more grim. A 64%
five year survival was reported in patients who had carcinoma confined to the mucosa and submucosa,
whereas none of the patients with cancer involving all layers of the gallbladder wall survived more than
2.5 years. Even though these tumors are relatively localized at the time of cholecystectomy,
cholecystectomy alone was not adequate treatment for long-term survival. In hopes of improving these
survival rates and considering the lymphatic and venous drainage of the gallbladder, it has been
recommended that gallbladder cancers be treated by cholecystectomy with wide resection of the liver
around the gallbladder bed (liver segments IV and V) and regional lymphadenectomy. This procedure
has been termed an "extended cholecystectomy." Although this treatment has prolonged median
survival, an overall improvement in five year survival has not been observed. When a gallbladder
cancer has been recognized incidentally at the time of pathology examination after routine
cholecystectomy, reexploration will likely show evidence of residual disease in the lymph nodes or
adjacent liver and therefore resection of this residual disease may improve survival. More radical
excision including hepatic resection and bile duct resection and sometimes even
pancreaticoduodenectomy has been reported. However, the associated increased morbidity and
mortality of these operations has not resulted in an improvement in survival.

153. Gallbladder cancer accounts for 3% to 4% of all gastrointestinal tumors. Which of the following
statement(s) is/are true concerning the incidence of gallbladder cancer?

a. Gallbladder cancer is much more frequent in men


b. Southwest Native Americans, Alaskans, Mexicans, and Hispanics have a greater incidence of
gallbladder cancer than the general population
c. The majority of patients with gallbladder cancer have gallstones
d. The association of gallstones with gallbladder cancer is greater in patients with multiple small
stones
Answer: b, c

Gallbladder cancer accounts for 3% to 4% of all gastrointestinal tract tumors and about 2% of all biliary
procedures are done for gallbladder cancers. Gallbladder cancer is much more frequent in women,
with a female/male ratio of 3:1. The disease is most commonly seen in elderly women with a mean age
of 65 at diagnosis. Southwest Native Americans, Alaskans, Mexicans, and Hispanics living in the United
States have an estimated five to six times greater incidence of gallbladder cancer than the general
population. On the other hand, gallbladder cancer is much less common in blacks. The association of
gallstones with gallbladder cancer is well known. Seventy to ninety percent of all patients with
gallbladder cancers have gallstones, and about 0.4% of all patients with gallstones have gallbladder
cancer. The association of gallstones with cancer may be related to gallstone size; larger stones have a
greater cancer risk. There is a 10-fold increase in the incidence of gallbladder cancer in patients who
have gallstones that are larger than 3 cm in diameter.

154. The following statement(s) is/are true concerning the prognosis for gallbladder cancer.

a. Average survival is in the range of one year


b. Five year survival rates approach 50%
c. The combination of postoperative adjuvant radiation and chemotherapy have been associated with
overall improved survival
d. For most patients, the goal of treatment is palliation
Answer: d

The prognosis for gallbladder cancer remains poor, with an average survival in the range of six months.
Less than 5% of patients survive five years because 90% of gallbladder cancer patients present with
stage V disease. For stage V disease, the goal of treatment is palliation. If these patients present with
obstructive jaundice, a major goal of treatment is relief of jaundice and its attendant symptoms such
as pruritus and cholangitis. Although radiation and chemotherapeutic regimens have been tried, none
have been associated with a good response.

155. Which of the following statement(s) is/are true concerning gallbladder polyps?

a. Sonographic findings of a gallbladder polyp include a filling defect which does not move with change
in position
b. Benign gallbladder polyps can cause symptoms similar to those caused by gallstones
c. Ultrasonography can readily distinguish benign from malignant gallbladder lesions
d. An asymptomatic gallbladder polyp 1 cm in size can usually be followed with serial ultrasounds and
does not require cholecystectomy
Answer: a, b

Benign gallbladder polyps cause symptoms similar to those caused by cholelithiasis. Biliary symptoms
including right upper quadrant pain and discomfort, fatty food intolerance, nausea, vomiting, and an
increase in flatulence are common complaints in patients with symptomatic benign gallbladder polyps.
Gallbladder tumors, when diagnosed preoperatively, may be visualized by ultrasound, oral
cholescintography, or less commonly, computed tomography. A filling defect that does not move with
changes in position is probably a benign or malignant gallbladder polyp. Symptomatic patients who
have lesions in the gallbladder should have a cholecystectomy. Since neither ultrasound nor
cholecystography can distinguish benign from malignant lesions, all gallbladders that contain polypoid
lesions should be removed.

156. The following statement(s) is/are true concerning the surgical therapy of cholangiocarcinomas.
a. A resectable distal bile duct carcinoma is best treated by a Whipple procedure
b. Klatskin tumors may require hepatic resection in attempt for potential cure
c. Regardless of the surgical resection for proximal biliary tumors, stenting of the biliary anastomosis is
important
d. Complete hepatic resection with hepatic transplantation has been associated with overall good
results
Answer: a, b, c

The surgical treatment of a cholangiocarcinoma is dependent upon its location. For lesions of the
middle third of the bile duct, resection of the bile duct tumor with reanastomosis is the procedure of
choice when possible. For distal lesions, the Whipple procedure (pancreaticoduodenectomy) is often
necessary. Overall, both middle and lower third lesions have a better prognosis than tumors in the
hilum. The prognosis for patients with hilar bile duct cancer (Klatskin tumors) is extremely poor with
mortality rates of 80% to 90% in five years. Most patients are unresectable at the time of diagnosis.
Unless contraindicated for other reasons, however, surgical exploration should be performed in all
patients whose tumors are potentially resectable. Hepatic lobectomy is indicated for potential cure if
the bile duct lesion extends into the hepatic parenchyma. Regardless of the surgical therapy, stenting
of the biliary anastomosis is important since postoperative strictures or recurrent tumors are common
and long-standing stents allow for cholangiographic followup and for dilatation should strictures arise.
These stents have also been used for placement of iridium seeds used for local irradiation. More
aggressive surgical therapy including complete hepatic resection followed by hepatic transplantation
have been performed. However, the results have been associated with a very high rate of recurrence in
the post-transplantation period and therefore at the present time this treatment is not recommended.

157. The following statement(s) is/are true concerning carcinoma of the bile ducts.

a. Carcinoma of the bile ducts is more common than gallbladder carcinoma


b. Similar to gallbladder cancer, bile duct cancer is more common in females
c. Unlike gallbladder cancer, there is no association of bile duct cancer and gallstones
d. Ulcerative colitis is associated with an increased incidence of bile duct cancer
e. Choledochal cysts may be associated with the development of bile duct cancer
Answer: d, e

Cancer of the bile ducts is even less common than gallbladder carcinoma and is seen in 0.01% to 0.46%
of all autopsies. Unlike gallbladder cancer, which is more frequent in females, males have a higher
incidence of bile duct cancer. The average age range of diagnosis between is 50 and 70 years. Similar
to gallbladder cancer, there is an association between bile duct cancer and gallstones. This association
is, however, not as direct as for gallbladder cancer with only 25% to 57% of patients with bile duct
cancer having gallstones. Biliary tract infection is associated with these tumors; for example, patients
with Clonorchis sinensis infection and chronic typhoid carriers have a higher incidence of bile duct
cancer than the general population. Choledochal cysts are also associated with bile duct cancer.
Patients with ulcerative colitis have a marked increase incidence of bile duct tumors which tend to
follow a more aggressive course.

158. Which of the following statement(s) is/are true concerning the association of gallstones with
gallbladder cancer?

a. The association of gallbladder cancer with gallstones is such that young patients with asymptomatic
gallstones should undergo routine cholecystectomy
b. Calcification of the wall of the gallbladder is associated with an increased incidence of gallbladder
cancer
c. Larger gallstones are associated with a greater risk of gallbladder cancer than smaller gallstones
d. The 20 year risk of developing gallbladder carcinoma in patients in the general population with
gallstones is approximately 1%
Answer: b, c

The association of gallstones with gallbladder cancer is well known. Seventy to ninety percent of
patients with gallbladder cancer have gallstones. The association of gallstones with cancer may be
related to gallstone size, with larger stones having a greater cancer risk. The 20-year risk of developing
gallbladder carcinoma in patients with gallstones has been estimated to be in the range of 0.13% for
the general population. It is evident that this small risk of developing gallbladder cancer in patients
with gallstones, even over the long-term, does not warrant routine cholecystectomy in all patients with
gallstones. Calcification of the wall of the gallbladder, the so-called porcelain gallbladder, is
associated with a 25% to 60% incidence of gallbladder cancer. In the presence of a porcelain
gallbladder, a cholecystectomy should be performed unless contraindicated for other reasons.

159. A 65-year-old man presents with obstructive jaundice. The patient’s workup begins with a CT
scan. Which of the following statement(s) is/are true concerning his diagnosis?
a. A CT scan demonstrating intrahepatic biliary obstruction with a decompressed gallbladder and a
nondilated extrahepatic biliary tree will be consistent with a Klatskin tumor
b. The presence of biliary obstruction seen on the CT scan requires further evaluation with invasive
cholangiography either percutaneously or endoscopically
c. Percutaneous transhepatic cholangiography would be the preferred technique for a suspected
proximal cholangiocarcinoma in that it will better visualize the proximal extent of the tumor in the
biliary tree
d. The placement of a transhepatic biliary catheter can prove useful in surgical management of
proximal bile duct cancers
e. There is little role for angiography in the evaluation of a patient with suspected cholangiocarcinoma
Answer: a, b, c, d

Jaundice is the most frequent presenting symptom in patients with cholangiocarcinoma, occurring in
90% of patients. The diagnosis begins with abdominal imaging to detect ductal dilatation. A CT
appearance of a proximal cholangiocarcinoma or a Klatskin tumor will show a dilated intrahepatic
biliary tree with a decompressed gallbladder and normal caliber distal bile duct. When biliary
obstruction is present, further visualization of the biliary tree is required either through percutaneous
transhepatic cholangiography (PTC) or endoscopic retrograde cholangiopancreatography (ERCP). PTC is
preferred for more proximal lesions because ERCP may fail to adequately visualize the proximal portion
of the biliary tree. For lower bile duct lesions, ERCP may be the preferred route. The management of
hilar tumors can often be facilitated by the placement of transhepatic percutaneous catheters which
can facilitate both resection, reconstruction, and may serve as a route for administration of local
radiation therapy. These tumors are often found to be unresectable because of direct vascular invasion
into the portal vein or hepatic arteries. Therefore, selective celiac angiography can be helpful
preoperatively to determine if these major adjacent vascular structures are involved with the tumor.

160. Benign gallbladder and bile duct tumors are extremely rare. Which of the following statement(s)
is/are true concerning benign biliary tumors?

a. Cholesterol polyps are due to foamy macrophages filled with cholesterol


b. Inflammatory polyps and cholesterol polyps are not considered to be premalignant
c. Gallbladder adenomas, like adenomas in other gastrointestinal organs, are considered premalignant
d. Most gallbladder adenomas are associated with gallstones
e. An adenomyoma is a mucosal lesion of the gallbladder not considered to be premalignant
Answer: a, b, c
Benign gallbladder and bile duct tumors are extremely rare and can include pseudotumors or
hyperplastic conditions which are thought to result from inflammatory states, or adenomas, which are
most likely premalignant lesions. Cholesterolosis, or a strawberry gallbladder, is manifested by yellow
spots visible on the surface of the gallbladder mucosa. This proliferation of foamy macrophages filled
with cholesterol in the lamina propria may also result in the formation of polyps called cholesterol
polyps. These polyps are thought to result from disturbance in cholesterol metabolism. Other
pseudotumors of the gallbladder are inflammatory polyps which are composed of vascular connective
tissue stalk with a single layer columnar epithelial cells and have a chronic inflammatory infiltrate.
These lesions are not considered to be premalignant and are felt to be the result of chronic
inflammation. Adenomas with hyperplasia of the epithelial layer of the gallbladder, like adenomas in
other gastrointestinal organs can be sessile or papillary. Carcinoma in situ has been reported in these
lesions, and they are thought to be premalignant. The cause of adenomas of the gallbladder is
unknown; but most are not associated with the presence of gallstones. An adenomyoma of the
gallbladder is a rare intramural mass or nodule. This lesion is characterized by a proliferation of the
mucosal epithelium and hypertrophy of the mucosal layers of the gallbladder. Histologically in addition
to muscular hypertrophy, invagination of the epithelial mucosa between muscle layers is evidenced.
These lesions are also not considered premalignant.

161. Which of the following statement(s) concerning bile duct strictures due to chronic pancreatitis
is/are true?

a. Most patients present with progressive jaundice


b. Strictures are classically long and tapered involving the entire intrapancreatic bile duct
c. Patients may be asymptomatic and diagnosed only by persistent elevation of serum alkaline
phosphatase
d. An excellent option for surgical management is choledochoduodenostomy
Answer: b, c, d

The clinical presentation of patients with common bile duct strictures secondary to chronic
pancreatitis is variable. A large number of patients are asymptomatic with a diagnosis of bile duct
strictures suggested only by abnormal liver function tests. The serum alkaline phosphatase appears to
be the most sensitive laboratory finding and is elevated in over 80% of patients. Although in most
cases, transient jaundice may occur, progressive jaundice is rare. Cholangiography will demonstrate a
classic long smooth gradual tapering of the common bile duct throughout its entire intrapancreatic
segment. Biliary reconstruction is the appropriate management for most patients.
Choledochoduodenostomy is preferred by many surgeons because it does not divert bile from the
duodenum, is technically easier to perform, and leaves the jejunum intact for any associated
procedures that may be required for decompression of the obstructed gastrointestinal tract or
pancreatic duct.

162. If a bile duct injury is suspected at laparoscopic cholecystectomy, appropriate management


includes which of the following?

a. Conversion to open cholecystectomy and intraoperative cholangiography


b. Small ducts (< 3 mm) demonstrated by cholangiography to drain a single liver segment can be ligated
c. If the injured segment is greater than 1 cm. in length an end-to-end ductal anastomosis is the
procedure of choice
d. Postoperative external drainage should be avoided
Answer: a, b

In many cases, proper initial management of a bile duct injury recognized at the time of
cholecystectomy can avoid the development of a bile duct stricture. Unfortunately recognition of a bile
duct injury is uncommon during either open or laparoscopic cholecystectomy. It must be emphasized
that should bile leakage be noted or if "a typical" anatomy is encountered during laparoscopic
cholecystectomy, early conversion to an open technique and prompt cholangiography is imperative. If a
segment of accessory duct less than 3 mm has been injured, and cholangiography demonstrates
segmental or sub-segmental drainage of the injured ductal system, simple ligation of the injured duct
is indicated. If the injured duct is 4 mm or larger, however, it is likely to drain multiple hepatic
segments or the entire right or left lobe and thus requires operative repair. If the injured segment of
the bile duct is short (< 1 cm) and the two ends can be opposed without tension, an end-to-end
anastomosis can be performed with placement of a T-tube through a separate choledochotomy either
above or below the anastomosis. For proximal injuries, or if the injured segment of bile duct is greater
than 1 cm in length, an end-to-end bile duct anastomosis should be avoided because of the excessive
tension that usually exists in these situations. The use of Roux-en-Y jejunal limb is preferable for
creation of the anastomosis. Regardless of the type of anastomosis, all repairs at the time of initial
operation should involve some form of external drainage either with a T-tube or intraoperatively-
placed transanastomotic stent.
163. Which of the following statement(s) is/are true concerning the incidence of bile duct injury
following cholecystectomy?

a. Data from the pre-laparoscopic cholecystectomy era would suggest the incidence of bile duct injury
during open cholecystectomy to be 0.1–0.2%
b. The current incidence of bile duct injury during laparoscopic cholecystectomy is greater than 1%
c. The experience of the surgeon performing laparoscopic cholecystectomy can be correlated with the
incidence of bile duct injury
d. Intraoperative cholangiography during laparoscopic cholecystectomy will prevent bile duct injury in
virtually all cases
Answer: a, c

A number of large surveys encompassing thousands of patients undergoing open cholecystectomy would
suggest the incidence of bile duct injury to be 0.1–0.2%. Although a number of early individual series of
laparoscopic cholecystectomy reported bile duct injuries in 1% of patients, as larger series have been
reported and surveys including thousands of patients have appeared, the true incidence would appear
to be 0.3–0.6%. A number of factors are associated with bile duct injury during laparoscopic
cholecystectomy including the experience of the surgeon. This reflects the steep learning curve with
this procedure. Although strongly debated, there is no evidence that intraoperative cholangiography
prevents bile duct injury during laparoscopic cholecystectomy. The use of intraoperative
cholangiography may however detect the injury early in the course of the procedure and thus minimize
the extent of injury.

164. Primary sclerosing cholangitis has a number of treatment options—both medical and surgical.
Which of the following statement(s) is/are true?

a. A number of immunosuppressive oral agents can provide specific effective treatment for primary
sclerosing cholangitis
b. Biliary reconstruction with long-term transanastomotic stents can be useful in selected patients with
focal strictures at the hepatic duct bifurcation
c. Biliary reconstruction should be reserved only for patients with established biliary cirrhosis
d. Hepatic transplantation for primary sclerosing cholangitis can be associated with survival rates
similar to other indications for transplantation
Answer: b, d
There is no known specific, effective medical therapy for primary sclerosing cholangitis. Although
encouraging results from a prospective, randomized, placebo-controlled trial suggests that
ursodeoxycholic acid significantly improves serum liver function tests and clinical symptoms. Because
of the lack of effective medical therapy, an aggressive surgical approach is indicated for most
symptomatic patients with primary sclerosing cholangitis. One surgical approach, in patients with a
predominant stricture at the hepatic duct bifurcation, is resection of the bifurcation and long-term
transhepatic stenting with silastic stents. Results in patients without established cirrhosis are
excellent. However, in those patients with secondary biliary cirrhosis present before surgery,
perioperative morbidity and mortality have been high and long-term results poor. Patients with
established secondary biliary cirrhosis should be referred for hepatic transplantation. Recent reviews of
the experience with hepatic transplantation for primary sclerosing cholangitis would suggest survival to
be similar to those reported for hepatic transplantation for patients with any diagnosis.

165. The following statement(s) regarding the elective repair of a bile duct stricture is/are true:

a. A transanastomotic stent is necessary for a successful result


b. Long-term stenting for approximately one year is necessary for an anastomosis performed at the
distal common hepatic duct
c. A Roux-en-Y hepaticojejunostomy provides the best route for restoring biliary-enteric continuity
d. Preoperatively-placed biliary catheters facilitate dissection and identification of the stricture and
are useful in placement of transanastomotic stents when employed
Answer: c, d

Several principles are associated with successful repair of a biliary stricture. Although many surgeons
favor the use of transanastomotic stents, a number of series have reported successful results without
the use of such stents. The length of stenting depends upon the location of the stricture. If the injury
involves the common bile duct or common hepatic duct at least 2 cm distal to the hepatic duct
bifurcation, and adequate proximal bile duct mucosa can be defined, the use of long-term biliary
stents is not necessary. In these situations transanastomotic stenting for 4–6 weeks postoperatively is
adequate. When adequate proximal bile duct is not available for a good mucosa-to-mucosa
anastomosis, long-term stenting of the biliary-enteric anastomosis with silastic transanastomotic stents
for at least a year is recommended. For established strictures, simple excision and end-to-end
anastomosis or repair of the damaged duct can rarely be accomplished because of the invariable loss of
duct length as a result of fibrosis associated with injury. Thus in almost all cases, hepaticojejunostomy
constructed to a Roux-en-Y limb of jejunum is the preferred procedure. The use of preoperatively-
placed transhepatic biliary catheters can aid in the dissection and identification of the biliary tree
especially in patients with prior attempts at repair where scarring and fibrosis may be significant. In
addition, the biliary catheters can assist in the placement of long-term transanastomotic stents.

166. The vast majority of benign bile duct strictures occur following operations in or near the right
upper quadrant. Other causes of benign bile duct strictures include:

a. Chronic pancreatitis
b. Ulcerative colitis
c. Primary sclerosing cholangitis
d. Intrahepatic arterial infusion of 5-fluorouracil
Answer: a, c, d

Most bile duct strictures are postoperative in nature with over 80% occurring after injury to the bile
duct during cholecystectomy. A number of inflammatory conditions can also result in strictures of the
biliary tree. The chronic inflammation and fibrosis associated with chronic pancreatitis can result in a
stricture of the intrapancreatic bile duct. Primary sclerosing cholangitis is an idiopathic disease
believed to be autoimmune in nature which is characterized by intrahepatic and extrahepatic
inflammatory strictures of the biliary tree. Although primary sclerosing cholangitis is frequently
associated with ulcerative colitis, this colonic disease has no direct causal relationship with benign bile
duct strictures. A rare cause of benign bile duct strictures in both the intrahepatic and extrahepatic
biliary tree has been the use of intrahepatic arterial infusion of 5-fluorouracil used to treat hepatic
metastasis from colorectal carcinoma.

167. A 37-year-old female presents with obstructive jaundice due to a mid-bile duct stricture four
months after laparoscopic cholecystectomy. Which of the following statement(s) are true:

a. Surgical reconstruction is the only option for management of this patient


b. Excellent long-term results can be expected in approximately 80% of patients following surgical
biliary reconstruction
c. One year follow-up after successful repair is satisfactory regardless of the method of management
d. Surgical reconstruction offers a better chance of long-term success than either percutaneous or
endoscopic dilatation
Answer: b, d
Excellent long-term results can be achieved in 70–90% of patients who undergo surgical repair of bile
duct strictures. The definition of satisfactory results in most series requires patients with no symptoms
of jaundice or cholangitis. Length of follow-up is important in analyzing results, however, because
recurrent strictures can occur up to 20 years after the initial procedure. About two-thirds of
restrictures are evident within two years and 90% within seven years. Although operative management
of bile duct strictures in most cases can result in excellent results, the nonoperative approaches of
percutaneous or endoscopic dilatation are suitable alternatives in many patients. Although comparisons
between techniques are difficult, two retrospective comparative studies from single institutions have
suggested that surgical reconstruction offers a better chance of long-term success than either
percutaneous or endoscopic management.

168. Most patients with postoperative bile duct strictures after cholecystectomy present early after
their initial operation. Patients may present in which of the following manner(s)?

a. Obstructive jaundice
b. An external biliary fistula
c. Progressive accumulation of bile in the peritoneal cavity (bile ascites)
d. Biliary cirrhosis
Answer: a, b, c

Most patients with benign postoperative bile duct strictures present early after their initial operation.
Those patients suspected of having postoperative bile duct stricture within days to one week of an
initial operation usually present in one of two ways. One mode of presentation is the progressive
elevation of liver function tests, particularly serum bilirubin and alkaline phosphatase. These changes
can be seen as early as the second or third postoperative day. The second mode of early presentation is
with leakage of bile from the injured bile duct. Bilious drainage from the operatively-placed drains or
through the wound after cholecystectomy is abnormal and represents some form of biliary injury. In
those patients without drains, or in whom the drains have been removed, the bile may leak into the
peritoneal cavity as bile ascites or may loculate as a collection. Patients with markedly delayed
diagnosis of bile duct stricture may present with advanced biliary cirrhosis and other evidence of liver
dysfunction.

169. Nonoperative dilatation, performed either endoscopically or percutaneously, can be successfully


employed in which of the following etiologies of bile duct strictures?

a. Postoperative bile duct strictures following a hepaticojejunostomy used for reconstruction during a
Whipple procedure
b. Complete transection of the bile duct during laparoscopic cholecystectomy (the so-called "classic
laparoscopic cholecystectomy injury")
c. Primary sclerosing cholangitis
d. Oriental cholangiohepatitis
Answer: a, c

Nonoperative management of bile duct strictures is an available option at most institutions, however,
it has some technical limitations due to the anatomic situation. In the so-called "classic laparoscopic
bile duct injury" however, complete bile duct transection and discontinuity of the biliary tree
eliminates the possibility of nonoperative management. Percutaneous dilatation of biliary-enteric
anastomosis has been shown in a number of series to have a success rate approaching that of surgical
reconstruction. Although limited experience with either percutaneous or endoscopic dilatation in
primary sclerosing cholangitis has been reported, this alternative may provide at least temporary
improvement in symptoms and radiologic appearance. Oriental cholangiohepatitis is an unusual
infection of the biliary tree frequently associated with Clonorchis sinensis and other parasites.
Cholangiography will demonstrate multiple strictures of both the intrahepatic and extrahepatic biliary
tree with bile ducts filled with sludge and stones. Surgical management consisting of cholecystectomy
and improving biliary drainage with either a Roux-en-Y choledochojejunostomy or
choledochoduodenostomy is necessary in almost all patients. Access to the biliary tree for
postoperative management of intrahepatic stones or sludge should be maintained, however, with
transhepatic biliary stents.

170. The gold standard for evaluation of patients with bile duct strictures is cholangiography. The two
routes for cholangiography are percutaneous transhepatic cholangiography (PTC) or endoscopic
retrograde cholangiography (ERC). Which of the following statement(s) is/are true?

a. PTC is generally more valuable than ERC in defining the proximal biliary tree to be used in
reconstruction
b. ERC is technically easier in patients with bile leaks because the biliary tree is usually not dilated
c. Parenteral antibiotics should be administered prior to either procedure to prevent cholangitis
d. Biliary stents can be placed via either technique to control biliary leaks
Answer: a, b, c, d

The gold standard for evaluation of patients with a bile duct stricture is cholangiography. PTC is
generally more valuable than ERC in that it defines the anatomy of the proximal biliary tree that is
used in the surgical reconstruction. ERC is often less useful than PTC because the discontinuity of the
extrahepatic biliary tree usually prevents adequate filling of the proximal biliary tree. However, in
patients with biliary fistulas, the proximal biliary tree is often not dilated making PTC somewhat
technically more challenging. Parenteral antibiotics should be administered prior to either procedure
to decrease the risk of cholangitis. Biliary stents used either to temporarily control biliary leaks or to
stent a stricture after nonoperative dilatation can be placed by either the percutaneous or endoscopic
route.

171. The management of a suspected bile duct injury depends on a number of factors, most
importantly the mode and timing of presentation. Which of the following statement(s) is/are true
concerning a patient presenting with a suspected bile leak after laparoscopic cholecystectomy?

a. Laparotomy should be performed immediately


b. Cholangiography should be performed to determine the nature of the injury
c. Operatively-placed drains should be removed to allow the fistula to close
d. The patient should be discharged to home to allow the leak to close spontaneously
Answer: b

Patients presenting in the early postoperative period with a biliary leak may be septic with either
cholangitis or intra-abdominal bile collections. Sepsis must be controlled first with broad-spectrum
parenteral antibiotics, cholangiography with percutaneous biliary drainage and percutaneous or
operative drainage of biliary leaks. Once sepsis is controlled, there is no hurry in proceeding with
surgical reconstruction of the bile duct stricture. The combination of proximal biliary decompression
and external drainage allows most biliary fistulas to be controlled or even closed. At that time the
external drains may be removed. The patients can then be discharged to home to allow several months
to lapse for resolution of the inflammation in the periportal region and recovery of overall health
status.

Posted by MedCosmosat 6:57 PM
Labels: MCQ : Liver-Pancreas
0 comments:

Post a Comment

Newer PostOlder PostHome

Subscribe to: Post Comments (Atom)


MedCosmos Medical Blogs

 .All MedCosmos Series

Labels
 History Taking (6)
 Important Eponyms (1)
 MCQ : Breast (1)
 MCQ : Cardiac Surgery (1)
 MCQ : Endocrine (1)
 MCQ : General Surgery (1)
 MCQ : Hernia-Acute Abdomen (1)
 MCQ : Large Intenstine (1)
 MCQ : Liver-Pancreas (1)
 MCQ : NeuroSurgery (1)
 MCQ : Oncology (1)
 MCQ : Pediatric Surgery (1)
 MCQ : Small Intenstine (1)
 MCQ : Spleen (1)
 MCQ : Stomach-Esophagus (1)
 MCQ : Thoracic Surgery (1)
 MCQ : Thyroid Gland (1)
 MCQ : Transplantation (1)
 MCQ : Trauma and Burns (1)
 MCQ : Urology (1)
 MCQ : Vascular Surgery (1)
 Surgery Books (7)
 Surgery Lectures (1)
 Surgery Must Read (1)
 Surgery Pamphlets (2)

Blog Archive
 ▼  2008 (48)
o ▼  September (48)

 Paradoxical Aciduria
 Thoracic Surgery MCQ
 Cardiac Surgery MCQ
 NeuroSurgery MCQ
 Urology MCQ
 Pediatric Surgery MCQ
 Vascular Surgery MCQ
 Liver and Pancreas MCQ
 Large Intenstine MCQ
 Stomach and Esophagus MCQ
 Hernia and Acute Abdomen MCQ
 Trauma and Burns MCQ
 General Surgery MCQ
 Thyroid Gland MCQ
 Small Intenstine MCQ
 Oncology MCQ
 Transplantation MCQ
 Enocrine Surgery MCQ
 Breast MCQ
 Spleen MCQ
 Technical Basis Of Radiation Therapy
 Metastasis of Prostate Cancer
 Schwartz Manual Surgery 8th Edition
 Bailey & Love's Short Practice of Surgery
 Sabiston Textbook of Surgery
 A History of Plastic Surgery
 Endoscopic Surgery of Potential anatomical spaces
 1000 Eponyms in Surgery
 Sentinel Lymph Node
 History & Examination in Rheumatoid Arthritis
 Varicose veins examination
 Examination for a diabetic foot
 Thyroid Examination
 Lump History & Examination
 Named Hernia
 Deep Vein Thrombosis
 Post Operative Fever
 Staging of breast cancer
 PERIPHERAL VASCULAR DISEASE History
 Q-A Scrotal Swelling
 Management of Differentiated Thyroid Carcinoma
 Top 100 Secrets in Surgery
 Burns
 Sister Mary Joseph Nodule
 DD of rectal bleeding
 DD of scrotal swelling
 DD of breast lumps
 Lots of Surgery Pamphlets

Você também pode gostar